Mastering A&P 2

Réussis tes devoirs et examens dès maintenant avec Quizwiz!

An increase in sympathetic stimulation of the heart would increase stroke volume by increasing __________.

Contractility Correct Increased sympathetic activity increases heart contractility. This causes cardiac fibers to contract more forcefully at all levels of preload. Regardless of end diastolic volume, this mechanism increases stroke volume by reducing end systolic volume.

Review the sequence of events involved in sperm penetration of the secondary oocyte in Focus Figure 28.1. Place the events of sperm penetration leading to fertilization in the correct sequence, from left to right and without overlapping any steps.

Correct After sperm penetration, the secondary oocyte completes meiosis II, forming the ovum. Fertilization occurs in the uterine tube when chromosomes of the sperm (male) and ovum (female) pronuclei combine to form a zygote.

What is the most common cation found in the interstitial fluid? Na+ HPO4 2- K+ cl-

Na+

Which layer of the typical blood vessel is constructed from simple squamous epithelium?

Tunica Intima Correct The tunic intima of blood vessels, like the endocardium of the heart, is made of simple squamous epithelium.

The process of voiding the bladder is called _______.

micturition

The basic functional unit of the kidney is the _______. -major calyx -loop of Henle -glomerulus -nephron -renal corpuscle

nephron

Diuresis peaks __________ after drinking water. ten minutes half an hour one hour three hours

one hour

The P wave on an electrocardiogram represents __________. -ventricular depolarization -atrial depolarization -ventricular repolarization -atrial repolarization

-atrial depolarization Correct The P wave reflects the depolarization of the atria.

Which of the cell types would secrete their product if the blood glucose level was rising, as during the digestion of a meal? -alpha cells -beta cells -pancreatic acinar cells

-beta cells Correct Beta cells secrete when the blood sugar rises. Their hormone allows body cells to take up glucose from the blood.

Erythropoietin (EPO) stimulates the developmental process shown here. What part of the body does erythropoietin (EPO) target to increase erythropoiesis? -kidneys -lungs -liver -bone marrow

-bone marrow Correct The bone marrow, specifically red marrow, is the site of blood cell production and is stimulated by erythropoietin.

The amplification of the signal from a water-soluble hormone is achieved through an increase in _______. -plasma membrane receptors -water-soluble hormone in the blood -cAMP in the cytoplasm -adenylate cyclase in the plasma membrane -phosphodiesterase in the cytoplasm

-cAMP in the cytoplasm Correct Many cAMP can be generated as a second messenger to amplify the signal in response to hormone binding.

What is the target of thyroid hormones? -hypothalamus -anterior pituitary -thyroid -cells of the body

-cells of the body Correct Thyroid hormones secreted by the thyroid gland target cells of the body to modulate metabolism.

What keeps intracellular receptors from binding to DNA before a hormone binds to the receptor? -Receptors can't enter the nucleus until the hormone is bound to it. -transcription factors -chaperone proteins (chaperonins)

-chaperone proteins (chaperonins) Correct Yes, each receptor has two binding sites. The chaperone protein blocks the DNA binding site until a hormone binds at the hormone binding site.

What is the function of the ventral hypothalamic neurons? -control secretion of oxytocin -control secretion of thyroid hormones -control secretion of thyroid stimulating hormone (TSH) -control secretion of antidiuretic hormone (ADH)

-control secretion of thyroid stimulating hormone (TSH) Correct Yes, neurohormones from the ventral hypothalamic neurons, known as releasing (RH) and inhibiting (IH) hormones, control the release of anterior pituitary hormones such as adrenocorticotropic hormone (ACTH) and TSH.

What ovarian structure forms at the beginning of the secretory phase, encouraging its progress, and then degenerates just before menses? -corpus albicans -vesicular follicle -corpus luteum -primary follicle

-corpus luteum Correct: The corpus luteum forms at the beginning of the secretory phase and produces hormones that support the uterus. When it degenerates about 12 days later, the uterus enters menses.

Which of the following hormones has intracellular receptors? -epinephrine -insulin -cortisol

-cortisol Correct Yes, cortisol is one of the lipid-soluble steroid hormones. Thyroid hormones are also lipid soluble.

Which type of leukocyte provides protection against parasitic worms? -neutrophil -basophil -eosinophil -lymphocyte

-eosinophil Correct The most important role of eosinophils is to lead the counterattack against parasitic worms, such as flatworms and roundworms that are too large to be phagocytized.

Which of the following would NOT lead to a bleeding disorder? -vitamin K deficiency -thrombocytopenia -excess calcium in the diet -impaired liver function

-excess calcium in the diet Correct Calcium is essential for clotting pathways; a deficiency (not an excess) of calcium could lead to a bleeding disorder.

Which plasma constituent is the main contributor to clotting? -albumin -alpha globulins -beta globulins -fibrinogen

-fibrinogen Correct Fibrinogen comprises 4% of plasma and is an important component in the process of blood clotting.

How many oxygen molecules can be transported by one hemoglobin molecule? -two -four -eight

-four Correct Each iron atom on a heme group can bind reversibly with one oxygen molecule.

The __________ develop into the female duct system. mesonephric ducts urethral folds gonadal ridges paramesonephric ducts

paramesonephric ducts The mesonephric ducts develop into the male duct system, and the paramesonephric ducts develop into the female duct system.

Which of the following is not a physical factor that influences pulmonary ventilation? -partial pressure of oxygen in the air -alveolar surface tension -airway resistance -lung compliance

partial pressure of oxygen in the air Correct Pulmonary ventilation is affected by pressure of air in various respiratory structures. Partial pressures of individual gases in the air affect the diffusion and dissolving of these gasses into and out of the blood.

Which of the following plasma proteins is improperly matched with its function? -fibrinogen: clotting -alpha globulin: transport of metal ions and fat-soluble vitamins -gamma globulins: lipid transport -albumin: osmotic pressure

-gamma globulins: lipid transport Correct Gamma globulins are also called immunoglobulins (or antibodies) and are part of the immune response.

Your patient has been diagnosed with Type I diabetes mellitus, and you are explaining how to administer insulin. Your patient states, "I don't want to do this. My brother-in-law has diabetes, and he just takes a pill and watches what he eats." What is your best response? -"Perhaps you can switch to pills and a special diet once your diabetes is under control." -"Great. Maybe your relative can give you some tips on how to manage your condition." -"I'll ask the doctor if you can try an oral pill and a low carbohydrate diet before using insulin." -"Your relative may have Type II diabetes. People with that condition can make insulin, but their cells don't respond to it properly. Your pancreas doesn't manufacture insulin"

-"Your relative may have Type II diabetes. People with that condition can make insulin, but their cells dont respond to it properly. Your pancreas doesn't manufacture insulin" Correct A simple explanation of the differences between Type I and Type II diabetes may help your patient to understand the importance of the prescribed insulin regimen.

What percentage of sperm carry the Y chromosome?

50%

As many as _______ sperm can be released with each ejaculation.

750 million

Which of the following processes in urine formation is important for regulating blood pH?

Secretion

Which of the following is a source of dietary fiber and promotes timely movement through the colon? cellulose polypeptides amylase triglycerides

cellulose

A ruptured follicle is transformed into the:

corpus luteum

Which of the following is not a layer of the ureter?

endothelium

Which of the following would likely be increased by the release of atrial natriuretic peptide (ANP)? -blood pressure -aldosterone production -water excretion by the kidneys -antidiuretic hormone (ADH) production

water excretion by the kidneys ANP inhibits reabsorption in the kidney tubules, which then results in an increase in water excretion. ANP also directly inhibits the release of ADH, renin, and aldosterone.

Which of the following adrenergic receptors increase cAMP levels? α2 receptors α1 receptors β receptors

β receptors Correct Yes, these receptors are coupled to adenylate cyclase by Gs receptors that increase cAMP.

Assume a person is experiencing a hemorrhage and the HPc has dropped to 23 mm Hg at the arteriole end of the capillary. Calculate net filtration pressure (NFP) at the arteriole end of the capillary.

-2 mm Hg Correct The net filtration pressure would now be negative as a result of the blood pressure drop, which would result in little or no fluid moving across capillary walls into the interstitial fluid at the arteriole end. Net filtration pressure (NFP) for the arteriole end of the capillary = (HPc + OPif) − (HPif + OPc) = (23 + 1) − (0 + 26).

Determine which ECG shows a normal sinus rhythm. -A -B -C -D

-A Correct All waves are clearly present in this ECG of a normal sinus rhythm.

Mr. Watkins's blood type was determined to be A positive. Which of the following types of blood can he receive? -A positive, A negative, O positive, and O negative. -Only A positive and O negative. -A positive but not A negative. -A positive, O positive, and AB positive.

-A positive, A negative, O positive, and O negative Correct He can safely receive A and O blood.

Which of these would be an effect of an excess of thyroid hormones? -the thyroid would swell and produce a goiter -a would release less TRH -More thyroid hormone would be produced -B would release more TSH

-A would release less TRH Correct Excess thyroid hormone would have an inhibitory effect on A. This is a negative feedback mechanism.

Which ABO blood type is considered to be the universal recipient? -AB -B -O -A

-AB Correct Type AB blood is considered to be the universal recipient because neither anti-A or anti-B antibodies (agglutinins) are present.

Hypocalcemia could be caused by the ______. -apoptosis of parathyroid cells -failure of osteoclasts to respond to PTH (parathyroid hormone/parathormone) -malfunction of the parathormone receptors in kidney tubule cells -All of the listed responses are correct.

-All of the listed responses are correct Correct Parathyroid hormone (parathormone) is secreted in response to hypocalcemia. It functions to stimulate osteoclast activity, enhances the reabsorption of calcium by the kidneys, and increases absorption of calcium ions by the intestinal mucosal cells.

Determine which of the following electrocardiogram (ECG) tracings is missing P waves but is otherwise regular. -A -B -C -D

-B Correct B illustrates a QRS complex and a T wave, but it is missing the P wave.

Which of the following scenarios could result in HDN (hemolytic disease of the newborn)? -A-positive female pregnant with a B-positive baby -B-negative female pregnant with an AB-positive baby -AB-negative female pregnant with an AB-negative baby -O-positive female pregnant with a B-positive baby

-B-negative female pregnant with an AB-positive baby Correct Hemolytic disease of the newborn (HDN) can develop when an Rh-negative mother carries an Rh-positive fetus. Unlike anti-A and anti-B antibodies, anti-Rh antibodies are not preformed (i.e., a person doesn't produce them until they've been exposed to Rh-antigens for the first time). This initial exposure can occur when the Rh-negative mother carries her first Rh-positive baby. The first such pregnancy usually results in the delivery of a healthy baby. But during this pregnancy and particularly during delivery, the mother may be sensitized by her baby's Rh-positive antigens that pass into her bloodstream. If so, she will form anti-Rh antibodies unless treated with RhoGAM during pregnancy and around the time of delivery. RhoGAM is a serum containing anti-Rh antibodies. By agglutinating the Rh factor, it blocks the mother's immune response and prevents her sensitization. If the mother is not treated and becomes pregnant again with another Rh-positive baby, her antibodies will cross through the placenta and destroy the baby's RBCs, producing the condition known as hemolytic disease of the newborn (HDN).

Choose the incompatible transfusion. -Donate type O blood to a recipient with type AB blood. -Donate type B blood to a recipient with type AB blood. -Donate type B blood to a recipient with type O blood. -Donate type A blood to a recipient with type AB blood.

-Donate type B blood to a recipient with type O blood. Correct A person with type B blood can only give blood to people of type AB and B blood. The type O recipient has anti-A and anti-B antibodies in their plasma, which would produce a transfusion reaction with the B-antigens from the type B blood.

Part complete Insulin-like growth factors (IGFs) are intermediary hormones stimulated by which of the following hormones? -thyroid hormones -GH (growth hormone) -oxytocin -prolactin (PRL)

-GH (growth hormone) Correct Yes, GH stimulates IGFs from the liver. IGFs are required for the growth effect of GH on bone and skin.

Your patient has a hematocrit of 56%. You recall from your anatomy and physiology class that high hematocrits are often a sign of blood doping by athletes. However, your patient is an elderly man suffering from the flu. What is most likely accounting for his elevated hematocrit? -Your patient is on chemo and it is causing him to produce too many red blood cells. -He recently donated blood. -He is leaking blood into his abdominal cavity. -He is dehydrated from the flu.

-He is dehydrated from the flu. Correct If your patient is severely ill and not drinking enough fluids, especially if he has a fever, the ratio of cells to fluid will be shifted.

How do endocrine hormones reach their target cells? -Hormones are released at synapses adjacent to target cells. -Hormones are transported through the bloodstream to target cells. -Ducts transport hormones directly to target cells. -Hormones travel through the lymphatic system to target cells. -Hormones are produced by endocrine cells that are adjacent to target cells.

-Hormones are transported through the bloodstream to target cells Correct The bloodstream allows hormones to be distributed throughout the body.

Which second messenger causes the release of calcium from the endoplasmic reticulum? -cAMP -DAG -IP3 -tyrosine kinase

-IP3 Correct Yes, inositol trisphosphate releases calcium from intracellular storage sites

Why was Mr. Watkins given PRBCs in addition to normal saline solution? What problem does the infusion of PRBCs address that the saline solution could not? -PRBCs contain hemoglobin, so they are able to carry oxygen to the tissues. -PRBCs enhance clotting, sealing off the injured vessels. -PRBCs increase blood osmotic pressure, making the infused fluid stay in the vessels rather than move into the cells. -PRBCs help restore blood ion composition to normal.

-PRBCs contain hemoglobin, so they are able to carry oxygen to the tissues. Correct The PRBCs contain oxygen-carrying hemoglobin. While the saline replaces lost blood volume, it cannot replace the hemoglobin in the lost RBCs.

Which of these muscles is particularly associated with anchoring the right and left atrioventricular valves? -papillary muscles -pectinate muscles -myocardium -trabecular carneae

-Papillary Muscles Papillary muscles contract to tense the right and left atrioventricular valves via the chordae tendineae just before ventricular systole.

Which of the following enzymes are important in the deactivation of cAMP and termination of signaling? -G Protein -Protein Kinase -Adenylate Cyclase -Phosphodiesterase

-Phosphodiesterase Correct The increase in cAMP levels is usually short-lived because the enzyme phosphodiesterase is constantly present in the cytoplasm of the target cells.

Which of these vessels returns blood to the left atrium of the heart? -Pulmonary Veins -Superior Vena Cava -Pulmonary Trunk -Coronary Sinus

-Pulmonary Veins The pulmonary veins transport oxygen-enriched blood from the lungs to the left atrium

Which portion of the electrocardiogram represents the time during which the ventricles are in systole? -P wave -T wave -Q-T interval -QRS complex

-Q-T interval Correct The Q-T interval is the period from the beginning of ventricular depolarization through ventricular repolarization, during which the ventricles are in systole.

Before Baby A was born, she was not sending much blood to her lungs. Instead, the blood passed through the septal defect into the left atrium, which sent it to the left ventricle and to her body. But why didn't the baby die if no blood was going through her lungs? -She sent her blood out into her mother's body and through her mother's lung instead. -She as so small that the small amount of blood that did go through her lungs was enough. -She was getting her oxygen from the placenta, not from breathing -Before the last trimester, babies use anaerobic respiration, so she did not need oxygen

-She was getting her oxygen from the placenta, not from breathing Correct Before birth, Baby A was fine with an atrial septal defect. In fact, she could not have lived without one −− her heart would have worn out from trying to push blood through the collapsed lungs. Instead, she pushed blood through the umbilical artery into the placenta and picked up oxygen that diffused across from her mother's blood.

Krista's treatment will likely involve destroying or surgically removing her thyroid gland. What effect will this have on her level of TSH? -TSH will increase -it will have no effect because TSH is not secreted by the thyroid -TSH will decrease

-TSH will increase Correct Destroying or removing the thyroid gland will cause thyroid hormone levels to plummet. This will cause an increase in the secretion of TSH from the pituitary.

Which of these cells would be most effective in the disposal of inspired microorganisms that may enter the alveoli? -alveolar macrophages -type I alveolar cells -lymphocytes in blood circulating through the lungs -type II alveolar cells

-alveolar macrophages Correct Alveolar macrophages wander freely, ingesting and destroying invading microorganisms or foreign matter.

During reabsorption of water in the proximal convoluted tubule, what causes water to diffuse from the lumen into the interstitial space? -an increase in the osmolarity of the interstitium -active transport of water -a decrease in the osmolarity of the interstitium

-an increase in the osmolarity of the interstitium Correct Yes, the increase in osmolarity causes the water to move into the interstitium.

Drag and drop the labels into the appropriate blanks in the sentences. Each term will be used once.

-The semilunar valves open during the ventricular ejection phase of the cardiac cycle, allowing blood to exit the ventricles. -End diastolic volume and the first heart sound are observed during the isovolumetric contraction phase of the cardiac cycle. -The P wave and subsequent atrial contraction are evident during the ventricular filling phase of the cardiac cycle. -The dicrotic notch occurs during the isovolumetric relaxation phase of the cardiac cycle. Correct The cardiac cycle reflects a series of electrical and mechanical events; these changes can be observed on a series of graphs. The electrocardiogram (ECG) tracing reflects atrial and ventricular depolarization and repolarization. The P wave corresponds to atrial depolarization, the QRS complex to ventricular depolarization (and also atrial repolarization), and the T wave reflects ventricular repolarization. Pressure changes in the left ventricle, aorta, and left atria are correlated to changes in volume as blood moves through the four chambers of the heart; there is passive blood flow and the myocardial layer of the heart is responsible for contraction, or systole, and relaxation, or diastole. Two sets of valves—atrioventricular and semilunar—regulate blood flow and keep blood moving in a one-way direction. As these valves open and close, observe the changes in the pressure and volume graphs as well as the heart sounds that are produced.

Which of the following statements about myoid cells is true? -They form a duct that conveys sperm from the epididymis to the prostate gland. -When they contract, they cause wrinkling of the scrotal skin. -They surround the seminiferous tubules and contract rhythmically to squeeze sperm and testicular fluid through the tubules out of the testes. They elevate the testes.

-They surround the seminiferous tubules and contract rhythmically to squeeze sperm and testicular fluid through the tubules out of the testes. right answer feedback:This is the function of the myoid cells.

Which of the following statements is true? -A person with type O blood produces both type A and type B antigens (agglutinogens). -A person with type AB blood produces both anti-A and anti-B antibodies (agglutinins). -Type A blood can safely be given to recipients with type B blood. -Type O blood can safely be given to recipients with type B blood.

-Type O blood can safely be given to recipients with type B blood. Correct Type O blood, which does not contain either A or B agglutinogens, will not cross react with the anti-A agglutinins found in a recipient person with type B blood.

Part complete Under normal conditions, increased levels of thyroid hormone in the blood will cause _______. -an increase in TSH levels -a decrease in TSH levels -an increase in thyrotropin releasing hormone (TRH) -no change in TSH levels

-a decrease in TSH levels Correct Increased thyroid hormone levels will inhibit the release of TSH from the pituitary.

Which of the following would result from hypoparathyroidism? -a decrease in the release of calcium from bones -an increase in calcium absorption from food -an increase in calcium ion in circulating blood -an increase in calcium absorption in the kidney tubule

-a decrease in the release of calcium from bones Correct Hypoparathyroidism leads to reduced osteoclast activity in bones. The resulting lack of calcium in the blood increases excitability of neurons and muscles, producing tingling sensations.

If embryonic testes do not produce testosterone, a genetic male develops female external genitalia. However, if embryonic testes properly produce testosterone, but a genetic male nevertheless develops female external genitalia, the cause could be ______. -a mutation in the gene that codes for development of the paramesonephric ducts -a mutation in the gene that codes for the testosterone receptor -hyposecretion of MIF -hypersecretion of MIF

-a mutation in the gene that codes for the testosterone receptor Correct: Testosterone must bind to its receptor to induce the development of male reproductive organs by inducing changes in gene transcription. Return to Assignment

Which is the correct order of events for hormones activating Gs proteins? -activation of a G protein, tyrosine kinase receptor, phosphorylation of intracellular proteins -activation of G protein, binding of GTP, activation of adenylate cyclase, conversion of ATP to cAMP -activation of G protein, binding of GTP, activation of phospholipase C, activation of DAG and IP

-activation of G protein, binding of GTP, activation of adenylate cyclase, conversion of ATP to cAMP Correct Yes, Gs proteins are stimulatory proteins that activate cAMP.

What is the mechanism of action of lipid-soluble hormones? -increasing protein kinases -activation of genes, which increases protein synthesis in the cell -phosphorylation of intracellular proteins

-activation of genes, which increases protein synthesis in the cell Correct Yes, lipid-soluble hormones diffuse into the nucleus or they diffuse into the cytoplasm and then move into the nucleus, where they affect transcription and translation.

After a lipid-soluble hormone is bound to its intracellular receptor, what does the hormone complex do? -directly alters protein synthesis at the ribosome -acts as a transcription factor and binds to DNA, activating a gene -activates a protein kinase -phosphorylates a protein

-acts as transcription factor and binds to DNA, activating a gene Correct Yes, then mRNA is synthesized.

Where is the thyroid gland located? -at the base of the brain near the hypothalamus -posterior to the sternum in the thoracic cavity -adjacent to the trachea in the neck -posterior to the parathyroid glands in the neck

-adjacent to the trachea in the neck Correct The thyroid is a butterfly or H-shaped gland in the anterior region of the neck just inferior to the larynx, with the lobes extending to each side of the trachea.

What tropic hormone stimulates cortisol from the adrenal gland? -adrenocorticotropic hormone (ACTH) -luteinizing hormone (LH) and follicle-stimulating hormone (FSH) growth hormone (GH) -thyroid-stimulating hormone (TSH)

-adrenocorticotropic hormone (ACTH) Correct Yes, ACTH stimulates the adrenal cortex to produce cortisol. Stress and ACTH are the main stimuli for cortisol.

Suppose Krista's hyperthyroidism was a result of a tumor that causes the anterior pituitary gland to become overactive. Which of the following would Krista's lab reports likely show? -low thyroid hormone, high TSH -high thyroid hormone, low TSH -high thyroid hormone, high TSH -low thyroid hormone, low TSH

-high thyroid hormone, high TSH Correct An increase in TSH from the anterior pituitary will increase levels of thyroid hormone. The high levels of thyroid hormone will not be able to inhibit the anterior pituitary by negative feedback due to the presence of the tumor.

The body's tendency to maintain relatively constant internal conditions is called -homeostasis -positive feedback -diabetes -negative feedback -none of the above

-homeostasis

Which of the following is not a function of blood? -protection from infection -transport of metabolic wastes -hormone production -homeostatic regulation

-hormone production Correct The blood does not produce hormones but does provide for their transport throughout the body.

What type of stimulation controls parathyroid release? -paracrine -hormonal -neural -humoral

-humoral Correct The parathyroid is stimulated by the actual level of calcium in the blood rather than by neural or hormonal stimulus.

Which of the following does NOT stimulate erythrocyte production? -hyperventilating -a drop in blood oxygen levels -erythroprotein -testosterone

-hyperventilating Correct Hyperventilating leads to an above-normal level of oxygen in the blood, which would not stimulate RBC production.

Acromegaly may be caused by all EXCEPT which of the following? -hypersecretion of GHRH (growth hormone-releasing hormone_ -pancreatic tumor -lack of negative feedback by insulin-like growth factors -hyposecretion of GH in adulthood

-hyposecretion of GH in adulthood Correct Acromegaly is a disorder resulting from excessive GH secretion after epiphyseal plates have closed. There are usually no adverse effects of GH hyposecretion in adults. However, GH hyposecretion in children slows the growth of long bones and, unless treated, results in pituitary dwarfism. Growth hormone secretion is stimulated by GHRH (growth hormone-releasing hormone).

Where is thyrotropin-releasing (TRH) made? -follicular cells of the thyroid gland -posterior pituitary -hypothalamus -anterior pituitary

-hypothalamus Correct The hypothalamus regulates the release of several hormones by producing releasing and inhibitory hormones, including thyrotropin-releasing hormone (TRH).

Where is antidiuretic hormone (ADH), also known as vasopressin, synthesized? -kidney -hypothalamus -posterior pituitary -anterior pituitary

-hypothalamus Correct Yes, ADH is synthesized mainly in the supraoptic nucleus of the hypothalamus. It is stored in the posterior pituitary in axon terminals.

As the adrenal gland releases glucocorticoids, which of the following effects may be observed? Select all the answers that apply.

-increased blood glucose levels -increased blood levels of amino acids -decrease in immune system function -lipid breakdown in the liver will increase Correct With long-term stress, the glucocorticoid hormones, chiefly cortisol, are instrumental in the body. The primary targets include the liver, resulting in increased gluconeogenesis and increased catabolism of lipids and proteins. In larger amounts, the depression of the immune system can result.

Adrenocortical androgens are normally converted in females into estrogens. However, in adrenogenital syndrome, females develop a beard and a masculine pattern of body hair distribution; this occurs due to ______. -negative feedback that inhibits estrogen production -conversion of estrogens into testosterone -insufficient level of enzymes that convert androgens into estrogens -increased testosterone secretion

-insufficient level of enzymes that convert androgens into estrogens Correct During the end steps of sex hormone, synthesis androstendione is converted into either testosterone or estrogen. If the body lacks the enzyme to convert androstendione to estrogen, the remaining pathway to testosterone will predominate.

A blow to the head may cause diabetes insipidus by ______. -triggering the hypersecretion of hypothalamic-inhibiting hormones -triggering the hyposecretion of hypothalamic-inhibiting hormones -interfering with the normal transmission of nerve impulses to the posterior pituitary -interfering with the normal transmission of nerve impulses from the posterior pituitary

-interfering with the normal transmission of nerve impulses to the posterior pituitary Correct Specialized neurons called neurosecretory cells produce anti-diuretic hormone in the paraventricular nuclei of the hypothalamus. This hormone is then transported down the axons of these neurons and stored in their axon terminals, which are located in the posterior pituitary. The hormone is released from the axon endings when associated neurons fire action potentials, and it passes down the axon to the axonal terminals.

What is the significance of the slight swelling (called a goiter) in Krista's neck? -her thyroid gland is producing but not secreting excessive amounts of thyroid hormone -the auto-antibodies of Graves' disease have caused inflammation of her thyroid gland -it is a sign that her thyroid gland is overactive -it is unrelated to her condition

-it is a sign that her thyroid gland is overactive Correct Goiters almost always occur in hyperthyroidism and result from increased cell numbers and cell size in the thyroid gland as thyroid hormone production increases.

Which heart chamber receives blood from the pulmonary veins? -right ventricle -left atrium -right atrium -left ventricle

-left atrium Correct Yes, the left atrium receives oxygenated blood from the pulmonary veins.

Which chamber pumps oxygenated blood out the aorta to the systemic circuit? -left ventricle -left atrium -right ventricle -right atrium

-left ventricle Correct Yes, the left ventricle pumps oxygenated blood out the aorta to the entire body (systemic circuit).

If a person's parathyroids are responding properly to a drop in blood calcium, which of the following should result? -bone mass and density will increase -Vitamin D levels in the blood will drop -more calcium will be present in feces -less calcium will be excreted in the urine by the kidneys

-less calcium will be excreted in the urine by the kidneys Correct The kidneys would be encouraged by PTH to retain more calcium, which would result is less calcium in the urine.

Bilirubin is released into the bloodstream as old or damaged red blood cells are broken down by macrophages. Which organ is responsible for removing bilirubin from the blood stream? -pancreas -spleen -liver -kidneys

-liver Correct As RBCs are broken down, the heme of their hemoglobin is split off from globin. Its core of iron is salvaged and stored for reuse. The balance of the heme group is degraded to bilirubin, a yellow pigment that is released into the blood. Liver cells pick up the bilirubin and in turn secrete it (in bile) into the intestine, where it is metabolized to urobilinogen. Most of this degraded pigment leaves the body in feces, as a brown pigment called stercobilin.

What cells in the body respond to glucagon by breaking down glycogen and releasing glucose? -liver cells and cells in the pancreas -muscle cells -intestinal cells -liver cells -cells in the pancreas

-liver cells

Body cells that respond to insulin include -muscle cells only -liver cells only -liver cells, as well as most other cells of the body -liver cells and muscle cells only -intesitnal cells only

-liver cells, as well as most other cells of the body

A surge in __________ directly triggers ovulation. -progesterone -estrogen -follicle-stimulating hormone (FSH) -luteinizing hormone (LH)

-luteinizing hormone (LH) Correct When estrogen reaches a certain level in the body, it triggers a positive feedback loop that triggers an LH surge. This triggers ovulation.

What is the most important regulatory factor controlling the circulating levels of thyroid hormone? -a circadian rhythm of release -thyrotropin-releasing hormone (TRH) -negative feedback -thyroid-stimulating hormone (TSH)

-negative feedback Correct Yes, negative feedback controls the levels of circulating thyroid hormone. If levels are high, negative feedback will decrease thyroid-stimulating hormone (TSH) (and thyrotropin-releasing hormone (TRH) to some degree), thus decreasing the thyroid hormones.

Which of the following leukocyte is not correctly matched with its function? -neutrophils: produce antibodies -basophils: inflammation -lymphocytes: immune response against viral infections -monocytes: develop into macrophage

-neutrophils: produce antibodies Correct Neutrophils are active phagocytes that ingest bacteria and some fungi. Antibodies are produced by certain types of leukocytes.

Which of the following represents a difference between extrinsic and intrinsic blood clotting pathways? -One leads to the formation of prothrombin activator and the other does not. -One is faster than the other. -One involves calcium ions, while the other does not. -One can be triggered by tissue damage, while the other cannot.

-one is faster than the other. Correct The extrinsic pathway is faster than the intrinsic pathway, which has many intermediate steps.

The basic difference between spermatogenesis and oogenesis is that __________. -one mature ovum is produced in oogenesis, and four mature sperm are produced in spermatogenesis -spermatogenesis involves mitosis and meiosis, but oogenesis involves meiosis only -the mature ovum is haploid and the sperm is diploid -the mature ovum is diploid and the sperm is haploid -two polar bodies are produced in spermatogenesis

-one mature ovum is produced in oogenesis, and four mature sperm are produced in spermatogenesis right answer feedback:One mature ovum is produced in oogenesis, and four mature sperm are produced in spermatogenesis.

Which of the following hormone is regulated by a neuroendocrine ("letdown") reflex?] -oxytocin -cortisol -antidiuretic hormone (ADH)

-oxytocin Correct Yes, suckling of the infant (or stretching of the uterus) increases release of oxytocin, which causes the milk let-down effect (or increased uterine contractions)

What erythrocyte production disorder results from an autoimmune disease associated with insufficient vitamin B12 absorption (step 6)? -pernicious anemia -hemorrhagic anemia -renal anemia -aplastic anemia

-pernicious anemia Correct A glycoprotein called intrinsic factor, produced by the stomach, is required to absorb adequate vitamin B12. Pernicious anemia results when the stomach does not produce adequate intrinsic factor, often because the cells that produce it are attacked by the body's immune system.

Cyclic AMP is degraded by __________. AMPadenylate cyclase G proteins phosphodiesterase protein kinase

-phosphodiesterase Correct Phosphodiesterase degrades cyclic AMP into AMP

Which intracellular substance degrades cAMP, thus inactivating the response to a hormone? -protein kinase C -adenylate cyclase -phosphodiesterase -phospholipase C

-phosphodiesterase Correct Yes, phosphodiesterase is an intracellular enzyme that degrades cAMP.

What is the role of activated protein kinases? -Phosphorylate ADP to ATP. -Degrade cAMP to AMP. -Phosphorylate proteins. -Activate adenylate cyclase. -Convert ATP to cAMP by phosphorylation.

-phosphorylate proteins Correct Phosphorylation can activate different proteins causing the response of the cell to water-soluble hormone.

Water-soluble hormones affect target cells by binding to __________. -adenylate cyclase -plasma membrane receptors -cytoplasmic receptors -protein kinases -cAMP

-plasma membrane receptors Correct Water-soluble hormones bind to specific receptors in the plasma membrane, whereas steroid hormones bind to cytoplasmic receptors.

What enzyme removes unneeded clots after healing has occurred? -fibrin -thrombin -plasmin -plasminogen

-plasmin Correct As healing progresses, the clot retraction process reduces the size of the clot, while plasminogen is activated into plasmin that digests the fibrin in the clot.

Which of the following is FALSE regarding the role of platelets in hemostatic reactions? -platelets do not stick to intact endothelial cells -platelets adhere to exposed collagen fibers -platelets release chemicals that attract and bind other platelets -platelets release the only chemical factors that can initiate coagulation

-platelets release the only chemical factors that can initiate coagulation. Correct Platelet chemicals initiate the intrinsic pathway, but tissue factors released from damaged endothelium start the extrinsic pathway for coagulation in motion.

Which of the following triggers the release of glucagon? -releases of somatostatin -hyperglycemia -release of insulin -prolonged fasting

-prolonged fasting Correct Glucagon acts to stimulate release of glucose into the blood to counteract falling levels.

What triggers erythropoietin (EPO) production to make new red blood cells? -a high hematocrit -reduced availability -excess oxygen in the bloodstream -too many platelets

-reduced availability of oxygen Correct Reduced oxygen delivery to the kidneys will result in the release of erythropoietin (EPO), which in turn promotes the release of more erythrocytes from the bone marrow.

Which chamber receives blood from the superior and inferior vena cavae? -left atrium -right ventricle -right atrium -left ventricle

-right atrium Correct Yes, the right atrium receives unoxygenated blood from the systemic circuit.

Which heart chamber pumps unoxygenated blood out the pulmonary trunk? -left ventricle -left atrium -right ventricle -right atrium

-right ventricle Correct Yes, the right ventricle pumps unoxygenated blood out the pulmonary trunk to the lungs.

Pheochromocytoma produces symptoms of uncontrolled sympathetic nervous system activity. Which of the following is common to both adrenal chromaffin cells and adrenergic fibers of the sympathetic nervous system? ' -the presence of axons capable of secreting substances that are capable of increasing heart rate -secretion of epinephrine -secretion of norepinephrine -All of the listed responses are correct

-secretion of norepinephrine Correct Adrenal chromaffin cells are capable of releasing the hormones epinephrine and norepinephrine into the systemic circulation. Post-ganglionic neurons of the symphathetic nervous system also release norepinephrine.

__________ is caused by a genetic mutation that results in the production of abnormal hemoglobin. Under low-oxygen conditions, the abnormal beta chains link together and form stiff rods that alter the shape of the RBCs. -leukopenia -polycythemia -thalassemia -sickle-cell anemia

-sickle-cell anemia Correct In sickle-cell anemia, the havoc caused by the abnormal hemoglobin, hemoglobin S (HbS), results from a change in just one of the 146 amino acids in a beta chain of the globin molecule. This alteration causes the beta chains to link together under low-oxygen conditions, forming stiff rods so that HbS becomes spiky and sharp. This, in turn, causes the red blood cells to become crescent shaped when they unload oxygen molecules or when the oxygen content of the blood is lower than normal. The stiff, deformed RBCs rupture easily and tend to jam up in small blood vessels. These events interfere with oxygen delivery, leaving the victims gasping for air and in extreme pain. Bone and chest pain are particularly severe, and infection and stroke often follow.

Meconium is __________. -the hormone that causes a positive feedback mechanism to continue milk production -sloughed-off epithelial cells, bile, and other substances -the hormone that is termed "the natural birth control" and inhibits the release of GnRH -a fluid initially secreted by the mammary glands before true milk is produced

-sloughed-off epithelial cells, bile, and other substances right answer feedback:Meconium is a tarry green-black paste containing sloughed-off epithelial cells, bile, and other substances.

All of the following conditions would stimulate the pancreas to release insulin EXCEPT one. Which one? -a rise in blood glucose levels -a rise in blood amino acid levels -acetylcholine release on the pancreatic cells -sympathetic activation

-sympathetic activation Correct Sympathetic activation is designed to increase blood glucose levels, so it inhibits the release of insulin.

Your patient has been admitted to the intensive care unit with a severe head injury. As you monitor his urine output, you find that it has fallen sharply. Lab tests indicate that his serum osmolality is lower than normal (i.e., his plasma is too dilute). What should you suspect? -parathyroid hormone (PTH) -syndrome of inappropriate antidiuretic hormone (SIADH) -thyroid-stimulating hormone (TSH) -prolactin

-syndrome of inappropriate antidiuretic hormone (SIADH) Correct Antidiuretic hormone (ADH) is released from the posterior pituitary in response to rising serum osmolality. This causes the kidneys to preserve water, helping to restore normal solute/water balance. Your patient is retaining water despite a dilute plasma, suggesting that he is making "inappropriate" ADH. This problem may occur after head trauma or in some cancers.

A liver cell responds to insulin by -releasing insulin -breaking down glycogen and releasing glucose -taking in glucose and converting it to glycogen -releasing glucagon

-taking in glucose and converting it to glycogen

.During the secretory phase of the uterine cycle, __________. -ovulation occurs -the functional layer of the endometrium regenerates -the endometrium is shed -the endometrium prepares for implantation

-the endometrium prepares for implantation Correct Secretions in the endometrium during the secretory phase aide in implantation.

When blood glucose levels are high -the pancreas releases insulin -the pancreas releases glucagon -the pancreas releases glucose -the liver releases insulin -the liver releases glucagon

-the pancreas releases insulin Correct The pancreas responds to high blood glucose levels by releasing insulin.

Which of the following is NOT a benefit of breathing through the nose? -filtration of the air entering the nasal cavity -recovering heat and moisture from the air leaving the nasal cavity -the production of smooth, laminar airflow as air passes by the nasal conchae -heating and moistening the air entering the nasal cavity

-the production of smooth, laminar airflow as air passes by the nasal conchae Correct In fact, airflow becomes turbulent (not smooth or laminar) as the gases of inhaled air swirl through the twists and turns created by the curved nasal conchae protruding medially from each lateral wall of the nasal cavity. Heavier, nongaseous particles deflect onto the mucus-coated surfaces, where they become trapped. As a result, few particles larger than 6 μm make it past the nasal cavity.

The decreased intracellular concentration of sodium in tubular cells during active transport is caused by which of the following mechanisms? -passive sodium channels -the sodium-potassium ATPase pump in the basolateral membrane -sodium-glucose cotransporter -the sodium-potassium ATPase pump in the luminal membrane

-the sodium-potassium ATPase pump in the basolateral membrane Correct Yes, this pump moves sodium out of the cell into the interstitium, thus decreasing intracellular sodium

What protein involved in coagulation provides the activation for the final step in clotting? -thrombin -fibrinogen -prothrombin activator -fibrin

-thrombin Correct Thrombin catalyzes the conversion of fibrinogen into fibrin. This is the final step in coagulation.

Which hormone's receptor is always bound to DNA, even when the receptor is empty? -thyroid hormone -cortisol -insulin

-thyroid hormone Correct Yes, thyroid hormones are lipid soluble and their receptors are bound to the response elements of the DNA.

Which of the following is NOT secreted by the thyroid? -triiodothyronine -calcitonin -thyroxine -thyroid-stimulating hormone

-thyroid-stimulating hormone Correct Thyroid-stimulating hormone (TSH) is secreted by the anterior pituitary gland.

The heart is actually (one, two, or three) pumps? -one pump -two pumps -three pumps

-two pumps Correct Yes, the right side of the heart pumps to/from the lungs (pulmonary circuit) and the left side of the heart pumps to/from the rest of the body (the systemic circuit).

Growth factor hormones, such as insulin, bind to which type of receptor? -tyrosine kinase receptors -intracellular receptors -G proteins

-tyrosine kinase receptors Correct Yes, insulin binds to tyrosine kinase receptors and works without a second messenger.

During pleurisy, the inflamed parietal pleura of one lung rubs against the inflamed ______. thoracic wall parietal pleura of the other lung visceral pleura of the same lung -visceral pleura of the other lung

-visceral pleura of the same lung Correct Normally the visceral and parietal pleura of one lung glide easily over one another during breathing because they are smooth and lubricated by pleural fluid. During pleurisy, they become rough and friction develops between the two layers..

What type of hormones bind to receptors located on the cell membrane? -lipid-soluble hormones, such as thyroid hormones and cortisol -water-soluble hormones, such as insulin and epinephrine

-water-soluble hormones, such as insulin and epinephrine Correct Yes, peptides and catecholamines are water-soluble hormones that cannot diffuse through the plasma membrane.

Focus your attention on the section called Net filtration pressure occurs at the arteriolar end of a capillary on page 2 of Focus Figure 19.1. Note that for an NFP at any point there are four pressures to be considered. Pay attention to which forces are larger and which are smaller. Note that the white arrow represents the net filtration pressure, the result of four different forces.Drag and drop the terms to the two appropriate blanks describing pressures for each sentence.

.1 The "pushing" pressure of this fluid is usually very weak: HPif ; 0 mm 2. The "pulling" pressure of this fluid is usually due to large albumens and globulins: OPc ; ~26 mm 3. The "pulling" pressure of this fluid resulting from nondiffusible solutes is very low: OPif; ~1 mm 4. The "pushing" pressure of this fluid is high and results from blood pressure: BPc ; ~35 mm

Use the following information to solve the problem using a Punnett square. Assume that the dominant "B" allele encodes brown eyes and the recessive "b" allele encodes blue eyes. An individual with the genotype BB has children by mating with an individual of the genotype bb. What is the percent chance that they would have children with blue eyes? 0% 25% 50% 100%

0% All the offspring would have brown eyes.

Why does this patient have signs and symptoms so different from Mr.G's?

1. Mr. V's partly blocked tricuspid valve restricts blood flow from the RIGHT ATRIUM to the RIGHT VENTRICLE, turbulent blood flow is heard during DIASTOLE as blood is forced through the narrow opening. 2. The RIGHT ATRIUM does not empty normally, so it cannot accept a normal amount of blood from the BODY; blood accumulates in the veins and tissues, causing distended JUGULARS and SWOLLEN tissues. The excess fluid in tissues leads to weight GAIN and a HEAVY feeling in Mr. V's legs.

Match the term in the left column to the blanks near their definition on the right.

1. Stroke volume (SV): the volume of blood per heart beat pumped out by one ventricle 2. Venous return (VR): the volume of blood per minute flowing into one atrium 3. Heart rate (HR): the number of heart beats per minute 4. Cardiac output (CO): the volume of blood per minute pumped out by one ventricle 5. End diastolic volume (EDV): the volume of blood in one ventricle before contraction 6. End systolic volume (ESV): the volume of blood in one ventricle after contraction

Most water-soluble hormones exert their effects through the second messenger cyclic AMP (cAMP). This activity will test your understanding of the events that occur during cAMP signaling. Drag the events of cAMP signaling in the correct sequence from left to right.

1. Water-soluble hormone binds receptors 2. Receptor activates G protein 3. G protein activates adenylate cyclase 4. Adenylate cyclase generates cAMP 5. cAMP activates protein kinases Correct Activated protein kinases can then phosphorylate a variety of intracellular proteins to elicit the cell's response to the hormone.

There is a note in Mr. G''s chart that he had a color flow Doppler echocardiogram done to prepare for this visit. You pull up the results on the computer. You can see that the blood is flowing up through his mitral valve during systole and that his left ventricle wall is thickened, and his left atrium is dilated. The right ventricle looks normal. His ejection fraction is 43% normal levels are 55-75% (Heart Rhythm Society, 2013). How does Mr. G's altered blood flow explain his signs and symptoms? Let's use one of the diagrams you have already labeled to explain it.

1. When LEFT VENTRICLE contracts, some blood goes up through the mitral valve into the LEFT ATRIUM . Turbulent flow through valve causes SYSTOLIC murmur, LEFT ATRIUM is over-filled and DILATED. 2. Because some blood re-entered the LEFT ATRIUM less is pumped out into AORTA - EJECTION fraction is only is only 43&. 3. Less blood enters the AORTA so SBP is LOW. Baroreceptor reflex causes INCREASED heart rate; wall of overworked left ventricle becomes THICKER 4. Because it is already partly full from the ventricle, LEFT ATRIUM accepts less blood from the LUNGS. The LUNGS over fill with fluid; resulting in DIFFICULTY breathing.

In a typical or "average" cycle, ovulation occurs on day __________. 1 5 14 21 28

14 right answer feedback:In a typical cycle, ovulation occurs on day 14.

The number of chromosomes in a human gamete is __________; this is referred to as a __________ chromosome number. -23;haploid -23; diploid -46; haploid -46; haploid

23; haploid

When two people heterozygous for a trait produce a child, what is the probability that their child will be homozygous recessive for that trait?

25%

Drag and drop the items below into the correct sequence from left to right.

1st Step: Hypothalamus secretes corticotropin-releasing hormone 2nd Step: Corticotropin-releasing hormone travels through the portal system to the anterior pituitary 3rd Step: Adrenocorticotropic hormone is secreted from the anterior pituitary 4th Step: ACTH travels in the blood to the adrenal cortex 5th Step: Glucocorticoids and mineralocorticoids are secreted 6th Step: Target cells produce metabolic and renal effects Correct In the long-term stress response, stressful stimuli cause the hypothalamus to release corticotropin-releasing hormone, or CRH. CRH then travels through the hypothalamic portal system to the anterior lobe of the pituitary gland. The anterior pituitary releases adrenocorticotropic hormone, or ACTH, which travels via the blood to the adrenal gland. The adrenal cortex is the specific target for ACTH, which produces more glucocorticoids, chiefly cortisol, and mineralocorticoids, namely aldosterone. As you have seen, glucocorticoids have pronounced metabolic effects on the liver, while the mineralocorticoids target the kidney to retain more water and sodium, resulting in increased blood pressure and blood volume.

Correctly order the steps involved cellular immunity: 1.The Tc recognizes the infected host cell 2.The Tc interacts with epitope presented by MHC-I on the dendritic cell 3.The Tc secretes perforin and granzyme, causing apoptosis 4.The helper T cell activates the Tc cell

2 4 1 3

Place the following steps of phagocytosis in the order that they occur: 1.Endosome fuses with lysozome 2.Dendritic cell engulfs Rhinovirus 3.Epitopes are attached to MHC-II 4.Digestion of the Rhinovirus 5.MHC-II plus the attached epitope move to the outside of the dendritic cell

2. 1. 4. 3. 5.

Every day the kidneys filter nearly __________ of fluid from the bloodstream. 50 liters 100 liters 200 liters 500 liters

200 liters

Arrange the following structures to represent the sequence in which urine passes through them to the external environment: (1) ureter, (2) renal pelvis, (3) calyx, (4) urinary bladder, and (5) urethra. 3, 2, 1, 4, 5 1, 2, 3, 4, 5 3, 4, 1, 5, 2 2, 4, 1, 3, 5

3, 2, 1, 4, 5

Which of the following would reflect the typical net hydrostatic pressure (HP) at the arterial end of the capillary?

34 mm Hg Correct Yes, HPc (35 mm Hg) - HPI (1 mm Hg) = 34 mm Hg, which is the net hydrostatic pressure at the arterial end. The hydrostatic pressure of the blood is much higher at the arterial end of the capillary, thus favoring filtration.

The diploid chromosomes number in humans is -12 -23 -24 -46

46 The normal chromosome number in most body cells is referred to as the diploid chromosome number. In humans, this number is 46. The number of chromosomes in gametes is called the haploid number. The haploid number is half the diploid number. Thus, in humans, the haploid chromosome number is 23.

Using the spirographic data shown, what is the minute ventilation of an average adult male at rest breathing at a rate of 10 breaths per minute? 5000 milliliters per minute 36000 milliliters per minute 48000 milliliters per minute 60000 milliliters per minute

5000 milliliters per minute Correct The minute ventilation is calculated by multiplying the breaths per minute by the tidal volume, so in this case, 10 breaths/minute × 500 ml/breath = 5000 ml/minute.

Any arterial pH between __________ is considered physiological acidosis. 6.5 and 7.0 6.5 and 7.35 7.0 and 7.35 7.35 and 7.45

7.0 and 7.35

Calculate the stroke volume if the end diastolic volume (EDV) is 135 mL/beat and the end systolic volume (ESV) is 60 mL/beat.

75 mL/beat Correct The SV is calculated by subtracting the ESV from the EDV. You are comparing the volume of the ventricle at its fullest to its emptiest.

Calculate the cardiac output if heart rate (HR) is 90 beats per minute, stroke volume (SV) is 110 ml/beatml/beat, end diastolic volume (EDV) is 140 mlml, and end systolic volume (ESV) is 30 mlml.

9.9 L/min Correct Cardiac output is the product of heart rate times stroke volume. Cardiac output increases to meet increased metabolic demand, often by increasing both heart rate and stroke volume at the same time. This occurs, for example, during maximal exercise, when cardiac output may increase to four or five times the resting level.

Refer again to the entire focus figure. Using the same client, Mr. Orange, with the same pressures, what is the client's NFP at the venous end, NFPv?

= (22 + 1) - (26 + 0) = -3 mm Correct Blood pressure, HPc, decreases by about 50% from arteriolar to venous end of the capillary bed.

Calculate the net filtration pressure if capillary hydrostatic pressure is 60 mm Hg, capillary osmotic pressure is 25 mm Hg, and capsular hydrostatic pressure is 10 mm Hg. =50 mm Hg =25 mm Hg =60 mm Hg =35 mm Hg

=25 mm Hg Correct Yes, 60 - (25 + 10) = 25 mm Hg. The two pressures that oppose filtration must be subtracted from the force favoring filtration.

During the preschool examination of a 4-year-old boy, you are unable to palpate his testes within the scrotal sac. His parents state that previous exams have been normal, and that the testes are clearly visible during baths. What is the most likely explanation? -A chilly examination room or your cold hands have activated the cremasteric reflex. -The testes may have migrated back up into the boy's abdomen since the last examination. -Because he does not trust you, the boy is deliberately causing his testes to retract close to his body. -The parents' observations are probably incorrect.

A chilly examination room or your cold hands have activated the cremasteric reflex. Correct The cremaster muscles elevate the testes, pulling them higher in the scrotum in response to cold temperatures.

Which of the following describes a hapten?

A hapten is a foreign molecule that becomes immunogenic when combined with body proteins. Correct Haptens trigger adaptive defenses when combined with body proteins, making them appear foreign. Examples of haptens can be found in poison ivy, animal dander, and certain drugs.

How does heart rate affect stroke volume?

A high heart rate reduces the end diastolic volume (EDV) and stroke volume (SV), because there is less time for ventricular filling. Correct A slow heart rate allows more time for ventricular filling. Conversely, a faster heart rate allows less time. The amount the heart fills during diastole (EDV) has a direct impact on stroke volume.

What type of intravenous infusion would you give to a runner who has collapsed after drinking too much water during the course of her marathon and why? A hypertonic saline solution to pull water out of her cells A hypertonic saline solution to cause more water to enter her cells An isotonic saline solution to pull water out of her cells A hypotonic saline solution to pull water out of her cells

A hypertonic saline solution to pull water out of her cells

homologous chromosomes are:

A pair of chromosomes with one from the mother and one from the father

In the male sexual response, what is resolution?

A period of muscular and psychological relaxation

Examine the left side of page 1 of Focus Figure 19.1.Assume you have a U-tube with a semipermeable capillary membrane made of simple squamous epithelium separating the solutions on either side. Blood will be on the left side of the tube, and watery extracellular fluid (ECF) will be on the right side of the tube. Predict what will occur to the levels of fluids by hydrostatic pressure, the pressure of watery fluid pushing on a boundary.

A pump pushing down on the left arm of the U-tube containing blood will increase ECF fluid levels on the right side. Correct Remember that the pushing by water, or hydrostatic pressure, against a boundary can force watery fluid through a semipermeable membrane, from higher to lower pressure.

Urea transport out of the medullary collecting duct is enhanced by __________. ADH angiotensin II prostaglandin E2 renin

ADH

In severe dehydration or blood loss, would ADH levels be high or low, and would urine production be high or low? ADH - high; urine production - high ADH - low; urine production - high ADH - high; urine production - low ADH - low; urine production - low

ADH - high; urine production - low Correct ADH (antidiuretic hormone) causes aquaporins to be inserted into the cell membranes of the collecting duct cells, facilitating the reabsorption of water from the filtrate, back into the blood. By doing this, ADH helps a dehydrated individual to minimize water loss through urine, and maximize water reabsorption to help rehydrate cells.

How does ADH contribute to the formation of concentrated urine?

ADH increases the permeability of the collecting duct to water by stimulating the insertion of aquaporins into the luminal membrane.

Which of these structures conduct(s) action potentials the slowest?

AV node Correct Action potentials slow down as they pass through the AV node. This gives the atria time to finish contracting before the ventricles are depolarized.

Examine the reaction below, and then answer the question. A+B⟶Enzyme1C+D⟶Enzyme2->E+F⟶Enzyme3->G+H Absence of which enzyme would cause the greatest reduction in the production of G and H? Enzyme 1 Enzyme 2 Enzyme 3 Absence of any of the enzymes would have the same effect—decrease or block the production of G and H.

Absence of any of the enzymes would have the same effect—decrease or block the production of G and H. Correct Excellent! These reactions are like a domino effect. Each reaction leads to the next, so losing any enzyme slows that reaction, which in turn slows the rest of the pathway.

Which of the following is NOT a step that ultimately leads to antibody production?

Activation of cytotoxic T cells by helper T cells Correct The activation of cytotoxic T cells leads down the path of cell-mediated immunity.

Which of the following statements is true?

Adaptive defenses include both humoral and cellular immunity. Correct This answer is correct. Adaptive defenses are operating optimally when both humoral and cellular immunity are working together in response to a pathogen.

Through the tubuloglomerular feedback mechanism, how would an increase in filtrate NaCl concentration affect afferent arteriole diameter? Afferent arteriole diameter would stay about the same. Afferent arteriole diameter would decrease. Afferent arteriole diameter would increase.

Afferent arteriole diameter would decrease. Correct High NaCl concentration in the filtrate at the JGA indicates that GFR is too high. By decreasing the diameter of the arteriole delivering blood to the glomerulus, HPg is decreased, resulting in lower GFR.

Why do high levels of potassium stimulate aldosterone secretion?

Aldosterone stimulates the sodium-potassium pump to reabsorb sodium while simultaneously secreting potassium.

The human GI tract is home to a complex ecosystem of microbial cells, which comprise the gut microbiota. Which of the following is true of the microbiota? Generally, the highest concentration of GI tract microbes is found in the colon. Metabolism by the gut microbiota results in small amounts of vitamins and short-chain fatty acids which can be absorbed and utilized by cells of the body. Normally, beneficial bacteria vastly outnumber harmful bacteria in the microbiota. All of the above.

All of the Above

Emphysema can result in an ______. -increased level of carbaminohemoglobin -increased lev el of deoxyhemoglobin -increased likelihood of the skin of Caucasians developing a slightly blue coloration -All of the listed responses are correct

All of the listed responses are correct. Correct Using your textbook, review the structure of hemoglobin and how oxygen and carbon dioxide bind. Additionally review the pathophysiology of emphysema.

What is meant by the clonal expansion of a B cell?

An activated B cell divides into cells that give rise to memory B cells and plasma cells. Correct This answer is correct. The activated B cell divides until there are many clones. Some differentiate into memory cells, other become plasma cells that produce and secrete antibodies.

Your patient has had a total gastrectomy (removal of the stomach). Which of these potential problems are you MOST concerned about? Excessive bleeding as a result of absent clotting factors in the blood. Inability to digest starch because of the absence of amylase Inability to tolerate fats because of absent bile salts Anemia

Anemia Correct The parietal cells of the stomach produce intrinsic factor, which permits the absorption of vitamin B12, a nutrient required for the maturation of red blood cells. Unless your patient receives vitamin B12 replacement therapy, he or she will develop pernicious anemia.

Which best describe the isovolumetric contraction phase of the cardiac cycle?

As ventricular systole starts, the AV valves are closed and the semilunar valves are closed. Because the ventricles are contracting and both valves are closed, pressure increases rapidly leading to ejection. Correct This closed pressure system is important for the ventricles to be able to generate enough pressure to open the semilunar valves and eject blood from the heart.

Which of the following is NOT a vasoconstrictor?

Atrial Natriuretic Peptide Correct Atrial natriuretic peptide (ANP) decreases peripheral resistance by causing vasodilation

Which of the following hormones will lower blood pressure?

Atrial Natriuretic Peptide (ANP) Correct Atrial natriuretic peptide (ANP) is a hormone released by the atria of the heart. It stimulates the kidney to excrete more sodium and water in the urine, which decreases blood volume, leading to a reduction in blood pressure. It also causes generalized vasodilation.

Which pair of molecules do NOT directly interact with one another?

BCR and TCR Correct Both of these molecules interact with epitopes. BCRs interact with epitopes on the whole pathogen. TRCs interact with processed epitopes when presented on MHC-II.

So why is Mr. G's mitral regurgitation causing a systolic murmur? Remember, a murmur is caused by abnormal turbulent blood flow. -Because his mitral valve leaks, when the ventricles contract, they push blood back up through it into the right atrium -Because his mitral valve leaks, when the ventricles contract, they push blood back up through it into the aorta -Because his mitral valve leaks, when the ventricles contract, they push blood back up through it into the left atrium -Because his mitral valve leaks, when the ventricles relax, they push blood back up through it into the left

Because his mitral valve leaks, when the ventricles contract, they push blood back up through it into the left atrium Correct Good work. Knowing the cardiac cycle helps explain why Mr. G's leaky mitral valve causes a systolic murmur. Let's look at the other problems it might be causing him

What causes the abnormal swishing or whooshing sound that is heard as blood regurgitates back into an atrium from its associated ventricle? -blood turbulence -aortic recoil -semilunar valve closure -pulmonary trunk expansion

Blood Turbulence

You have a patient with a very low blood pressure of 80/60 mm Hg. What are the possible consequences of this low blood pressure on capillary function?

Both A and C are corerct. (Mean arterial pressure MAP would not be high enough to adequate capillary exchange to occur & Tissues would become ischemic because of lack of adequate perfusion) Correct We need a high enough mean arterial pressure (MAP) for fluid to be pushed out of the capillaries (i.e., overcoming osmotic pressure drawing fluid back in and to ensure that oxygen and nutrients are pushed out to the tissues). Without an adequate MAP, perfusion would not be adequate to deliver oxygen and nutrients to the tissues.

How is the chemiosmotic process shown in the figure similar to secondary active transport (cotransport) of solutes (discussed in Chapter 3)? Both processes result in the synthesis of ATP. Both processes are powered by an electron transport chain, which transports ions across a membrane. Both processes typically involve H+ (proton) gradients. Both processes depend on the stored energy in an electrochemical gradient.

Both processes depend on the stored energy in an electrochemical gradient. Correct During electron transport, energy is stored in the form of a proton gradient. H+ ions' diffusion back across the membrane releases this energy and powers ATP synthesis. Secondary active transport uses the energy released by diffusion of ions such as sodium to power the cotransport of other solute molecules, such as glucose.

What is the correct general equation for cellular respiration? 6 CO2 + 6 H2O + ATP energy → C6H12O6 + 6 O2 C6H12O6 + 6 H2O → 6 CO2 + 6 O2 + ATP energy C6H12O6 + 6 O2 → 6 CO2 + 6 H2O + ATP energy C6H12O6 + 6 CO2 → 6 O2 + 6 H2O + ATP energy 6 O2 + 6 H2O + ATP energy → C6H12O6 + 6 CO2

C6H12O6 + 6 O2 → 6 CO2 + 6 H2O + ATP energy Correct Cellular respiration extracts energy from glucose (C6H12O6) to produce smaller energy packets (ATP).

Which of the following intestinal hormones stimulates the release of bile from the gall bladder? motilin gastrin GIP (gastric inhibitory peptide) CCK (cholecystokinin)

CCK (cholecystokinin) Correct Yes, CCK causes the gall bladder to contract, releasing bile and an enzyme-rich pancreatic juice to be secreted from the pancreas.

Despite the fact that the partial pressure difference is so much smaller for CO2, why is there as much CO2 exchanged between the alveoli and blood as there is O2, ? -The capillary walls are more permeable to CO2 than O2. -CO2 is much more soluble in blood than O2. -CO2 is a smaller molecule and diffuses faster than O2O2.

CO2 is much more soluble in blood than O2 Correct Yes, because CO2CO2 is very soluble in blood, it does not require as large a pressure gradient as O2O2.

_______ has a greater partial pressure in the pulmonary capillaries than in the alveoli, so it diffuses into the -O2; pulmonary cavities -CO2; pulmonary capillaries -CO2; alceoli -O2; alveoli

CO2; alveoli Correct Yes, CO2 diffuses along its partial pressure gradient from the pulmonary capillaries into the alveoli until equilibrium is reached.

What is, overall, the LEAST abundant ion, including all three compartments? K+ Na+ Cl− Ca2+

Ca 2+ Correct Ca2+ is the least abundant ion when summing all three compartments. It is particularly scarce in the intracellular compartment, although some cells, such as muscle, do sequester la

How does carbonic acid alone directly affect the pH of blood? Carbonic acid does not significantly affect blood pH. Carbonic acid acts as a buffer by binding to a strong acid, producing a weak acid and salt, thus lowering blood pH. Carbonic acid is a strong acid that, if added to blood, causes the pH to rise. Carbonic acid is a strong acid that dramatically lowers blood pH unless immediately neutralized by a buffer.

Carbonic acid does not significantly affect blood pH Correct The difference between strong acids and weak ones is their ability to dissociate in water. Weak acids such as carbonic acid do not completely dissociate, which limits their effect on pH, whereas strong acids such as HClHCl completely dissociate.

In contrast to a skeletal muscle cell action potential, why does the action potential for a cardiac muscle cell contain a "plateau" phase?

Cardiac muscle cells contain slow Ca2+Ca2+ channels in their sarcolemma that continue to allow influx of Ca2+Ca2+ ions after Na+Na+ channels are inactivated. Correct Fast voltage-gated Na+Na+ channels are open only for a very brief time before they are inactivated, but the resulting depolarization opens slow Ca2+Ca2+ channels that start a positive feedback cycle that temporarily keeps the membrane potential high.

Why do you have to carefully monitor certain patients who are on "loop diuretics" to lower their blood pressure? They may excrete too much protein instead. Certain diuretics are not specific and in addition to getting rid of excess water, additional ions like potassium will be secreted. You do not want them to build up too much water in the renal pyramid. They are at an increased risk for a urinary tract infection.

Certain diuretics are not specific and in addition to getting rid of excess water, additional ions like potassium will be secreted. Correct If the diuretic is not potassium sparing, you must be careful to instruct patients that they may need to adjust their diet or take a potassium supplement while on this medicine.

How do physiological buffer systems differ from chemical buffer systems? -Physiological buffer systems operate faster than chemical buffers, but they are less powerful. -Chemical buffer systems convert strong acids or bases into weak acids or bases, while physiological buffer systems remove acids and bases from the body. -Physiological buffer systems use different compounds than those used by chemical buffer systems. -Physiological buffer systems regulate the pH within specific organs, while chemical buffer systems regulate body fluid pH.

Chemical buffer systems convert strong acids or bases into weak acids or bases, while physiological buffer systems remove acids and bases from the body. Correct Physiological buffer systems work by regulating the quantity of acids and bases in the body by excreting compounds such as carbon dioxide or hydrogen ions.

Which of the following best describes the chloride shift as seen in the figure? -Chloride is taken out of the blood to counterbalance the inflow of carbon dioxide. -Chloride rushes into RBCs to counterbalance the outflow of bicarbonate. -Chloride binds to bicarbonate, which allows more of it to be carried in the bloodstream. -Chloride is removed from hemoglobin when carbon dioxide binds to it.

Chloride rushes into RBCs to counterbalance the outflow of bicarbonate. Correct The outflow of negative bicarbonate ions from RBCs is balanced by the inflow of chloride ions.

What negatively charged ion is reabsorbed by the renal tubules when the blood pH is drawing near its alkaline limit? -PO4 -3 -Cl- -HCO3 - -NO2 -

Cl-

A person who has AIDS contracts rare and often life-threatening infections because their helper T cell count is so low. Which of the following components of the immune response still respond to antigen despite the low helper T cell count?

Clonal selection of B cells Correct B cells can still bind to antigen, which is the process of clonal selection. However, without a helper T cell, clonal expansion and antibody production will not occur.

Which of the following can be heard with a stethoscope most easily?

Closing of atrioventricular valves Correct Heart sounds are an extremely helpful diagnostic tool that is not invasive. The first heart sound, often described as a lubb, corresponds to the closing of the atrioventricular valves. The second heart sound is a sharper and crisper dupp that corresponds to the closing of the semilunar valves. Consequently, the two heart sounds mark the beginning and the end of ventricular systole.

Focus your attention on the section of the capillary bed in the area called The big picture in Focus Figure 19.1. Drag and drop the correct identification label to the figure with a leader line.

Correct Recall that blood flows from a larger diameter artery forward to a microscopic arteriole that can carry blood into a capillary bed. The vessel exiting the capillary bed, the microscopic venule, carries blood to a vein for its journey back to the heart. Along this pathway, the blood hydrostatic pressure changes. There is a significant drop in blood pressure, ~50%, between the arteriolar end of the capillary bed and the venule end.

These activities change heart rate by changing the balance of parasympathetic and sympathetic stimulation of the heart. Sort each of the activities according to whether it decreases or increases heart rate. Drag the appropriate items to their respective bins.

Correct The balance between parasympathetic and sympathetic stimulation to the heart controls heart rate. Increased parasympathetic activity decreases heart rate, and sympathetic activity increases heart rate.

Drag and drop the labels onto the figure to identify the events associated with the cardiac cycle. Use each label once.

Correct The cardiac cycle reflects pressure changes in several key structures of the heart; the volume changes as blood moves through the heart and, finally, the electrical events associated with heart contraction. Review Focus Figure 18.2 for each of the line graphs and their relation to the cardiac cycle.

Drag the hormone on the left to its correct function on the right

Corticotropin-releasing hormone: Stimulates the anterior pituitary to release ACTH Epinephrine: Increased dilation of bronchioles and heart rate Adrenocorticotropic hormone: Targets the adrenal gland to release glucocorticoids and mineralocorticoids Cortisol: Increased metabolic effects from the liver Aldosterone: Increased sodium and water absorption Correct Each of these hormones plays a distinctive role in the body. Epinephrine, along with norepinephrine, target the cardiovascular and respiratory systems, producing increases in heart rate and blood pressure, along with increased dilation of the respiratory bronchioles. Blood glucose and metabolic rate increases are also important functions of epinephrine. Corticotropin-releasing hormone and adrenocorticotropic hormone are tropins; that is, their target organs are other endocrine glands. Corticotropin-releasing hormone targets the anterior pituitary, causing the release of adrenocorticotropic hormone; this, in turn, affects the adrenal cortex to release glucocorticoids and mineralocorticoids. Lastly, cortisol and aldosterone are the chief secretions of the glucocorticoids and mineralocorticoids, respectively; cortisol targets the liver for various metabolic effects, while aldosterone impacts the kidneys, causing them to retain more sodium and water.

Cancer cells would be attacked by which of the following cells?

Cytotoxic T cells Correct Yes, cytotoxic T cells attack cancer cells, since they are part of cell-mediated immunity.

How do cytotoxic cells directly attack target cells?

Cytotoxic cells bind to the target cell and secrete chemicals that induce apoptosis. Correct Cytotoxic cells are able to dock with antigens on the target cell membrane. Once docked, the cytotoxic cell releases perforins and granzymes, which weaken the cell membrane and induce apoptosis.

Which of the following structures would be the LEAST vulnerable to damage caused by oxygen toxicity? brain spleen muscles costal cartilages

costal cartilages Cartilage is normally avascular and receives oxygen by diffusion from surrounding capillaries

Which of these combinations of values would help you determine if your patient was suffering from metabolic acidosis? -Increased blood HCO3- levels and decreased pH -Increased PC02 levels in the blood and decreased pH -Decreased blood HCO3- levels and decreased pH -Decreased PC02 levels in the blood and increased pH levels

Decreased blood HCO3- levels and decreased pH Correct If the problem is not respiratory, then often carbon dioxide levels are normal. If the kidneys are excreting too much HCO3-, then your pH will decrease.

Captain Fallowell's drop in blood pressure when standing up is most likely due to:

Decreased cardiac output due to low stroke volume. Correct His heart rate is elevated and his blood vessels are vasoconstricting, yet his blood pressure is low; stroke volume must be the factor that has decreased.

Which statement is correct? -The greater the available surface area the lower the amount of gas exchange during internal respiration. -During internal respiration, carbon dioxide is unloaded from the blood. -During external respiration, oxygen is unloaded from the blood. -During external respiration, equilibrium is reached for O2O2 when the partial pressure for O2O2 in the pulmonary capillaries and the alveoli are the same.

During external respiration, equilibrium is reached for O2O2 when the partial pressure for O2O2 in the pulmonary capillaries and the alveoli are the same. Correct Yes, equilibrium is reached for O2O2 when the partial pressure of O2O2 in the pulmonary capillaries and the tissue cells are the same.

__________ occurs when a woman has a deformed or male-like pelvis, resulting in prolonged and difficult labor; this condition can lead to fetal brain damage, ultimately causing cerebral palsy or epilepsy. Dyspnea Parturition Preeclampsia Dystocia

Dystocia If a woman has a deformed or male-like pelvis, labor may be prolonged and difficult. This condition is called dystocia. Besides extreme maternal fatigue, another possible consequence of dystocia is fetal brain damage, resulting in cerebral palsy or epilepsy.

Which type of vessel is most suited to expanding and recoiling in response to the ejection fo blood from the heart?

Elastic Artery Correct Elastic arteries absorb most of the shock when the heart ejects blood. The expansion stores energy; the recoil disperses this energy by moving blood further through the circulation.

How does the electron transport chain participate in generating ATP? -Electrons in the electron transport chain move through ATP synthase, providing the energy needed to generate ATP. -Electrons in the electron transport chain allow respiratory enzymes to pump hydrogen ions into the intermembrane space, creating an electrochemical gradient. -Electrons in the electron transport chain are passed to oxygen molecules that then combine with carbon ions to form carbon dioxide. -Electrons in the electron transport chain are added to ADP molecules to convert them back into high-energy ATP molecules.

Electrons in the electron transport chain allow respiratory enzymes to pump hydrogen ions into the intermembrane space, creating an electrochemical gradient. Correct The concentrated hydrogen ions in the intermembrane space diffuse through ATP synthase, providing it the energy needed to synthesize ATP.

One way the kidneys maintain HCO3 - balance by __________. -reabsorbing H+ -reabsorbing CO2 -generating Na+ ions -generating new HCO3-

generating new HCO3-

What is the role of estrogen on luteinizing hormone (LH) production? -Estrogen levels have no effect on LH production. -Estrogen may have either an inhibitory or an excitatory effect on LH production depending on the overall estrogen levels. -Estrogen only has an excitatory effect on LH release so that as estrogen levels increase, there will always be a corresponding increase in LH levels. -Estrogen only inhibits LH production so that as estrogen levels increase, there will always be a corresponding decrease in LH levels. Submit

Estrogen may have either an inhibitory or an excitatory effect on LH production depending on the overall estrogen levels. Correct Estrogen exerts negative feedback on LH production until estrogen levels reach a certain concentration, and then estrogen exerts positive feedback on LH production, which promotes ovulation.

Which of the following would NOT be a cause of edema in your patient's lower extremities?

Excess protein in the diet Correct If a patient's plasma protein, especially albumin, is too low, then his or her osmotic pressure is low. Recall that plasma proteins act as "water magnets," resisting the force of hydrostatic pressure (i.e., blood pressure) that pushes fluid out of the capillaries. Edema can be a consequence of low plasma protein.

Which of the following statements about fluid movement is NOT correct? -Exchange between the plasma and the intracellular fluid occurs across the cell membrane. -Under normal circumstances, lymph vessels help maintain fluid balance, especially between the plasma and the interstitial fluid. -Exchange between interstitial fluid and intracellular fluid occurs across the plasma membrane. -Exchanges between plasma and interstitial fluid happen between capillary walls.

Exchange between the plasma and the intracellular fluid occurs across the cell membrane Correct Exchange between the plasma and the intracellular fluid would have to involve exchange through the interstitial fluid.

Most fluid in the body is in the extracellular fluid (ECF) compartment. True False

False

Most of the hydrogen ions in the body come from acidic substances in the foods we ingest. -True -False

False

Parathyroid hormone (PTH) increases Ca2+ deposition in the bones. -True -False

False

The most abundant cation in the extracellular fluid is potassium. -True -False

False

The perirenal fat capsule is a transparent capsule that prevents infections in surrounding regions from spreading to the kidney. True False

False

Coitus interruptus is among the most effective birth control methods. True False

False Coitus interruptus, or withdrawal of the penis just before ejaculation, is unreliable because control of ejaculation is never ensured. Additionally, sperm may be present in the pre-ejaculatory fluid secreted by the bulbourethral glands.

After DNA replication, each individual chromosome becomes a homologous pair. -True -False

False Correct After DNA replication, each chromosome is composed of two sister chromatids.

The ectoderm is the primary germ layer from which the digestive, respiratory, and urogenital systems and the associated glands form. True False

False Ectoderm fashions the structures of the nervous system and the skin epidermis.

The first major event in organogenesis is gastrulation. True False

False Gastrulation lays down the basic structural framework of the embryo and sets the stage for the rearrangements that occur during organogenesis. The first major event in organogenesis is neurulation.

The ovaries are situated retroperitoneally next to the kidneys. True False

False The ovaries are situated in the peritoneal cavity flanking the uterus.

The main component of semen is sperm.

False. Sperm is actually a minor component of seminal fluid. Most (about 70%) of semen fluid volume is attributed to secretions of the seminal vesicles. The prostate gland and bulbourethral glands also contribute to seminal fluid. The urethra does not add anything to seminal fluid.

Where in the metabolic pathway can fats be utilized as an energy source? Fatty acids can be converted into acetyl CoA, and glycerol can enter glycolysis. Fatty acids can enter the Krebs cycle, and glycerol can enter glycolysis. Fats are catabolized by a process called lipogenesis. Fats are used as an energy source in a process called glycogenolysis.

Fatty acids can be converted can glycolysis Correct The process of lipolysis involves glycerol's entering the glycolytic pathway while fatty acids are broken down into acetyl CoA molecules.

__________ is a sexually transmitted disease that is associated with cervical cancer. Trichomoniasis Syphilis Genital warts Genital herpes

Genital Warts HPV (human papillomavirus) causes genital warts. There are about 60 different types of HPV. Some papillomaviruses are sexually transmitted and cause genital warts, which are associated with cervical cancer. Some strains of HPV are more likely to cause cervical cancer than others. Fortunately, there is now a vaccination to protect women against the HPV most closely associated with cervical cancer.

Why is glucose in the urine an indicator of diabetes mellitus?

Glucose occupies all the transport carriers and it is no longer reabsorbed.

Which of the following represents the order in which reproductive hormones are secreted in males? FSH, GnRH, LH, testosterone LH, GnRH, FSH, testosterone GnRH, LH/FSH, testosterone, inhibin FSH, GnRH, testosterone, LH FSH, LH, GnRH, testosterone

GnRH, LH/FSH, testosterone, inhibin right answer feedback:The hypothalamic-pituitary-gonadal axis regulates the production of gametes and sex hormones in males.

__________ stimulates the anterior pituitary to release follicle-stimulating hormone (FSH) and luteinizing hormone (LH). Inhibin Progesterone Gonadotropin-releasing hormone estrogen

Gonadotropin-releasing hormone Gonadotropin-releasing hormone (GnRH) stimulates the anterior pituitary to release follicle-stimulating hormone (FSH) and luteinizing hormone (LH). In females, progesterone, estrogen, and inhibin discourage the secretion of FSH and LH.

__________ acts upon the __________ to encourage the release of follicle-stimulating hormone (FSH) and luteinizing hormone (LH). Testosterone; anterior pituitary Testosterone; sustentocytes Gonadotropin-releasing hormone (GnRH); anterior pituitary Gonadotropin-releasing hormone (GnRH); sustentocytes

Gonadotropin-releasing hormone (GnRH); anterior pituitary Gonadotropin-releasing hormone (GnRH) acts upon the anterior pituitary to encourage the release of follicle-stimulating hormone (FSH) and luteinizing hormone (LH).In males, FSH stimulates sustentocytes (Sertoli cells) to produce androgen-binding protein (ABP). In response to rising LH levels, testosterone is produced by interstitial endocrine cells.

What is the role of the granulosa cells in the ovarian cycle? -Granulosa cells become the secondary oocyte once the antrum is formed. -Granulosa cells guide the development of the oocyte. -Granulosa cells release LH to trigger ovulation. -Granulosa cells produce progesterone. Submit

Granulosa cells guide the development of the oocyte. Correct Granulosa cells are directly connected to the oocyte by gap junctions, allowing for direct communication between the oocyte and granular cells. These cells guide each other's development.

What directly stimulates the central chemoreceptors, thus increasing respiration? -H+ (hydrogen ions) low O2 (oxygen high CO2 (carbon dioxide) high pH

H+ (hydrogen ions) Correct Yes, hydrogen ions (H+) stimulate the central chemoreceptors. CO2 is converted to H+ in the extracellular fluid of the brain.

What is the role of helper T cells in the adaptive immune response?

Helper T cells activate B cells and cytotoxic T lymphocytes to kill infected host cells. Correct Helper T cells activate B cells that are displaying antigen, causing clonal expansion. Helper T cells also activate cytotoxic T cells, which will search for and destroy infected host cells.

What is the effect of high blood pressure on cardiac output?

High blood pressure increases afterload and reduces cardiac output (CO). Correct High blood pressure does produce a resistant force to systolic contractions, thus increasing afterload and end systolic volume (ESV). If ESV increases, stroke volume decreases.

Why does a graph of the membrane potential of living pacemaker cells never demonstrate a flat line? -Hyper-polarization of pacemaker cells triggers the opening of slow Na+ channels and starts a new slow depolarization phase. -The vagus nerve continually stimulates the pacemaker cells. -As pacemaker cells hyperpolarize, the excessive influx of K+K+ ions increases the membrane potential. -Repolarization of pacemaker cells triggers the opening of fast Ca2+Ca2+ channels, triggering another rapid depolarization.

Hyperpolarization of pacemaker cells triggers the opening of slow Na+ channels and starts a new slow depolarization phase. Correct At the end of an action potential, as repolarization and hyperpolarization occur, slow Na+ channels open, causing the membrane potential to immediately start to increase again (called the pacemaker potential), eventually triggering another action potential.

Which of the following are properly matched?

IgG: most abundant antibody Correct IgG is the most abundant class of antibodies in plasma. Additionally, IgG is the only class of antibodies that normally crosses the placenta to protect the baby in utero.

Which of the following is an effect of luteinizing hormone (LH) after ovulation? -It inhibits the formation of secretory glands in the mucosal endometrium during the secretive stage. -It maintains the corpus luteum. -It stimulates production of a new functional layer of the endometrium. -It thins the cervical plug to facilitate the passage of sperm. Submit

It maintains the corpus luteum. Correct LH maintains the corpus luteum. Increased levels of estrogen promote the release of LH at ovulation, but the increased levels of progesterone after ovulation inhibit the release of LH, which causes the degradation of the corpus luteum.

Focus your attention on the graph shown, from the left side of the Focus Figure. The percent of O2O2 saturation of hemoglobin is plotted (on the y-axis) against PO2PO2 (mm Hg) (on the x-axis). Use this graph to complete Parts A-C below. On this graph, the y-axis (the vertical edge) tells you how much O2O2 is bound to hemoglobin (Hb). At 100%, each Hb molecule has four bound oxygen molecules. The x-axis (the horizontal edge) tells you the relative amount (partial pressure) of O2O2 dissolved in the fluid surrounding the Hb. If more O2O2 is present, more O2O2 is bound. However, because of Hb's properties (O2O2 binding strength changes with saturation), this is an S-shaped curve, not a straight line. Which of the following represents a correct statement about data presented in the graph? -In blood with 30% oxygen saturation of hemoglobin, there is a PO2PO2 of ~60 mm Hg in surrounding fluid. -In blood with 60% oxygen saturation of hemoglobin, each individual hemoglobin binds ~2.5 oxygens. -Blood with three oxygens per hemoglobin represents a saturation of PO2PO2 at ~30 mm Hg. -In blood with a PO2PO2 of 30 mm Hg, the average saturation of all hemoglobin proteins is 60%.

In blood with a PO2 of 30 mm Hg, the average saturation of all hemoglobin proteins is 60%. Correct Cooperative binding of oxygen to an individual hemoglobin protein occurs with one oxygen per hemoglobin representing 25% saturation; two oxygens per hemoglobin, 50%; three oxygens per hemoglobin, 75%; and four oxygens per hemoglobin, 100% saturation. Only collectively with all hemoglobins can there be percentages in between, as seen in the S-shaped curve graph of saturation plotted against PO2PO2 of blood.

Choose the FALSE statement about oogenesis. In oogenesis, three polar bodies and one functional gamete are produced. In humans, the secondary oocyte must complete meiosis II before it can be fertilized. Oogenesis occurs in the ovaries. Primary oocytes are stalled in prophase I until puberty.

In humans, the secondary oocyte must complete meiosis II before it can be fertilized. Female infants are born with primary oocytes that are stalled in prophase I until puberty. At the onset of puberty, each month, one primary oocyte continues with meiosis I. During oogenesis, three polar bodies and one functional gamete are produced. Oogenesis is achieved through meiosis. However, the ovulated secondary oocyte arrests in metaphase II and does not complete meiosis II until it is fertilized.

Match the phases of meiosis to the events that occur in each phase. Using the drop-down menus, match each numbered item on the left with a lettered item on the right. telophase I---> two genetically unique haploid cells are formed metaphase I ---> tetrads align randomly along the cell's equator prophase I ---> crossovers form anaphase I ---> homologous chromosomes separate from one another telophase II ---> four genetically unique haploid cells are formed

In prophase I, crossing over occurs between replicated homologous chromosomes. Tetrads align randomly at the metaphase plate during metaphase I. During anaphase I, homologous chromosomes separate but sister chromatids remain together. Following telophase I, two genetically unique haploid cells are formed. After telophase II, four genetically unique haploid cells are formed.

Which statement is correct? -Oxygen is released from the mitochondria as a product of cellular respiration. -Carbon dioxide diffuses from the alveoli into surrounding capillaries. In the blood, oxygen is bound to hemoglobin, a protein found in red blood cells. -As oxygen diffuses from the lungs into capillaries, blood becomes deoxygenated. -Oxygen diffuses from large blood vessels into the body's cells.

In the blood, oxygen is bound to hemoglobin, a protein found in red blood cells. Correct When oxygen diffuses from the alveoli to the surrounding capillaries, it enters a red blood cell and binds to hemoglobin.

Which of the following is an effect of epinephrine, norepinephrine, and thyroxine?

Increase contractility Correct Epinephrine, norepinephrine, and thyroxine secreted by the sympathetic nervous system directly increase contractility and heart rate, for the overall effect of increasing cardiac output. Likewise, end systolic volume would decrease with increased contractility.

A muscle that is being regularly contracted during exercise will attract blood flow by dilating arterioles. Which of these factors would contribute to this effect?

Increased Levels of carbon dioxide Correct Carbon dioxide produced in muscle respiration would be a metabolic mechanism, which would dilate local arterioles and increase blood flow.

For each of the following, indicate whether it would cause an increase in aldosterone secretion, a decrease in aldosterone secretion, or no change in aldosterone secretion.

Increased aldosterone -dehydration -high plasma potassium levels -low plasma sodium levels Decreased aldosterone -high plasma sodium levels -low plasma potassium levels No change in aldosterone -balanced plasma sodium and potassium levels Correct ADH and aldosterone allow the reabsorption of water (via ADH) and sodium (via aldosterone) from the filtrate. These two hormones allow "fine-tuning" of the filtrate water, electrolyte, and acid-base composition in order to meet the ever-changing needs of the body.

Which of the following lowers blood pressure by reducing the sympathetic vasomotor response?

Increasing the secretion of atrial natriuretic peptide (ANP)

Which of the following statements about the foramen ovale is FALSE? It is one of the shunt systems encountered by blood entering and leaving the heart before birth. It is an opening in the interatrial septum. It allows blood to pass directly from the right atrium into the left ventricle. It serves to bypass the nonfunctional lung.

It allows blood to pass directly from the right atrium into the left ventricle. The foramen ovale allows blood to pass from the right atrium into the left atrium.

When considering drug administration, why is it important to know the renal clearance rate of the drug?

It dictates the dosage and frequency of administration of drugs.

Which of the following is not a change that occurs with old age?

Nocturia decreases

After blood becomes oxygenated. -it does not return to the heart, but goes directly to the lungs. -it returns to the heart, and is then pumped to the lungs. it does not return to the heart, but goes directly to capillaries that supply the body's cells with oxygen. -it does not return to the heart, but goes to the nose and mouth. -it returns to the heart, and is then pumped to body cells.

It returns to the heart, and is then pumped to body cells

What is the fate of the corpus luteum if pregnancy occurs? -It secretes progesterone and some estrogen for about three months. -It produces progesterone and some estrogen for about 10 days and then degrades. -It continues to stimulate the development of the oocyte until ovulation can occur. -It implants into the uterine wall to begin pregnancy. Submit

It secretes progesterone and some estrogen for about three months. Correct It implants into the uterine wall to begin pregnancy. Submit

Hypoalbuminemia is a medical condition in which blood plasma levels of albumin are abnormally low. One cause of this disorder is the increased movement of plasma albumin into the interstitial space through leaky blood capillaries. How would this condition affect the volume of lymph that is produced?

It would increase lymph volume.

What is the most abundant intracellular cation? Cl- K+ HPO4 2- Na+

K+ Correct K+ is the most abundant cation in the intracellular fluid.

__________ are considered "bad" cholesterol; high blood levels are correlated with increased risk of cardiovascular disease. Chylomicrons VLDLs HDLs LDLs

LDL's Correct LDLs are considered "bad" cholesterol because high levels are correlated with an increased risk of cardiovascular disease. The job of cholesterol-rich LDLs is to transport cholesterol to peripheral tissues, making it available to tissue cells to synthesize membranes or hormones, and to store it for later use.

A woman who wants to ensure conception might buy an "ovulation predictor" kit. Which of the following hormones, when detected by the kit, would be the best indicator of imminent ovulation?

LH

Which hormone stimulates the ruptured ovarian follicle to become the corpus luteum? FSH GnRH estrogen LH

LH right answer feedback:As indicated by its name, luteinizing hormone stimulates the ruptured ovarian follicle to become the corpus luteum, which will secrete large quantities of progesterone to maintain the stratum functionalis in case conception occurs.

The greater the mass of tissue in an organ, the greater its need for an adequate blood supply. Which chamber of the heart has the highest probability of being the site of a myocardial infarction? -right atrium -left atrium -left ventricle -right ventricle

Left Ventricle Correct The left ventricle actively pumps blood to almost all of the body's tissues via the aorta. It has to generate much greater force during contraction than any other heart chamber.

Water lost through expired air is referred to as __________. metabolic water water of oxidation insensible water loss water intake

insensible water loss

Which statement correctly describes the origin of lymph fluid?

Lymph is excess tissue fluid formed from plasma that has leaked from capillaries Correct As blood circulates through the body, nutrients, wastes, and gases are exchanged between the blood and the interstitial fluid. The hydrostatic and colloid osmotic pressures operating at capillary beds force fluid (plasma) out of the blood at the arterial ends of the beds and cause most of it to be reabsorbed at the venous ends. The fluid that remains behind in the tissue spaces, as much as 3 L daily, becomes part of the interstitial fluid. The lymphatic vessels collect this excess interstitial fluid (now called lymph) and return it to the bloodstream.

Which of the following is true about meiosis? Meiosis is how most cells in the body divide. Meiosis produces cells that have a haploid chromosome number. Meiosis leads to the production of genetically identical cells. Meiosis involves only one cell division event.

Meiosis produces cells that have a haploid chromosome number. Meiosis is used to produce haploid gametes. Most body cells divide by mitosis, not meiosis. Unlike mitosis, meiosis involves two rounds of cell division. Meiosis leads to the production of four genetically unique cells.

The student who caught the cold caused by this specific Rhinovirus was exposed to the exact same Rhinovirus 18 months later. What component of the immune system will protect her from getting the same cold again?

Memory B cells Correct Memory cells to that specific virus are stored in the lymph nodes for many years. When the student comes into contact with this specific Rhinovirus, these memory cells quickly divide and differentiate into antibody-producing plasma cells. The antibodies will prevent the virus from reaching an infectious titer (number that causes infection).

What roles do memory B cells play when a patient is re-exposed to an antigen?

Memory B cells trigger a secondary immune response, which is faster, more prolonged, and more effective than the first immune response. Correct Memory B cells that have already been activated by an antigen are primed and ready for action. They can respond very quickly to that antigen if it is encountered again, and the secondary response is faster and more effective.

Which of the following is NOT a property of mucous membranes?

Mucus contains macrophages that attack invading pathogens. Correct Mucus itself does not typically contain macrophages.

Review the entire Focus Figure 19.1 again. During your review, consider what would happen if one pressure were atypical. Read through Focus Figure 19.1. Mr. Orange has hypertension (HTN) and other vessel disorders, and his brachial blood pressure is ~160/100. Assume his HPc is ~44 mm and other pressures are unaffected. What is his NFPa?

NFPa = (44 + 1) - (0 + 26) = +19 mm Correct Increase in blood pressure will typically increase the net filtration pressure at the capillary bed.

The only cation exerting significant osmotic pressure in the ECF is __________. K+ Ca2+ Fe2+ Na+ Mg2+

Na+ right answer feedback:Na+ is the only cation exerting significant osmotic pressure in the ECF.

Which of the following transporters in the luminal membrane results in secretion? -Na+ -glucose cotransporter -potassium ion channel -glucose carrier transporter -Na+ -H+ countertransporter

Na+ -H+ countertransport Correct Yes, because this is a countertransporter, Na+Na+ is transported into the cell and H+H+ is transported out of the cell into the lumen.

Renal acid-base control mechanisms are coupled to __________. water intake plasma protein levels Na+ transport ADH secretion

Na+ transport

__________ are the structural and functional units of the kidneys, which carry out the processes that form urine. Major calyces Nephrons Renal pyramids Glomerular capsules

Nephrons

A recent measles outbreak in your community has sparked interest in antibody testing to determine which patients are immune, which are at risk for infection, and which patients actually have measles. Which patient status and lab values are incorrectly matched?

Never immunized and ill with measles for 3 days: IgG +/ IgM - Correct If your patient is experiencing an initial infection with measles, he or she should have only IgM this early in the course of this illness. This result would not be expected.

Baby A was scheduled for surgery to repair her atrial septal defect. The operation went well, and she is recovering nicely. Her mother wants to know whether she will need transfusions since her blood pressure was so low before the surgery. Will she need transfusions to maintain her systemic volume? -No. When she had the shunt, her body increased its blood volume to keep blood pressure up. For the rest of her life, her blood pressure will be too high. -Yes. Her body has become adapted to having low blood pressure and will not raise it up again. For the rest of her life, she will have low blood pressure. -No. Now that the shunt has been repaired, her blood is not being diverted from the systemic circuit into the pulmonary circuit, and her blood pressure should return to normal. -Yes. The shunt was causing excess blood to enter her lungs, and now that it is repaired, that excess blood has been removed.

No. Now that the shunt has been repaired, her blood is not being diverted from the systemic circuit into the pulmonary circuit, and her blood pressure should return to normal. Correct Good work! In this pediatric case, the nurse's job involves assessment, interpretation of lab results, and educating a concerned mother. Like many of us, patients often remember things much better when you present them in pictures. You have made Mrs. A feel much better, as she understands what is going on with her baby and how the treatment should help. She will be better able to identify any problems as she cares for her daughter at home.

Which of these would not be a risk factor for metabolic syndrome? Normal BMI Blood pressure of 140/90mmHG Blood glucose in the impaired range of 110-125mg/100ml. Blood HDL levels below 40mg/dl

Normal BMI Correct A body mass index (BMI) below 25 is considered normal would be associated with smaller waist circumference.

Which way would O2O2 and CO2CO2 diffuse during internal respiration? -Both O2 and CO2 would diffuse into the systemic capillaries. -O2 would diffuse into the systemic capillaries, and CO2CO2 would diffuse into the cells. -O2 would diffuse into the cells, and CO2 would diffuse into the systemic capillaries. -O2 would diffuse into the pulmonary capillaries and CO2CO2 would diffuse into the alveoli.

O2 would diffuse into the cells, and CO2CO2 would diffuse into the systemic capillaries. Correct Yes, the PO2 would be higher in the systemic capillaries, and the PCO2PCO2 would be higher in the tissues.

Which of the following stages is the true moment of fertilization?

Once the chromosomes of the male and female pronucleus are united

What is the major difference between spermatogenesis and oogenesis?

Oogenesis results in the formation of one viable oocyte.

What is the role of oxygen gas (O2) in aerobic cellular respiration? -Oxygen atoms act as electron acceptors in the electron transport chain. -Oxygen atoms provide the energy needed to generate ATP. -Oxygen must be available in the cytoplasm for glycolysis to occur. -O2 binds to carbon released from the citric acid cycle to form CO2.

Oxygen atoms act as electron acceptors in the electron transport chain Correct Oxygen atoms are highly electronegative. Oxygen atoms receive electrons exiting the electron transport chain. Negatively charg

__________ and __________ are both hormones and powerful uterine muscle stimulants that cause contractions to become more frequent and more vigorous. Relaxin; oxytocin Estrogen; progesterone Human chorionic gonadotropin; relaxin Oxytocin; prostaglandins

Oxytocin; prostaglandins As birth nears, certain fetal cells begin to produce oxytocin, which causes the placenta to release prostaglandins. Both hormones are powerful uterine muscle stimulants that cause contractions to become more frequent and more vigorous.

Which of the following hormones is NOT involved in the regulation of the number of sperm produced in the testes? testosterone gonadotropin-releasing hormone PSA inhibin

PSA right answer feedback:PSA is a prostate cancer marker and is not involved in the regulation of spermatogenesis.

__________ refers to events that occur from the time of fertilization (conception) until the infant is born. Gestation period Conceptus Embryo Pregnancy

Pregnancy Pregnancy refers to the events that occur from the time of fertilization (conception) until the infant is born. Gestation period extends by convention from the last menstrual period until birth.

Which of the following components in semen stimulates the uterus to contract in reverse peristalsis?

Prostaglandins

Which of the following CANNOT diffuse across a capillary wall to or from the surrounding interstitial fluid?

Proteins Correct Large molecules, such as proteins, are actively transported in pinocytotic vesicles. They are too large to pass through the intercellular clefts or fenestrations, which is how smaller water-soluble molecules cross the capillary wall.

Which of the following occurs 17-20 weeks into pregnancy, during the fetal period? Quickening occurs. The head of the fetus is nearly as large as the body. Sex is readily detected by observing the genitals. The eyes open.

Quickening occurs. At 17-20 weeks (month 5) quickening occurs (the mother feels spontaneous muscular activity of the fetus).

Which of the following is NOT a step used by cytotoxic T cells to kill infected host cells?

Recognition of infected host cell using its CD4 glycoprotein Correct The cytotoxic T cell uses its CD8 glycoprotein to bind to the MHC-I of an infected host cell.

Refer to Focus Figure 18.2, specifically the graph reflecting pressure changes in the cardiac cycle. Why is the dicrotic notch important?

Reflects an increase in aortic pressure as blood rebounds against the closed aortic semilunar valve Correct Review each of the pressure line graphs in Focus Figure 18.2. As ventricular pressure decreases below aortic pressure, the aortic semilunar valve closes. In response, blood rebounds against this closed valve, producing a slight increase in the aortic pressure. This is shown on the line graph as the dicrotic notch.

The right tricuspid valve prevents backflow of blood from the right ventricle into the _______________. -left atrium -left ventricle -pulmonary trunk -right atrium

Right Atrium Flow from the right ventricle to the right atrium is prevented by the right tricuspid valve.

Which part of the intrinsic conduction system normally initiates the depolarizing impulse that causes a heartbeat?

SA node Correct Like the rest of the intrinsic conduction system, the SA node contains pacemaker cells that spontaneously depolarize. The cells within the SA node, however, depolarize faster than the other cells within the system. This causes action potentials to initiate in the SA node rather than in any of the other structures.

Which of the following is NOT a function of saliva? Saliva contains enzymes that begin the chemical breakdown of proteins -Saliva dissolves food chemicals so that they can be tasted -saliva moistens food and helps compact it into a bolus Saliva cleanses the mouth

Saliva contains enzymes that begin the chemical breakdown of proteins Correct Saliva contains components with functions ranging from starch breakdown to fighting bacterial infection, but it does not contain any enzymes that digest protein.

Salivary amylase is a digestive enzyme not featured in the Concept Map. Which of the following describes its function? breaks down complex starches into smaller units, and where in the body does this occur? Salivary amylase breaks down complex starches into smaller units in the mouth. Salivary amylase breaks down lipids into fatty acids in the mouth. Salivary amylase breaks down proteins into amino acids in the stomach. Salivary amylase breaks down steroid hormones into cholesterols in the stomach.

Salivary amylase breaks down complex starches into smaller units in the mouth Correct Salivary amylase in saliva breaks down starch into oligosaccharide

When the patient stood up, his heart rate increased. This was most likely caused by which of the following?

The baroreceptor reflex activated by blood pooling in his lower body due to gravity. Correct An incompetent aortic valve will interfere with the left ventricle's job of moving blood from the lungs to the body. Blood will accumulate in the lungs, leading to pulmonary congestion.

Which of the following does NOT describe a function of trophoblast cells? They secrete hCG. They take part in placental formation. They display L-selectin molecules on their surface after blastocyst hatching. The inner layers of trophoblasts lose their plasma membranes and invade the endometrium.

The inner layers of trophoblasts lose their plasma membranes and invade the endometrium. right answer feedback:The outer layers of trophoblasts lose their plasma membranes and invade the endometrium.

Drag and drop the items below to the appropriate bin, depending if the item is associated with the short-term or long-term stress response.

Short-term stress response -activated by neural stimuli -hormone released; epinephrine -triggers the fight-or-flight response -targets the adrenal medulla Long-Term Stress Response -activated by hormonal stimuli -targets the adrenal cortex -hormone released; glucocorticoids -increased blood pressure; increased protein and fat catabolism Correct The long-term and short-term stress responses vary in their mechanisms: in the short-term response, stressful stimuli target the hypothalamus, which activates the sympathetic nervous system, ultimately impacting the adrenal medulla. The adrenal medulla releases the catecholamine hormones - epinephrine and norepinephrine - which result in many of the effects associated with the "fight or flight" mechanism. In the long-term response, stressors to the hypothalamus cause the release of corticotropin-releasing hormone, which targets the anterior pituitary gland. Adrenocorticotropic hormone travels in the blood and targets the adrenal cortex, resulting in the production of mineralocorticoids and glucocorticoids. The mineralocorticoids impact the kidney with increased sodium and water retention, while the glucocorticoids target the liver to increase production of glucose from fats and proteins, or mobilize them for energy.

The echocardiogram revealed an atrial septal defect as well as confirming the enlarged right ventricle and pulmonary trunk. Baby A's systemic blood pressure is also decreased. What is the most likely explanation? Since she has a pulmonary infection, more blood is being diverted to her lungs; this distends her pulmonary trunk and leaves less blood in the systemic circuit. -Her repeated pulmonary infections have weakened the right side of her heart, so it is enlarged. -Since blood is moving between her atria, the blood in her ventricles and arteries is staying where it is, distending those structures. -Since blood is moving from her systemic circuit into her pulmonary circuit, the pulmonary circuit is distended and the systemic circuit is low on blood.

Since blood is moving from her systemic circuit into her pulmonary circuit, the pulmonary circuit is distended and the systemic circuit is low on blood.

Why are the male testes located in the scrotum, outside the abdominopelvic cavity? -The testes begin their development outside of the abdominopelvic region. -Spermatogenesis is temperature sensitive. -Their external location is necessary for penile development. -Their external location influences the development of male secondary sexual characteristics.

Spermatogenesis is temperature sensitive. Correct Spermatogenesis, the production and maturation of sperm, requires a temperature lower than core body temperature.

Which of the following is a myogenic factor that influences blood flow?

Stretch of vascular smooth muscle Correct Myogenic control of vascular smooth muscle occurs in response to the stretch of the vessel because of blood pressure fluctuations.

Mr. G has mitral regurgitation. How has this caused a systolic murmur? First of all, let's sort out what is happening during systole and diastole.

Systole -Ventricles are contracting -Blood is leaving the heart -Semilunar Valves are open -AV valves are closed -Blood is flowing through semilunar valves Diastole -Ventricles are relaxed -AV Valves are open -Blood is entering the heart -Semilunar valves are closed -Blood is flowing through AV valves

Which of the following is/are the most specific internal defense against disease?

T cells Correct T cells are a part of the adaptive (specific) defenses against disease. They are involved in cell-mediated immunity as they defend the body against specific pathogens.

Which of the following is FALSE regarding the female sexual response? -The female sexual response is similar to that of males. -Female orgasm is not required for conception. -The final phase of the female sexual response is orgasm. -Testosterone is the hormone that is primarily responsible for female libido.

Testosterone is the hormone that is primarily responsible for female libido. The female and male sexual responses are similar. The final phase of the female sexual response is orgasm; however, orgasm is not necessary for conception to occur. It was originally thought that testosterone was the main regulator of libido in males and in females. However, recent evidence suggests that dehydroepiandrosterone (DHEA) is the main androgen that is associated with female libido.

What metabolic effect would you expect if someone developed an autoimmune disease in which the person's interstitial cells were destroyed?

Testosterone production would plummet.

What would happen to the SA node if a chemical blocker was used to reduce transport of Na+ into the pacemaker cells? -The SA node would depolarize more quickly, decreasing the heart rate. -The SA node would depolarize more quickly, increasing the heart rate. -The SA node would depolarize more slowly, reducing the heart rate. -The SA node would depolarize more slowly, increasing the heart rate. -There will be no change.

The SA node would depolarize more slowly, reducing the heart rate. Correct Diffusion of Na+ into the pacemaker cell causes a gradual depolarization of the cell membrane, called the pacemaker potential. If the rate of depolarization slows down, it will take longer for the membrane to reach threshold and trigger the next action potential, which will reduce heart rate.

What is the role of the atrioventricular bundle?

The atrioventricular bundle provides the only pathway for electrical signals to pass from the atria to the ventricles. Correct The atrial cardiac cells are not connected by gap junctions to the ventricular cardiac cells. The only means by which the electrical signals of the heart can pass from atria to ventricles is the atrioventricular bundle.

Refer to Focus Figure 18.2, specifically the pressure in the left atrium. Which statement best explains the second rise in atrial pressure on the line graph?

The bicuspid/mitral valve closes; blood is contained within the closed chamber, exerting pressure against the atrial wall. Correct As blood flows into the left atrium, it moves passively into the left ventricle. As the atrium contracts to squeeze remaining blood downward into the ventricle, the pressure in this chamber is reflected from this contraction by the initial rise on the left atrial-pressure graph. As blood fills the ventricle, the ventricle will eventually contract, resulting in a rise in ventricular pressure and forcing the bicuspid valve flaps to close superiorly. As the valve is now closed, the pressure rises slightly in the atrium because blood is forced against the walls of the closed chamber.

Which of the following does NOT correctly describe basic regulation of digestive activity? pH, stretch, and osmolarity are some of the stimuli that can trigger digestive reflexes. The enteric nervous system is under conscious control. Short reflexes act locally in the GI tract. Digestive system effectors are smooth muscle and glands.

The enteric nervous system is under conscious control Correct Neural control over the digestive system is through the autonomic nervous system, which does not function under conscious control mechanisms.

Why is it necessary to send some blood through the pulmonary circuit in a fetus?

The fetal lung tissue requires oxygen to allow for lung tissue development and growth.

Focus your attention again on Focus Figure 22.1. Focus Figure 22.1. A firefighter breathes in air normally as he enters a building following an explosion and fire. He has a meter that predicts the PO2PO2 will approximate 15 mm Hg in his tissue fluids as he actively moves about the room. Select the best statement. -The large changes in PO2 tissue environments cause only very small changes in hemoglobin O2 saturation, and no oxygen is needed. -The firefighter is at about 10% hemoglobin O2O2 saturation, and he requires an external air tank. -The firefighter's hemoglobin saturation will be about one oxygen per hemoglobin, and he will require an external air tank. -The S-shaped saturation curve of hemoglobin is flat at this PO2PO2, and O2O2 saturation doesn't change much with PO2PO2 changes in mm Hg.

The firefighter's hemoglobin saturation will be about one oxygen per hemoglobin, and he will require an external air tank. Correct An O2O2 saturation of hemoglobin of ~25% is insufficient for the firefighter moving about the room, and he should use external oxygen.

Which of the following differentiates the follicular and luteal phases of the ovarian cycle? -The luteal phase occurs directly before ovulation while estrogen levels are high, and the follicular phase occurs directly after ovulation when estrogen levels drop. -The follicular phase is the period during which the follicle secretes estrogen, and the luteal phase is when the follicle secretes luteinizing hormone. -The follicular phase occurs directly before ovulation, and the luteal phase occurs directly after ovulation. -The follicular phase ends with a peak in progesterone production that triggers ovulation and the start of the luteal phase.

The follicular phase occurs directly before ovulation, and the luteal phase occurs directly after ovulation. Correct The follicular phase is the period in which the dominant follicle prepares for ovulation and secretes large amounts of estrogen. The luteal phase occurs after ovulation as the corpus luteum secretes progesterone and some estrogen.

Why is red-green color blindness more predominant in males?

The gene for color blindness is on the X chromosome and is said to be sex-linked.

Which of the following occurs eight weeks into pregnancy? Sex is readily detected by observing the genitals. The eyes open. Quickening occurs. The head of the fetus is nearly as large as the body.

The head of the fetus is nearly as large as the body. At eight weeks (end of embryonic period) the head is nearly as large as the body, all major brain regions are present, and the first brain waves are generated in the brain stem. Question

Your clinical instructor wants a brief description of the four cardinal signs of inflammation. What would you tell him?

The heat and redness are caused by increased blood flow to the infected areas, and the pain and swelling are caused by excessive leakage of fluid from the capillaries into the infected area. Correct Your response is succinct and accurate, which are good skills for a clinician.

Now that you have reviewed normal blood flow, why does a patient with left-sided heart failure have a low systolic blood pressure? -The left side of the heart contains the ventricles, so it controls blood pressure. -The left side of the heart pumps blood into the body, so when it fails, less blood will be pumped into the arteries. -The left side of the heart does not directly move blood into the body, but when it fails, the right side will also fail, and that will decrease blood pressure out in the body. -The left side of the heart receives blood from the body, so when it fails, blood will build up in the blood vessels.

The left side of the heart pumps blood into the body, so when it fails, less blood will be pumped into the arteries. Correct Since the left ventricle is the one responsible for pumping blood into the arteries, it makes sense that left-sided heart failure would reduce the amount of blood sent into the arteries and therefore the systolic blood pressure. That is not what brought Mr. G to the doctor, though.

Mr. G talks in short bursts with frequent breaths. He tells you he used to feel breathless when he was walking, but now he has been having more and more trouble breathing even when he is lying down at night. When you listened to his lungs you heard fluid in them. Could this be due to his heart failure, or is it another problem? -The left side of the heart receives blood from the lungs and pumps it out to the body, so if it fails, blood will remain in the lungs. -If the left side of the heart fails, it will not pump enough blood to the body, and that means less blood will reach the right side of the heart and be sent to the lungs. -The left side of the heart does not have anything to do with the lungs, but if it fails, the right side is going to fail too, and then circulation through the lungs will decrease. -The left side of the heart pumps blood to the lungs, so if it fails, circulation in the lungs will decrease.

The left side of the heart receives blood from the lungs and pumps it out to the body, so if it fails, blood will remain in the lungs. Correct Difficulty breathing, or dyspnea, is a common side effect of left-sided heart failure. It happens when the left side of the heart is not able to move blood out of the lungs to the body fast enough. The right side of the heart, meanwhile, is pumping more blood into the lungs. As a result, the lung capillaries are overfilled, the pulmonary blood pressure is increasing, and more fluid is forced out of the capillaries into the lungs. Mr. G does not feel so bad during the day because when he stands or sits, gravity pulls a lot of his blood into his legs. When he lies down at night, though, much of that blood moves into his chest, and the amount of fluid going into his lungs increases. He may wake up with the feeling that he is drowning −− which is accurate, in a way!

What is the functional difference between a male urethra and a female urethra?

The male urethra is shared with the reproductive system.

Examine this reaction: starch⟶amylase->disaccharides⟶disaccharidase->monosaccharides If there is the normal amount of amylase present but less disaccharidase than usual, which of the following would most likely happen? -No starch could be broken down to disaccharides. -The normal amount of disaccharide would be produced, but fewer monosaccharides would be produced. -There would be no overall impact because the normal amount of amylase is present. -There would be less disaccharide and less monosaccharide produced.

The normal amount of disaccharide would be produced, but fewer monosaccharides would be produced. Correct Excellent! The first reaction will proceed as usual because all of the requirements are met, but the second reaction will be impaired because its enzyme is deficient.

What is the main difference between oogenesis and spermatogenesis in terms of meiosis? -Oogenesis produces three polar bodies, while spermatogenesis produces only one. -Oogenesis does not include a second meiotic division. -Oogenesis takes place in the uterus, while spermatogenesis takes place in gonadal tissue. -The number of functional gametes produced is different.

The number of functional gametes produced is different. Correct In spermatogenesis, four functional gametes are produced, while oogenesis produces only one functional gamete because of the loss of three polar bodies.

Match the following. Using the drop-down menus, match each numbered item on the left with a lettered item on the right. Uterus -> receives, retains, and nourishes a fertilized ovum fimbriae->fingerlike projections that brush over the ovary uterine (fallopian) tubes->generally the site of fertilization of the ovulate oocyte vagina -> thin-walled structure that connects the uterus to the body exterior ovaries -> house and form the female gametes

The ovaries are the primary sex organs in females: They produce and store the female gametes. The uterine (fallopian) tubes are generally the site of fertilization of the ovulated oocyte. Since the uterine tubes are not directly connected to the ovaries, the ends of the tubes that are near the ovaries are equipped with fimbriae. Fimbriae are fingerlike projections that brush over the ovary to sweep ovulated eggs into the uterine tubes. The fertilized ovum is received, retained, and nourished by the uterus. The inferior end of the uterus is called the cervix; it connects the body of the uterus to the vagina. The vagina is a thin-walled structure that connects the uterus to the body exterior.

Which of the following is FALSE regarding the vagina of an adult female? -The vagina is also called the birth canal. -The pH of the adult vagina is alkaline. -The vaginal mucosa lacks glands. -The mucosa of the vagina is stratified squamous epithelium.

The pH of the adult vagina is alkaline. Correct The pH of the vagina is acidic

When blood glucose levels are low -The pancreas releases insulin, which eventually causes blood glucose levels to increase. -The pancreas releases glucagon, which eventually causes blood glucose levels to increase. Liver cells convert more glucose to glycogen. -The pancreas releases glucagon, which eventually causes blood glucose levels to decrease. -The pancreas releases insulin, which eventually causes blood glucose levels to decrease.

The pancreas releases glucagon, which eventually causes blood glucose levels to increase.

Which statement best describes the autonomic nervous system's role in regulating heart rate?

The parasympathetic division normally inhibits the heart, keeping it beating at a slower rate than it would be on its own Correct The inhibitory signals from the vagus nerve inhibit the heart rate, thus keeping the heart rate lower than it would be if the vagus nerve were severed.

How would the partial pressures of O2O2 and CO2CO2 change in an exercising muscle? -The partial pressure of O2O2 would increase, and the partial pressure of CO2CO2 would decrease. -The partial pressures of O2O2 and CO2CO2 would remain unchanged. -The partial pressure of O2O2 would decrease, and the partial pressure of CO2CO2 would increase.

The partial pressure of O2O2 would decrease, and the partial pressure of CO2CO2 would increase. Correct Yes, cells use O2O2 and produce CO2CO2 during cellular respiration to produce ATP. Exercising muscles need more ATP.

Which of the following is true of a patient with arterial blood pH of 7.6? The patient has alkalosis but is in the normal homeostatic range. The patient has alkalosis and is out of the normal homeostatic range. The patient has acidosis and is outside of the normal homeostatic range. The patient has acidosis but is in the normal range.

The patient has alkalosis and is out of the normal homeostatic range Correct The normal pH range of arterial blood is 7.4 to 7.45. A higher pH is a sign of alkalosis

A woman in her 24th week of pregnancy fails a glucose challenge test and is diagnosed with gestational diabetes. Once the baby is born, the woman's glucose regulation is back to normal. How can this be explained?

The placenta releases the hormone human placental lactogen, which interferes with glucose uptake.

Which of the following descriptions accurately describes Boyle's law? -How well a gas dissolves in a liquid such as blood depends on both its partial pressure and its solubility. -The pressure of gas in your lungs is inversely proportional to the volume in your lungs. -The partial pressure of a gas in the air you breathe in is equal to the total atmospheric pressure times the fractional concentration of the gas.

The pressure of gas in your lungs is inversely proportional to the volume in your lungs. Correct Yes, Boyle's Law describes how air moves into and out of the lungs during inspiration and expiration. By changing the volume of the thoracic cavity, the pressure changes in the lungs. Increasing volume of the thoracic cavity leads to a decreased pressure, causing air to flow into the lungs (down its pressure gradient) and thus causing inspiration.

Which of the following represents a correct statement about pressures at the arteriolar side of the capillary bed?

The pressures directed into the blood at the arteriolar end are OPc and HPif. Correct Two pressures force fluid into interstitial fluid, HPc pushes and OPif pulls. Two other pressures, HPif pushing and OPc pulling, move fluids into the blood.

Match the following. Using the drop-down menus, match each numbered item on the left with a lettered item on the right. tunica albuginea-->fibrous capsule of the testes seminiferous tubules ---> site of sperm production in the testes epididymis ---> site of sperm storage pampiniform venous plexus-->cools arterial blood to keep the testes below core temperature interstitial endocrine cells---> make testosterone

The production of sperm is optimal at about 3°C below core body temperature. The pampiniform venous plexus is an adaptation that cools arterial blood that is being delivered to the testes to keep them below core temperature.Each testicle is surrounded by two layers (tunics). The outer tunic is the two-layered tunica vaginalis. Deep to this serous layer is the tunica albuginea, which is the fibrous capsule of the testis.The production of sperm occurs in the seminiferous tubules, which are packed into the lobules of the testis. Surrounding each seminiferous tubule are three to five layers of smooth muscle-like myoid cells. By contracting, these cells help to squeeze sperm and testicular fluids through the tubules and out of the testes. Sperm are stored in the epididymis.Sperm production requires testosterone. Testosterone is produced in the testes by interstitial endocrine cells (Leydig cells).

Which statement correct regarding the ventricles? -The right ventricle empties into the pulmonary trunk -the left ventricles empties into the pulmonary circuit -the right ventricle forms a complete circle in cross section -the left ventricle has a thinner wall than the right ventricle

The right ventricle empties into the pulmonary trunk Flow from the right ventricle to the right atrium is prevented by the right tricuspid valve.

Part complete How is the adaptive third line of defense able to target specific pathogens?

The third line of defense responds to antigens on the pathogen. Correct All foreign invaders of the body present antigens that can activate the adaptive immune response. The adaptive immune response can then produce antibodies that bind to the antigens to help identify, disable, and destroy the pathogens.

What role do the vasa recta play in urine formation?

The vasa recta protect the medullary osmotic gradient by preventing rapid removal of salt.

Which feature of the bladder predisposes it to being able to stretch and relax repeatedly?

The walls are highly folded into rugae and the epithelium is transitional.

Examine the left side of page 1 of Focus Figure 19.1. Assume you have a U-tube with a semipermeable membrane (boundary) separating the solutions on either side. Predict what will occur to the water level if large, non-diffusible solutes are added only to the left arm of the tube.

The water level on the left side of the U-tube will rise. Correct Remember that only nondiffusible solutes will affect the water level, because the solute concentration will increase on the left side, creating osmotic pull. In the blood, nondiffusible solutes are large plasma proteins pulling or drawing fluid into the blood.

Mr. G's online file also has the results of his recent blood tests. The only thing that looks abnormal is something called ANP - atrial natriuretic peptide. It is elevated. Why would this hormone be elevated in Mr. G, and what is it doing to him? -his hormone comes from damaged ventricles and indicates that Mr. G's left ventricle is beginning to fail. -This hormone comes from the atria and causes the atria and causes the body to release ore K+ in the urine protecting Mr. G against a possibly fatal arrhythmia. -This hormone is the normal secretion from heart muscle cells, and it is elevated because Mr. G's left ventricle is larger. =This hormone comes from the overstretched atrium and makes Mr. G lose more Na+ and water in his uring, lowering blood volume reducing the load on the atrium. -This hormone controls sodium and water retention and is secreted by the adrenal cortex as part of the central response that raises blood pressure.

This hormone comes from the overstretched atrium and makes Mr. G lose more Na+ and water in his urine, lowering blood volume reducing the load on the atrium. Correct Even though they both had rheumatic fever, you can see that Mr. G and Mr. V have completely different heart problems −− one has a leaky valve, and the other a partly blocked valve; one has a systolic murmur, and the other a diastolic murmur. This sort of thing will really confuse you unless you work it out carefully step by step!

Which statement describes the citric acid cycle? This process uses energy captured from electrons flowing to oxygen to produce most of the ATPs in cellular respiration. This process splits glucose in half and produces 2 ATPs for each glucose. This process joins 2 pyruvic acid molecules into a molecule of glucose. This process produces some ATP and carbon dioxide in the mitochondrion. This process converts pyruvic acid to acetyl CoA.

This process produces some ATP and carbon dioxide in the mitochondrion. Correct The citric acid cycle breaks down carbon molecules, releasing carbon dioxide and forming some ATP.

Which statement describes glycolysis? This process joins 2 pyruvic acid molecules into a molecule of glucose. This process uses energy captured from electrons flowing to oxygen to produce most of the ATPs in cellular respiration. This process splits glucose in half and produces 2 ATPs for each glucose. This process produces some ATP and carbon dioxide in the mitochondrion. This process converts pyruvic acid to acetyl CoA.

This process splits glucose in half and produces 2 ATPs for each glucose Correct In glycolysis, glucose is split into two molecules of pyruvic acid. The released energy is stored in ATP and the electron carrier NADH.

Which statement describes the electron transport chain? This process joins 2 pyruvic acid molecules into a molecule of glucose. This process converts pyruvic acid to acetyl CoA. This process produces some ATP and carbon dioxide in the mitochondrion. This process splits glucose in half and produces 2 ATPs for each glucose. This process uses energy captured from electrons flowing to oxygen to produce most of the ATPs in cellular respiration.

This process uses energy captured from electrons flowing to oxygen to produce most of the ATPs in cellular respiration. Correct In the electron transport chain, electrons move from one electron carrier to another, eventually reaching oxygen. The released energy is used to make ATPs.

Alcohol and many drugs prescribed for hypertension are examples of diuretics. True False

True

In situations in which there is an extreme change of blood pressure (e.g., mean arterial pressure of less than 80 mm Hg), extrinsic controls take precedence over intrinsic blood pressure controls. True False

True

The descending limb of the nephron loop is relatively impermeable to solutes and freely permeable to water. True False

True

A person's genetic makeup is referred to as his or her genotype. True False

True A person's genetic makeup is referred to as his or her genotype. The way that genotype is expressed in the body is called one's phenotype.

An Apgar score of 8 to 10 indicates a healthy baby. True False

True An Apgar score of 8 to 10 indicates a healthy baby.

A weight gain of approximately 13 kg (about 28 lb) usually occurs during pregnancy. True False

True Because some women are over- or underweight before pregnancy begins, it is almost impossible to state the ideal or desirable weight gain. However, summing up the weight increases resulting from fetal and placental growth, increased size of the maternal reproductive organs and breasts, and greater blood volume during pregnancy, a weight gain of approximately 13 kg (about 28 lb) usually occurs

If your core temperature becomes colder, it is more difficult for oxygen to dissociate from hemoglobin at any PO2. -True -False

True Correct A decrease in temperature increases hemoglobin's binding affinity for O2, making it more difficult to dissociate (unload) O2 from hemoglobin. This is illustrated by the leftward shift of the oxygen-hemoglobin dissociation curve.

Lymph arrives at the lymph nodes via afferent lymphatic vessels.

True Correct Afferent vessels deliver lymph to the lymph node. The lymph then filters through the subcapsular sinus of the node, then into a number of smaller sinuses that cut through the node's cortex and lead into the medulla. Eventually, lymph exits the node at the hilum as it is carried away by efferent vessels.

The hydrostatic pressure in glomerular capillaries is the chief force pushing water and solutes out of the blood and across the filtration membrane. True False

True The net filtration pressure (NFP), responsible for filtrate formation, involves forces acting at the glomerular bed. Glomerular hydrostatic pressure (HPg), which is essentially glomerular blood pressure, is the chief force pushing water and solutes out of the blood and across the filtration membrane.

By the end of the third month of pregnancy, the placenta is usually fully functional as a nutritive, respiratory, excretory, and endocrine organ. True False

True The placenta is usually fully functional as a nutritive, respiratory, excretory, and endocrine organ by the end of the third month of pregnancy.

Human chorionic gonadotropin (hCG) levels spike within the first two months of pregnancy and then sharply decline by four months. True False

True Usually detectable in the mother's blood one week after fertilization, blood levels of human chorionic gonadotropin (hCG) continue to rise until the end of the second month. Then blood levels decline sharply to reach a low value by four months, a situation that persists for the remainder of gestation.

The urethra has two functions in men but only one in women. True False

True feedback:The length and functions of the urethra differ in the two sexes. The male urethra has a double function: it carries semen as well as urine out of the body.

Which layer of the typical vessel can be regulated to constrict or dilate the lumen?

Tunica Media Correct The tunica media consists of smooth muscle that is regulated by the sympathetic nervous system. It is used to regulate the diameter of the lumen.

In children with infant respiratory distress syndrome (IRDS), the walls of the alveoli cling to each other and make them difficult to inflate. It is common in babies born prematurely. What cells in these infants are NOT fully developed and are NOT doing their job? -endothelial cells of alveolar capillaries -alveolar macrophages -type II alveolar cells -type I alveolar cells

Type II Alveolar Cells Correct Type II alveolar cells secrete a detergent-like surfactant that lessens the surface tension on the alveolar walls, preventing them from sticking to each other. Infants with IRDS can be treated until their cells produce adequate surfactant.

Why does only one egg, rather than four eggs, develop during oogenesis, given that spermatogenesis results in four sperm formed from one stem cell? -Once formed, spermatids, but not oocytes, undergo additional rounds of division by mitosis. -Only one egg can be fertilized at a time. -Unequal cytoplasmic division ensures that a fertilized egg has ample nutrients for its journey to the uterus. -The egg does not go through the meiotic division processes that the sperm undergo.

Unequal cytoplasmic division ensures that a fertilized egg has ample nutrients for its journey to the uterus. Correct The unequal cytoplasmic division that results in one egg and three polar bodies ensures that a fertilized egg has ample nutrients for its journey to the uterus. Lacking nutrient-containing cytoplasm, the polar bodies degenerate and die. This is very different than the result of sperm formation.

Which of the following constitutes the largest solute component of urine?

Urea

Enteric interneurons that are inhibitory to smooth muscle use which of the following neurotransmitters? substance P ACh (acetylcholine) norepinephrine VIP (vasoactive intestinal peptide)

VIP (vasoactive intestinal peptide) Correct Yes, both VIP and NO (nitric oxide) are inhibitory to smooth muscle.

During exercise, cardiac output may increase by more than 170%% to meet the body's increased O2O2 demands. This increase in cardiac output increases blood pressure. But the accompanying increase in arterial pressure is relatively small—only about 40%%. What limits this increase in blood pressure so that it doesn't reach dangerously high levels during exercise?

Vasodilation causes arterial diameter to increase in the exercising skeletal muscle. Correct During exercise, total peripheral resistance decreases. This is primarily due to an increase in the diameters of arterioles supplying the exercising muscles. Vasodilation of the blood vessels increases blood flow, providing the oxygen and nutrients necessary to support increased muscle metabolism. In addition, vasodilation of the arterioles in the skin also allows for greater blood flow to the skin for evaporative cooling.

What is happening during the "pause" phase when the heart is resting (relaxing)? -Ventricles are filling -Valves are closing -Ventricles are contraction -Atria are contractin

Ventricles are filling

Which of the following is NOT a fat-soluble vitamin? vitamin K vitamin B vitamin A vitamin D

Vitamin B Vitamin B is a water soluble vitamin

Which statement about acids is true? When high in the body, they cause alkalosis. Weak acids only dissociate partially in solution. They have little physiological effect on chemical reactions in cells. Most acid is a result of ingestion with food. Our bodies have few ways to deal with excess amounts of acids.

Weak acids only dissociate partially in solution right answer feedback:Strong acids dissociate fully, and weak acids dissociate incompletely.

If a person exercises for a long time, lactic acid will start to build up in his or her muscles. Which of the following would you expect to happen as the lactic acid first starts to be formed? -Weak acids in the muscles will act as buffers and resist or minimize any pH change. -More water will move to the muscles to buffer the acid. -More water will move to the muscles to move out the acid. -Weak bases in the muscles will act as buffers and resist or minimize any pH change.

Weak bases in the muscles will act as buffers and resist or minimize any pH change. Excellent! Bases buffer acids, and acids buffer bases.

From a hormonal standpoint, why is the first day of menstrual bleeding counted as the first day of a new cycle rather than as the end of the previous cycle?

Without feedback from a fertilized zygote, the corpus luteum degenerates and pituitary hormones are no longer under ovarian negative feedback.

An embryo in week 9 of development is revealed to be male. Which of the following structures would be present in this embryo?

Wolffian ducts

Suppose a patient develops a myocardial infarction that disables the sinoatrial node. Would the heart still pump blood to the aorta and the pulmonary trunk? -No, because there will be no contraction of the atria. -Yes, because the ventricles will depolarize on their own without nodal stimulation at a rate of 50 times per minute. -No, because there would be no more connection between the atrioventricular node and the bundle branches. -Yes, because the atrioventricular node will still stimulate ventricular systole.

Yes, because the atrioventricular node will still stimulate ventricular systole. Correct The atrioventricular node spontaneously depolarizes similarly to the sinoatrial node, but more slowly. It can lead to the ventricles pumping blood to the aorta and pulmonary trunk around 50 times per minute.

Which of the following does NOT prevent polyspermy? the zona reaction a decrease in zonal inhibiting proteins the release of enzymes into the extracellular space beneath the zona pellucida the release of Ca2+ after the sperm has entered the ovum the cortical reaction

a decrease in zonal inhibiting proteins right answer feedback:There is an increase in zonal inhibiting proteins after the sperm enters the ovum, which destroy sperm receptors on the ovum, preventing polyspermy.

An increase in blood CO2 levels leads to __________. a decrease in carbonic acid an increase in blood pH a drop in blood pH a decreased respiratory rate

a drop in blood pH Correct An increase in CO2 results in the formation of more carbonic acid, which leads to an increase in acidity, or a drop in pH.

Which of the following would induce the loss of oxygen from the hemoglobin and the blood? -decreases in plasma carbon dioxide -a drop in blood pH increase in hemoglobin that has -oxygen bound to it already -a decrease in blood temperature

a drop in blood pH Correct The pH in blood tends to drop when plasma reacts with carbon dioxide, a common condition in tissue. This pH drop causes weakening of the Hb-O2 bond, a phenomenon called the Bohr effect.

Which of the following is most likely to trigger the release of renin? -an increase in aldosterone production -a drop in blood pressure -an increase in water reabsorption -an increase in blood pressure

a drop in blood pressure When baroreceptors in the heart and blood vessels sense a drop in blood pressure, they trigger juxtaglomerular cells of the kidney to release renin.

Which of the following is NOT an important source of resistance to blood flow?

total blood volume Correct Total blood volume is not an important source of resistance to blood flow. However, blood volume does have a direct effect on blood pressure.

Which of the following is NOT produced by meiosis? - a sperm cell -a liver cell -an egg, or ovum - all of the given choices are produced by meiosis

a liver cell Correct Liver cells are somatic cells, and they are all produced by mitosis, not by meiosis.

At what point in the cardiac cycle is pressure in the ventricles the highest (around 120 mm Hg in the left ventricle)?

ventricular systole Correct Left ventricular systole typically produces maximum pressures of around 120 mm Hg. Provide FeedbackNext

A major stimulus for the release of antidiuretic hormone is __________. -an increase in potassium levels in the ICF -stimulation of the baroreceptors -a rise in plasma osmolality -an increase in ECF water concentration -inhibition of the osmoreceptors

a rise in plasma osmolality right answer feedback:A major stimulus for the release of antidiuretic hormone is a rise in plasma osmolality. ADH results in increased water reabsorption.

Which of the following is NOT one of the most common causes of incontinence in adults? nervous system problems a side effect after general anesthesia physical pressure during pregnancy emotional problems

a side effect after general anesthesia

Ovulated oocytes enter the fallopian tubes:

via undulations of the fimbriae that draw in the oocyte.

Breast-feeding provides several advantages to newborns. Which of the following is (are) (a) benefit(s) of breast-feeding?

a)Breast milk, especially colostrum, is rich in maternal antibodies. b)The fat in breast milk is fully absorbed by the infant. c)Breast milk contains antimicrobial molecules. ***d)All of the above are benefits of breast-feeding.

Estrogens are chemically similar to __________ and are therefore associated with water retention. ADH aldosterone ANP rennin

aldosterone

The pH of the adult female vagina is normally __________. neutral basic acidic alkaline

acidic The pH of the adult vagina is acidic. This is because bacteria in the vagina ferment glycogen to form lactic acid. This acidity protects the vagina from a number of pathogens.In adolescents, the vaginal fluid tends to be alkaline (basic). Question

In response to a steak dinner, certain secretions are needed to aid digestion. What cells in the pancreas would provide these secretions? duct cells islet of Langerhans cells acinar cells

acinar cells Correct Yes, in response to a high fat and protein meal, CCK would be stimulated and in turn would stimulate an enzyme-rich secretion from the pancreas.

Which portion of the sperm contains the enzymes needed to penetrate the oocyte? -midpiece -acrosome -head -tail

acrosome Correct The acrosome contains the hydrolytic enzymes needed to digest the zona pellucida on the oocyte.

Which specific type of cell produces antibodies?

activated plasma cells Correct When activated by the presence of antigens, B lymphocytes proliferate into plasma cells. Plasma cells generate antigen-specific antibodies.

Costanza was picking grapes in her father's arbor when she felt a short prickling pain in her finger. She ran crying to her father, who removed an insect stinger and calmed her with a glass of lemonade. Twenty minutes later Costanza's finger was red, swollen, and throbbing where she had been stung. What type of immune response was she exhibiting?

acute, type I hypersensitivity reaction Correct Immediate hypersensitivities, also called acute or type I hypersensitivities, are simply what most of us would call allergies. An allergen is an antigen that causes an allergic reaction. Allergic reactions, mediated by IgA antibodies, begin within seconds after contact with the allergen and last about half an hour. Since histamines (and other inflammatory chemicals) induce the inflammatory response typical of allergies, antihistamines are most commonly used to treat this condition.

Which of the following tissues is the least hydrated? adipose tissue bone muscle tissue nervous tissue

adipose tissue

The blood supply leading directly into the nephron is the __________. -efferent arteriole -segmental artery -renal artery -interlobular artery -afferent arteriole

afferent arteriole

The myogenic mechanism of renal autoregulation primarily involves smooth muscle in which blood vessels? systemic arterioles glomeruli efferent arterioles afferent arterioles

afferent arterioles Correct By regulating afferent arteriole diameter, the myogenic mechanism affects HPg and therefore GFR. Under normal physiological conditions, this intrinsic control works to maintain GFR despite moderate changes in systemic blood pressure.

Drinking too much alcohol results in a headache the next day. Why does this happen?

alcohol inhibits ADH secretion

What hormone promotes active tubular secretion of potassium ions in the reabsorption of sodium ions in the distal convoluted tubule (DCT) and collecting ducts? -ADH antidiuretic hormone -aldosterone -PTH parathyroid hormone -ANP atrial natriuretic peptide

aldosterone Correct Aldosterone from the adrenal cortex promotes active tubular secretion of potassium ions in the late distal convoluted tubule (DCT) and collecting ducts. It is secreted in indirect response to actions of the juxtaglomerular apparatus.

The reabsorption of sodium in the DCT is regulated primarily by __________. aldosterone and ANP parathyroid hormone aldosterone renin and angiotensin

aldosterone and ANP

When the concentration of Na+ in the ECF decreases, __________. aldosterone levels increase a person experiences increased thirst more ADH is released osmoreceptors are stimulated ANP levels increase

aldosterone levels increase right answer feedback:When the concentration of Na+ in the ECF decreases, there is an increase in the level of aldosterone, which causes facultative Na+ reabsorption.

Which of the following hormones do males secrete? a)Testosterone b)FSH c)LH d)All of the above

all of the above

Which of the following might be reported by a man with benign prostatic hypertrophy (BPH)? -feeling of the bladder is full, even just after urinating -having to strain to urinate -pain after ejaculation or during urination -All of the above

all of the above Correct Use your textbook to review the anatomy of the prostate gland, bladder and urethra to understand how the function of these organs can be impacted by BPH.

In humans, the extraembryonic membrane that composes the structural base for the umbilical cord is the __________. yolk sac chorion allantois amnion

allantois In humans, the allantois is the structural base for the umbilical cord that links the embryo to the placenta.

A(n) _____________ is an alternative version of a trait

allele

Alternative forms of genes are called __________. phenotypes autosomes genomes karyotypes alleles

alleles Alternative forms of genes are called alleles. Many alleles are either dominant or recessive.

Which of the following is/are part(s) of the respiratory zone structures? -alveoli -terminal bronchioles -trachea -primary bronchi

alveoli Correct Alveoli are thin-walled air sacs that are the sites of gas exchange. Along with the respiratory bronchioles and alveolar ducts, the alveoli make up the respiratory zone.

From which structures do oxygen molecules move from the lungs to the blood? -alveoli =bronchioles -bronchi -nose -trachea

alveoli Correct Alveoli are tiny sacs in the lungs surrounded by capillaries. The alveoli are where oxygen diffuses from the lungs to the blood.

The __________ of the mammary glands are responsible for producing milk during lactation. lactiferous sinuses lactiferous ducts alveoli areolae

alveoli The lobules of the mammary glands contain glandular alveoli that are responsible for milk production when a woman is lactating. The alveoli then pass the milk they produce to the lactiferous ducts, which open to the outside at the nipple. Just deep to the areola, each lactiferous duct has a dilated region called a lactiferous sinus, where milk accumulates during nursing.

Of the various types of fetal testing, which is the most invasive?

amniocentesis

When a woman's "water breaks" just before delivery, it is the _______ that ruptures.

amnion

Which enzyme(s) is/are responsible for the chemical breakdown of carbohydrates? protease samylases nuclease trypsin

amylase

Quiet inspiration is __________, and quiet expiration is __________. -an active process; a passive process -an active process; also an active process -a passive process; also a passive process -a passive process; an active process

an active process; a passive process Correct During quiet breathing, inspiration requires muscular contractions of the diaphragm and external intercostals, while expiration occurs passively due to the elastic recoil of the lungs and the collapsing force of alveolar fluid surface tension.

What is the name of the unique area (specific region) that a lymphocyte recognizes and binds to?

an antigenic determinant Correct Yes, also called an epitope, the antigenic determinant is the specific part of an antigen that the antibody or lymphocyte antigenic determinant receptor binds to.

A decrease in blood pressure at the arterial baroreceptors would result in which of the following?

an increase in heart contractility Correct Yes, sympathetic nervous system activity would be increased because of the low blood pressure. Sympathetic fibers go to the ventricles of the heart and increase their contractility. An increase in contractility would increase stroke volume which would lead to an increase in cardiac output and blood pressure.

Stimulation of the adrenal medulla would result in which of the following?

an increase in heart rate and contractility Correct Yes, epinephrine and norepinephrine are released from the adrenal medulla and act as part of the sympathetic nervous system, increasing heart rate and contractility. Epinephrine and norepinephrine have other effects that would also increase blood pressure.

Which of the following individuals would have the most body water? a healthy young male an older obese person a healthy young female an infant

an infant

LH stimulates the production of _______ by the follicular thecal cells.

androgens

Which of the following chemicals do NOT directly trigger inflammation?

antibodies Correct Antibodies are proteins that recognize specific foreign antigens. Antibodies trigger complement reactions, which may trigger inflammation, but this is not a direct pathway.

Which molecules of the adaptive defense system provide humoral immunity by circulating freely in the blood and lymph, where they bind to extracellular antigens and inactivate them and mark them for destruction?

antibodies Correct Humoral immunity, also called antibody-mediated immunity, is provided by antibodies present in the body's "humors," or fluids (blood, lymph, etc.). Though they are produced by lymphocytes, antibodies circulate freely in the blood and lymph, where they bind primarily to extracellular targets—bacteria, bacterial toxins, and free viruses—inactivating them temporarily and marking them for destruction by phagocytes or complement.

Which of the following best describes humoral immunity?

antibodies being released in the body fluids Correct Humoral immunity refers to the presence of specific antibodies in the body fluids.

Which of the following homeostatic imbalances is indicative that glomerular blood pressure may be too low to cause filtration? anuria hydronephrosis pyelitis renal ptosis

anuria

Redox reactions __________. -are characterized by the exchange of water molecules -are characterized by one substance's gaining an electron while another substance loses an electron -do not occur under physiological conditions -rarely involve enzymes

are characterized by one substance's gaining an electron while another substance loses an electron

Vaccines provide what type of immunity?

artificially acquired active Correct When your B cells encounter antigens and produce antibodies against them, you are exhibiting active humoral immunity. Active immunity is acquired "artificially" when you receive a vaccine. Most vaccines contain pathogens that are dead or attenuated (living, but extremely weakened), or their components. Vaccines spare us most of the symptoms and discomfort of the disease that would otherwise occur during the primary response.

What is the most common method of carbon dioxide transport? -as bicarbonate ions in the plasma -chemically bound to hemoglobin as oxyhemoglobin -dissolvfed in the plasma -chemically bound to hemoglobin as carbaminohemoglobin

as bicarbonate ions in the plasma Correct Carbon dioxide reacts with water inside RBCs to form carbonic acid, which dissociates into bicarbonate and hydrogen ions. About 70% of carbon dioxide travels in the plasma as bicarbonate.

The heart begins to beat in the developing offspring __________. at about 3 ½ weeks at 3 to 4 months at birth at conception when it becomes a fetus

at about 3 ½ weeks right answer feedback:The heart begins to beat at about 3 ½ weeks. All organ systems are laid down in the embryonic period.

At which point in the uterine cycle are the ovarian hormone levels at their lowest? -at the beginning of the proliferative phase -about one week after ovulation -at the beginning of the menstrual phase -at ovulation

at the beginning of the menstrual phase Correct The ovarian hormone levels are lowest at the beginning of the menstrual phase. Their low levels are what trigger menstruation.

When does ovulation occur in the uterine cycle? -at the end of the secretory phase -at the end of the proliferative phse -at the end of the luteal phase -at the end of the menstrual period

at the end of the proliferative phase Correct A surge in luteinizing hormone triggers ovulation at the end of the proliferative phase of the uterine cycle, around day 14.

Any two matched genes that are __________ are called alleles. at the same position on the sex chromosomes found only on autosomes at the same locus on homologous chromosomes found only in the mother

at the same locus on homologous chromosomes Alleles are genes that are at the same locus on homologous chromosomes whether they are autosomes or sex chromosomes.

What best describes afterload?

back pressure exerted by arterial blood Correct Afterload refers to the back pressure exerted by arterial blood, or the pressure that must be overcome for the ventricles to eject blood.

Where does primary active transport of sodium occur in proximal convoluted tubule cells?

basolateral membrane

Lymphedema may be treated by all EXCEPT which of the following?

bed rest to allow enhanced blood flow and therefore healing of the affected area Correct Lymphedema is swelling that occurs when normal lymphatic drainage is blocked. If a person remains stationary, it decreases skeletal muscle contraction, which facilitates lymph drainage.

Which of the following inhibits salivation? relaxing after a meal being stressed or frightened the sight or smell of food ingestion of spicy foods

being stressed or frightened Correct The sympathetic division of the autonomic nervous system is associated with fight-or-flight responses. Stimulation of the salivary glands by the SNS tends to inhibit flow, as when you are scared or nervous and your mouth "goes dry."

The most important buffer system of extracellular fluid, such as plasma, is the __________ buffer system. bicarbonate protein hemoglobin phosphate

bicarbonate

The pH of the ECF is maintained in homeostatic balance by which chemical buffer system? bicarbonate phosphate lipid protein

bicarbonate

The secretion in the large intestine consists of which of the following? intrinsic factor digestive enzymes such as CCK and secretin bile bicarbonate- and potassium-rich mucus

bicarbonate- and potassium-rich mucus Correct Yes, an alkaline mucus secretion of bicarbonate and potassium protects the large intestinal wall from acids produced by resident bacteria.

The rate of kidney filtrate formation would normally be dependent upon all of the following factors except __________. -renal artery/arteriole diameters -blood calcium level -filtration membrane integrity -systemic blood pressure

blood calcium level

Varicose veins seen in the superficial veins of the legs are unsightly and are often treated by surgical removal. However, even without these veins being present, the return of all blood toward the heart from the legs is not diminished. This is primarily because ______.

blood can still return via the deep veins Correct The leg has a complex and highly branched venous system. The superficial and deep veins anastamose with each other at many points along their length.

Which of the following substances is not normally found in filtrate? -nitrogenous waste particles, such as urea -water and small solutes -blood cells and large particles -ions, such as sodium and potassium

blood cells and large particles Correct Yes, both blood cells and large particles, such as proteins, are not allowed to filter through a healthy glomerular membrane.

Filtrate in a typical healthy nephron will NEVER include______________. blood cells and proteins amino acid fatty acids glucose K+ and Na+ ions

blood cells and proteins Correct Proteins and blood cells are too large to pass through the filtration membrane between the blood and the glomerular capsule.

In the capillaries, hydrostatic pressure (HP) is exerted by __________.

blood pressure Correct Yes, blood pressure is the driving force for filtration

If the osmotic pressure of the blood were increased above normal levels, which of the following volumes would also increase? blood volume total body water intracellular fluid volume interstitial fluid volume

blood volume Correct The higher osmotic pressure in the blood would draw water from the interstitial fluid, resulting in a higher blood volume and increased blood hydrostatic pressure.

Why is the osmolarity of medullary fluid in the kidney almost four times higher than the osmolarity of plasma? a)The nephron loop acts as a countercurrent multiplier and contributes solutes to the interstitial fluid. b)Urea is recycled from the collecting duct and is transported to the interstitial fluid. c)The medullary cells in the kidney synthesize solutes to establish the high osmolarity. d)Both a and b occur.

both a and b occur

If testosterone were ineffective during development of a male's brain, what might be the outcome? a)The brain would develop without any noticeable difference from other males. b)The brain might have a female orientation. c)Gender identity may be ambiguous. d)Both b and c are possible.

both b and c are possible

Which of the following pathways acts as a source for the electrons carried by the NADH and FADH2 molecules shown in this figure? -Glycolysis only -the citric acid cycle only -both glycolysis and the citric acid cycle -neither glycolysis nor the citric acid cycle

both glycolysis and the citric acid cycle Correct High-energy electrons derived from the oxidation of food molecules during glycolysis and the citric acid cycle are transferred to the electron transport chain by the coenzymes NAD+ and FAD.

Which of the following structures is/are formed from ectodermal tissues? gonads cartilage brain bones muscles right answer feedback:

brain right answer feedback:The ectoderm develops into many neural structures, including the central nervous system, as well as several integumentary structures.

A patient with a restrictive lung disease such as tuberculosis is likely to see an increase in his or her __________. -vital capacity -functional residual capacity -breathing rate -residual volume

breathing rate Correct Restrictive lung diseases decrease vital capacity, total lung capacity, functional residual capacity, and residual volume. To provide adequate ventilation, the alveolar ventilation rate must increase.

Where are the enzymes for digestion of disaccharides and small polypeptides located? secretions from the pancreas stomach brush border saliva

brush border

How is Na+ reabsorbed? -by osmosis -by active transport using ATP -by receptor-mediated endocytosis -by diffusion -by facilitated diffusion

by active transport using ATP

How are vitamins A, D, E, and K absorbed by the body? by being incorporated into micelles by direct simple diffusion in the jejunum by binding to intrinsic factor to trigger receptor-mediated endocytosis by active transport of vitamin pumps in the intestinal epithelia

by being incorporated micelles Correct Fat-soluble vitamins are incorporated into micelles when fats are present. Without dietary fats available, these vitamins are not absorbed.

How are nutrient molecules such as glucose and amino acids reabsorbed through the apical surfaces of the tubule epithelia?. by simple diffusion by osmosis by primary active transport by secondary active transport

by secondary active transport

If a virus attacks a cell, which type of immunity would be activated?

cell-mediated immunity (cellular immunity) Correct Yes, cell-mediated immunity involves an intracellular pathogen such as a virus.

The sperm's acrosome __________. provides motility contains enzymes provides the blood testis barrier has a high density of mitochondria contains nutrients

contains enzymes right answer feedback:The acrosome contains enzymes that allow the sperm to penetrate the egg.

What vessels sustain a drop in pressure from approximately 35 mm Hg to around 17 mm Hg?

capillaries Correct 35 mm Hg and 17 mm Hg are typical entering and exiting pressures for a capillary bed. These pressures strongly affect the exchange of fluids across the capillary wall.

The net hydrostatic pressure (HP) is the hydrostatic pressure in the __________ minus hydrostatic pressure in the __________.

capillary; interstitial fluid Correct Yes, the capillary hydrostatic pressure (HPC; caused by blood pressure) is much higher than the interstitial hydrostatic pressure (HPI). The interstitial fluid is forced out of the capillaries.

Which type of nutrient functions primarily as an energy source for cells? -carbohydrates -vitamins -minerals -oxygen

carbohydrates Correct Carbohydrates are needed in relatively large amounts to supply glucose, the main energy source for body cells.

Where are the sensors for the arterial baroreceptor reflex located?

carotid sinus and aortic arch Correct Yes, the sensors are the arterial baroreceptors themselves, located at the carotid sinus and aortic arch. They detect changes in blood pressure by the degree of stretch on the blood vessel.

Within the ovary, progesterone is produced by the __________. secondary follicles tertiary follicles corpus albicans primary follicles corpus luteum

corpus luteum right answer feedback:Within the ovary, progesterone is produced by the corpus luteum.

HCl secretions convert pepsinogen to the active hormone pepsin. What cells in the gastric pits produce pepsinogen? -parietal -G cells -paracrine (also known as enteroendocrine cells) -chief

chief Correct Yes, chief cells produce pepsinogen

What is the most common bacterial sexually transmitted disease in the United States? genital warts chlamydia syphilis vaginitis gonorrhea

chlamydia right answer feedback:Chlamydia is the most common bacterial sexually transmitted disease in the United States.

The disc-shaped placenta is formed from the __________. yolk sac chronic villi and the decidua capsularis inner cell mass chronic villi and the decidua basalis

chronic villi and the decidua basalis The chronic villi and the decidua basalis form the placenta. The inner cell mass is not involved in the formation of the placenta. The chronic villi and the decidua basalis form the placenta. The decidua capsularis is not involved in the formation of the placenta. The chronic villi and the decidua basalis form the placenta. The yolk sac is not involved in the formation of the placenta. The chronic villi and the decidua basalis form the placenta.

In females, the _______ is (are) the homologous structure to the glans penis in males.

clitoris

The beginning of the QRS complex of the electrocardiogram (ECG) immediately precedes which of the following events?

closing of the atrioventricular valves Correct The QRS complex is a recording of ventricular depolarization. This depolarization begins just prior to the ventricular contraction it initiates. As the ventricles contract, the increase in ventricular pressure closes the atrioventricular valves, beginning isovolumetric contraction. As ventricular pressure increases above the corresponding blood vessel, only then do the semilunar valves open, initiating ventricular ejection.

The first heart sound (the "lub" of the "lub-dup") is caused by __________

closure of the atrioventricular valves Correct The first heart sound (the "lub" of the "lub-dup") is generated by the closure of the atrioventricular valves.

The ABO blood group inheritance is an example of _______.

codominance and multiple-allele inheritance

Which mechanism(s) of antibody action result(s) in cell lysis?

complement activation Correct Complement has the potential to bind antigen by itself (alternative pathway), but it may require an antibody to bind to the antigen first (classical pathway).

The energy needed for secondary active transport is provided by the __________. filtration membrane concentration gradient established by Na+ renal capillary hydrostatic pressure cleavage of ATP

concentration gradient established by Na+ feedback:The concentration gradient established by Na+ provides the energy needed for secondary active transport.

What type of T cell can directly attack and kill other cells, such as virus-infected cells?

cytotoxic T (TC) cells Correct Cytotoxic T (TC) cells can distinguish between self cells and foreign cells. If a cytotoxic T cell recognizes its own antigen on a virus-infected cell, the T cell will destroy it.

As a result of hyperventilation, what will happen to the partial pressures of CO2 (pCO2) and pH? -decreased pCO2 and decreased pH -increased pCO2 and decreased pH -increased pCO2 and increased pH -decreased pCO2 and increased pH

decrease pCO2 and increased pH Correct Yes, pCO2 would decrease and pH would increase. As CO2 is blown off, H+ would decrease, thus increasing pH.

Which of the following would cause vasodilation of arterioles?

decreased activity of the sympathetic nervous system Correct Yes, a decrease in the activity of the sympathetic nervous system would result in decreased activity of vasomotor fibers, resulting in vasodilation.

The functional layer of the endometrium is shed in response to __________. increased progesterone levels increased estrogen levels decreased progesterone levels decreased estrogen levels

decreased progesterone levels feedback:The endometrium has two layers: the stratum functionalis and the stratum basalis. The stratum functionalis (or functional layer of the endometrium) is shed during menstruation in response to declining levels of progesterone. During the proliferative phase of the uterine cycle, the stratum basalis will regenerate the stratum functionalis.

When atrial natriuretic peptide is released by the heart because of increased blood pressure, it causes __________. increased renin secretion vasoconstriction decreased sodium and water reabsorption increased antidiuretic-hormone secretion

decreased sodium and water reabsorption right answer feedback:ANP reduces blood pressure and blood volume by inhibiting nearly all events that promote vasoconstriction, sodium retention, and water retention.

Which of the following is NOT an effect of testosterone? stimulates maturation of male sex organs maintains male libido maintains male secondary sex characteristics decreases sperm production

decreases sperm production Testosterone stimulates sperm cell production, promotes the development of male secondary sex characteristics, maintains male libido, and stimulates maturation of male sex organs.

Which of the following would decrease total peripheral resistance to blood flow?

decreasing the hematocrit Correct Decreasing the hematocrit (fraction of whole blood consisting of blood cells) would lead to decreased blood viscosity, and thus would decrease resistance.

Which of the following would cause a DECREASE in cardiac output (CO)?

decreasing thyroid function (thyroxine) Correct Thyroxine increases basal metabolic rate, respiratory rate, and both heart rate and contractility.

Which of the following cells engulf antigens by phagocytosis and present fragments of them on their own surfaces for recognition?

dendritic cells Correct Dendritic cells are antigen-presenting cells that engulf antigens and then present fragments of them to their own surfaces, where T cells can recognize them.

Which muscles, when contracted, would increase the volume of air in the thoracic cavity? -internal intercostals and external oblique -diaphragm and internal intercostals -diaphragm and external intercostals

diaphragm and external intercostals Correct Yes, contraction of both the diaphragm (the diaphragm flattens) and the external intercostals (pulls the ribs up and out) will increase the volume of the thoracic cavity. This will cause air to move into the lungs (inspiration).

Mr. V has also had a recent echocardiogram and blood work. With tricuspid stenosis, what changes do you expect to see in the echocardiogram report? -dilated right atrium because blood is not flowing down into the right ventricle fast enough -enlarged left ventricle because blood is not flowing out into the body fast enough -decreased ejection fraction because blood cannot flow out into the aorta fast enough -dilated left atrium because blood is not flowing down into the left ventricle fast enough

dilated right atrium because blood is not flowing down into the right ventricle fast enough Correct Sure enough, when you check Mr. V's echocardiogram, you see a dilated right atrium and a slightly dilated superior vena cava. Does this help you to explain Mr. V's other signs and symptoms?

Which sequence shows the correct order of the stages of labor? dilation, expulsion, placental expulsion, dilation, placental dilation, placental, expulsion placental, dilation, expulsion expulsion, placental, dilation

dilation, expulsion, placental right answer feedback:The stages of labor are dilation, expulsion, and placental. The dilation stage is the longest period, lasting up to 12 hours on average.

Upon reaching what point in the nephron is reabsorption (1) dependent upon the body's needs at the time and (2) regulated by hormones? -proximal convoluted tubule -descending limb of the loop of Henle -distal convoluted tubule -ascending limb of the loop of Henle

distal convoluted tubule Correct Distal convoluted tubule (DCT) reabsorption only occurs under the influence of hormonal regulation.

In what part of the renal tubule does parathyroid hormone (PTH) promote the reabsorption of calcium ions? -proximal convoluted tubule (PCT) -descending limb of the nephron loop -distal convoluted tubule (DCT) -collecting duct

distal convoluted tubule (DCT) Correct Parathyroid hormone (PTH) promotes the reabsorption of calcium ions in the distal convoluted tubule (DCT).

When the diet lacks fiber and the volume of residues in the colon is small, the colon narrows and its contractions become more powerful, increasing the pressure on its walls. This promotes formation of __________. gallstones cystic fibrosis irritable bowel syndrome (IBS) diverticula

diverticula Correct When the diet lacks fiber and the volume of residues in the colon is small, the colon narrows and its contractions become more powerful, increasing the pressure on its walls. This promotes formation of diverticula, small herniations of the mucosa through the colon walls. This condition, called diverticulosis, most commonly occurs in the sigmoid colon, and affects over half of people over age 70. In 4-10% of cases, diverticulosis progresses to diverticulitis, in which the diverticula become inflamed and may rupture, leaking into the peritoneal cavity, which can be life-threatening.

An allele that can mask another allele is said to be __________. heterozygous homozygous recessive dominant

dominant A dominant allele cannot be masked. Its trait will appear regardless of its partner allele on the other matching chromosome.

When does the total number of chromosomes get reduced from 46 pairs to 23 individual chromosomes? during mitosis during metaphase II during meiosis I during interphase

during meiosis I Correct Chromosome pairs are separated during anaphase I, and this first division is called the "reduction division."

Which of the following would NOT be expected to occur as a result of dehydration? -edema -reduced urine flow -hypovolemic shock -shrinkage of body cells

edema Correct Edema is tissue swelling that results from an atypical accumulation of fluid in the interstitial space. During dehydration, this fluid would be reduced.

What serves as the direct energy source for the proton pumps in oxidative phosphorylation? -oxidation of oxygen -electrons passing from transfer molecules to oxygen -the Krebs cycle -glycolysis

electrons from transfer molecules to oxygen Correct As electrons move through the electron transport chain from transfer molecules to oxygen, energy is released and used to form an H+ (proton) gradient across the membrane.

During development, the conceptus is called a (an) _______ from fertilization through week 8, and a (an) _______ from week 9 through birth.

embryo; fetus

The _______ of the uterus receives the embryo and provides nourishment until the placenta is formed.

endometrium

Parathyroid hormone __________. results in a lower blood calcium level activates osteoblasts enhances release of calcium from bone is released when aldosterone levels climb targets the thyroid gland

enhances release of calcium from bone right answer feedback:PTH activates osteoclasts, which break down the bone matrix, releasing calcium.

Which chemical activates the transformation of trypsinogen to trypsin? chymotrypsin amylase carboxypeptidase enteropeptidase

enteropeptidase Correct Enteropeptidase is a protein produced by and bound to the membranes of intestinal cells. Contact with this protein converts trypsinogen to trypsin, the active form of the enzyme.

In females, one of the X chromosomes is inactivated by __________. epigenetic markers the number of dominant genes present on each X chromosome environmental factors enzymes coded for by mitochondrial DNA

epigenetic markers In females, epigenetic markers inactivate one of the X chromosomes

What structure prevents food and liquids from entering the trachea? -glottis -epiglottis -laryngeal prominence -vestibular folds

epiglottis Correct During swallowing, the larynx is pulled superiorly and the epiglottis tips to cover the laryngeal inlet. Because this action keeps food out of the lower respiratory passages, the epiglottis has been called the guardian of the airways.

During an allergic reaction, which of the following would aid respiration? -acetylcholine (ACh) -an increase in the parasympathetic nervous system -epinephrine -histamine

epinephrine Correct Yes, during an allergic reaction, there is increased resistance in the bronchioles and epinephrine dilates the bronchioles, thus making it easier to breathe. Epinephrine is released from the adrenal gland during stressful situations. People with severe allergies carry an EpiPen in case the allergic reaction produces anaphylaxis

In terms of the male sexual response, parasympathetic stimulation causes __________. ejaculation fertilization the penis to become flaccid spermiogenesis erection of the penis

erection of the penis right answer feedback:Parasympathetic stimulation causes NO to be released, causing penile arterioles to dilate, allowing the erectile bodies to fill with blood.

Which hormone is responsible for the development of secondary sex characteristics found in women? estrogen prolactin LH progesterone FSH

esetrogen

Which of the following INCREASES stroke volume?

exercise Correct Exercise increases venous return, which would in turn increase the amount of blood in the presystolic ventricle (preload).

Which of the following processes is NOT properly matched with its description? -external respiration: CO2 diffuses into the blood stream -internal respiration: O2 diffuses out of the blood -pulmonary ventilation: movement of gases into and out of the lungs -transport of respiratory gases: O2 is bound to hemoglobin

external respiration: CO2 diffuses into the blood stream Correct CO2 diffuses into the blood as it passes through the systemic capillaries of the tissues; this is internal respiration.

The movement of H+ through the ATP synthase is best described as an example of __________. active transport hydrolysis simple diffusion facilitated diffusion

facilitated diffusion Correct H+ cannot diffuse directly through the semipermeable phospholipid bilayer. The ATP synthase contains a channel that allows the diffusion of H+ down its concentration gradient. Diffusion of H+ releases energy that is used by the ATP synthase to phosphorylate ADP, producing ATP.

Water can leave the ascending limb of the nephron loop. True False

false

When threshold is reached at the SA node (an autorhythmic cell), what channels open causing further depolarization of the membrane?

fast calcium Correct Yes, unlike nerve cells or cardiac muscle cells, fast calcium channels are responsible for the depolarization phase of the autorhythmic cell action potential. When the fast calcium channels open, calcium rushes into the cell making it less negative (or more positive).

The breakdown products of which of the following are absorbed into lacteals? fats carbohydrates proteins

fats

Salivation is controlled almost entirely by the nervous system. Which of the following stimuli would inhibit salivation? -the thought of food -a lemon -nausea -fear

fear Correct Yes, fear, sleep, fatigue, and dehydration all inhibit salivation.

Which of the following is/are NOT associated with primary nocturnal enuresis? unusually sound sleep inadequate nocturnal ADH production a small bladder capacity females over the age of 60 years

females over the age of 60 years Primary nocturnal enuresis usually occurs in children under the age of 6 years.

As the kidneys filter blood, it is necessary for both fluid and dissolved chemicals in the plasma to be rapidly removed without the removal of larger proteins or cells. Which capillary would be best suited for the filtration?

fenestrated capillaries Correct Fenestrated capillaries are found wherever active filtrate formation occurs. Fenestrations are Swiss cheese-like holes that tunnel through endothelial cells that allow for the selective passage of smaller solute molecules.

Suppose that a patient has a failing liver and is unable to make normal levels of albumin for the blood plasma. If all other factors remain the same, what would be the effect on the amount of filtrate produced? -filtrate production would decrease -filtrate production would increase -filtrate production would remain the same

filtrate production would increase Correct If the liver were making insufficient levels of albumin, which is the most important contributor to the blood's osmotic pressure, the blood's osmotic pressure would drop, leading to a rise in filtrate production.

All of the following functions are carried out in the renal tubules EXCEPT __________. secretion filtration reabsorption formation of urine

filtration Filtration is carried out in the glomerulus, not in the renal tubules.

Meiosis results in __________. two identical diploid cells two nonidentical haploid cells four identical diploid cells four nonidentical diploid cells four nonidentical haploid cells

foru nonidentical haploid cells Correct: Meiosis is the processes by which genetically unique haploid gametes are formed.

Meiosis starts with a single diploid cell and produces -eight haploid cells -two haploid cells -four diploid cells -two diploid cells -four haploid cells

four haploid cells Correct Meiosis produces four haploid cells.

Which of the following is a likely result of meiosis? -two eggs, each with 23 chromosomes -two eggs, each with 46 chromosomes -four sperm, each with 23 chromosomes -four sperm, each with 46 chromosomes

four sperm, each with 23 chromosomes Correct In males, meiosis produces four sperm, each with 23 chromosomes.

In Baby A, the atrial septal defect did not close at birth. Blood is flowing through her defect in what direction? -from the right atrium into the left atrium -There will be no net flow across the defect. -from the left atrium into the right atrium -from the left atrium into the right ventricle

from the left atrium into the right across Correct Good. Baby A's atrial septal defect did not close properly on her first breath, so now blood is flowing from her left atrium into her right atrium. But that is not what brought her in to the doctor, is it? She is here because of respiratory infections and trouble breathing.

Mitochondrial DNA is inherited _______.

from the mother

Urine from a person with uncontrolled diabetes mellitus has a(n) _______ due to the presence of _______.

fruity odor; acetone

The uterine layer shed with each monthly cycle is the __________. perimetrium basal layer of endometrium functional layer of endometrium tunica albuginea myometrium

functional layer of endometrium right answer feedback:The uterine layer shed with each monthly cycle is the functional layer of endometrium. This innermost layer is shed when hormone levels drop at the end of the monthly cycle.

Action potentials generated by the autorhythmic cells spread to the contractile cells through what structures in the membrane?

gap junctions Correct Yes, action potentials generated by the autorhythmic cells spread waves of depolarization to contractile cells through gap junctions. If the depolarization causes the contractile cells to reach threshold, they will in turn generate an action potential.

GFR regulation mechanisms primarily affect which of the following? -glomerular hydrostatic pressure (HPg) -capsular osmotic pressure (OPc) -blood osmotic pressure (OPg) -capsular hydrostatic pressure (HPc)

glomerular hydrostatic pressure (HPg) Correct Much like other capillaries in the body, hydrostatic pressure within the glomerular capillaries produces net outward movement of fluid. Unique to glomerular capillaries, HPg is consistently higher than other capillaries (~55 mm Hg), which ensures the one-way movement of fluid and solutes out of the glomerulus under normal conditions.

Which capillary bed produces filtrate?

glomerulus Correct The glomerulus (capillary bed) in the glomerular capsule produces filtrate due to the high fluid pressure within the vessels.

What happens when glucose is NOT immediately available for glycolysis? -lactic acid formation -gluconeogenesis -lipogenesis -glycogenesis

gluconeogenesis Correct Gluconeogenesis refers to the process of converting glycerol or amino acids into glucose so that glycolysis can continue.

Which of the following is NOT an electrolyte? -potassium -glucose -chloride -sodium

glucose

Which of these should not normally appear in urine? -urea -sodium -creatine -glucose

glucose Correct Glucose would normally be completely reabsorbed during urine formation and should not be in a urine sample.

Which of the following materials is NOT reabsorbed in the nephron loop? glucose chloride ions sodium ions water

glucose Correct Virtually all of the glucose in the filtrate is reabsorbed in the proximal convoluted tubule before reaching the nephron loop.

Part complete If a person consumes excess carbohydrates and sugars, those excess sugars can go through an anabolic process that produces __________. -glucose molecules -glycogen molecules -amino acids -proteins

glycogen molecules Correct Glycogen is a polysaccharide that can be stored in liver and muscle cells.

Which of the following processes takes place in the cytosol of a eukaryotic cell? -citric acid cycle -acetyl CoA formation -electron transport chain -glycolysis -ATP production by ATP synthase

glycolysis Correct Glycolysis, the breakdown of glucose into two molecules of pyruvic acid, takes place in the cytosol, outside the mitochondria.

Which step of cellular metabolism breaks down a glucose molecule to produce two pyruvic acid molecules? -glycolysis -krebs cycle -oxidative phosphorylation -formation of acetyl CoA

glycolysis Correct The catabolism of glucose provides the pyruvic acid molecules that, once converted into acetyl CoA, can fuel the Krebs cycle.

Select the correct sequence of steps as energy is extracted from glucose during cellular respiration. citric acid cycle → electron transport chain → glycolysis → acetyl CoA glycolysis → acetyl CoA → citric acid cycle → electron transport chain glycolysis → citric acid cycle → acetyl CoA → electron transport chain acetyl CoA → citric acid cycle → electron transport chain → glycolysis electron transport chain → citric acid cycle → glycolysis → acetyl CoA

glycolysis → acetyl CoA → citric acid cycle → electron transport chain Correct Glycolysis produces pyruvic acid, which enters the mitochondrion. There, it is converted to acetyl CoA, which enters the citric acid cycle. Electron carriers bring electrons from the first three steps to the electron transport chain, and ATP is made.

What structure(s) is/are part of the juxtaglomerular complex and act(s) as a mechanoreceptor, sensing blood pressure in the afferent arteriole? podocytes granular cells extraglomerular mesangial cells macula densa

granular cells right answer feedback:As enlarged smooth muscle cells in the arteriolar walls, granular cells release renin in response to a drop in mean arterial blood pressure.

Which hormone is required to maintain the corpus luteum and prevent menstruation? -estrogen -progesterone -human placental lactogen -hCG

hCG Correct Human chorionic gonadotropin (hCG) secreted by the placenta is required to maintain the corpus luteum and prevent menstruation.

Which lymphocytes act as the bridge between the cellular and humoral responses?

helper T cells Correct Helper T cells are lymphocytes that organize the cellular and humoral immune response branches of the immune system.

Which of the following are NOT appropriately matched?

helper T cells: destroy infected body cells Correct Helper T cells are lymphocytes that manage or orchestrate an immune response, but they do not directly kill cells.

During prophase I of meiosis, -chromosome pairs are positioned in the middle of the cell. -there are two daughter cells, each with 23 chromosomes. -there are four haploid daughter cells. -the homologous chromosomes separate and move towards opposite poles. -homologous chromosomes stick together in pairs. Submit

homologous chromosomes stick together in pairs. Correct Homologous chromosomes stick together in pairs during prophase I.

What type of immunity can be transferred by bodily fluids from one person to another, thus conferring immunity to the recipient?

humoral immunity Correct Yes, humoral immunity involves antibodies that can be transferred from one person to another.

Acidosis results in increasing levels of what ion? hydrogen calcium phosphorus sodium chloride

hydrogen right answer feedback:Acidosis refers to high H+ concentration.

This type of chemical reaction typically occurs as nutrients are digested in the body. anabolic hydrolysis endergonic dehydration synthesis

hydrolysis Correct Hydrolysis refers to the enzymatic breakdown of large polymers into monomers while adding the parts of a water molecule to the broken bonds.

Renal ptosis may lead to __________. -degeneration of the perirenal fat -renal calculus formation -bleeding in the kidney tissue -hydronephrosis due to urine backup

hydronephrosis due to urine backup Renal ptosis is the dropping of a kidney to a lower position, which may cause a ureter to become kinked, causing urine to back up (hydronephrosis).

What is the chief force pushing water and solutes out of the blood and across the filtration membrane of the glomerulus? -colloid osmotic pressure in glomerular capillaries (OP gc) -hydrostatic pressure in glomerular capillaries (HP gc) -hydrostatic pressure in the capsular space (HP cs) -colloid osmotic pressure in the capsular space

hydrostatic pressure in glomerular capillaries (HP gc) Correct While diffusion rates are affected more by osmotic differences and concentration gradients, hydrostatic pressure in glomerular capillaries (HPgc) is the chief method of forcing water and solutes by filtration.

What is the primary driving force (pressure) that produces glomerular filtration? -colloid osmotic pressure of blood -gravity -hydrostatic pressure of blood (blood pressure)

hydrostatic pressure of blood (blood pressure) Correct Yes, the hydrostatic pressure of blood forces fluid out of the glomerular capillaries.

The condition in which sodium levels are too low is referred to as __________. Cushing's syndrome hyponatremia hypokalemia hypernatremia aldosteronism

hyponatremia

Which of the following would NOT be expected to lead to edema? incompetent venous valves hyponatremia inflammation hypoproteinemia

hyponatremia Correct Hyponatremia is low levels of sodium in the blood. This can lead to dehydration, not edema.

All of the following factors would stimulate the hypothalamic thirst center EXCEPT __________. increased angiotensin II secretion decreased saliva production hypotonic extracellular fluid a decline in blood volume right answer feedback:

hypotonic extracellular fluid

Which of the following is associated with a swelling of cells? -edema -hypoproteinemia -hypotonic hydration -dehydration

hypotonic hydration

What type of water imbalance increases the amount of fluid in both cells and tissue? -hypotonic hydration -inflammation -edema -dehydration

hypotonic hydration Correct Hypotonic hydration occurs when extracellular fluid becomes so hypotonic (dilute) that water is pulled into the cells. Excess water is present in cells and extracellular compartments.

Which condition would cause a drop in pH? hypokalemia hyperventilation hypoventilation hypernatremia

hypoventilation right answer feedback:Hypoventilation causes a drop in pH. Breathing slowly increases PCO2.

Which of the following are NOT correctly matched?

immediate hypersensitivity: allergic contact dermatitis Correct Allergic contact dermatitis is a delayed hypersensitivity response; it is not characterized by an immediate response. Return to Assignment

In what form do fats first enter the bloodstream? in the form of chylomicrons in the form of micelles in the form of fatty acids and glycerol in the form of glycerol

in the form of chylomicrons Correct Within enterocytes, fats are combined with phospholipids and cholesterol, and coated with proteins to form water-soluble lipoprotein droplets called chylomicrons. Chylomicrons are secreted first into lymphatic vessels where they are eventually emptied into the bloodstream.

In what part of the nephron is plasma filtered? in the renal corpuscle in the proximal convoluted tubule in the distal convoluted tubule in the collecting duct

in the renal corpuscle

Which teeth in the permanent dentition are best suited for cutting or nipping off pieces of food? canines premolars (bicuspids) incisors molars

incisors Correct The location and shape of the incisors make them ideally suited for cutting and nipping food.

When a heterozygous individual expresses a phenotype intermediate between the dominant and recessive phenotype, this is called _______.

incomplete dominance

If blood pressure is increased at the arterial baroreceptors, what would happen with the activity level of the parasympathetic nervous system (PNS) and sympathetic nervous system (SNS)?

increased PNS activity and decreased SNS activity Correct Yes, the PNS activity would increase and the SNS activity would decrease in an attempt to lower blood pressure.

What is a likely result of rapidly consuming excessive quantities of electrolyte? -decreased sensation of thirst -increased aldosterone production -decreased blood pressure -increased blood pressure

increased blood pressure Correct Increased electrolyte levels increase ECF osmolality, which then triggers an increase in blood volume via osmosis and, therefore, an increase in blood pressure.

What might be a sign that the renin-angiotensin-aldosterone mechanism is NOT producing enough aldosterone? increased blood volume increased excretion of water and NaClNaCl in urine increased blood pressure increased antidiuretic hormone (ADH) production Submit

increased excretion of water and NaCl in urine Correct Aldosterone is primarily responsible for triggering reabsorption of water and NaClNaCl.

If the efferent arteriole constricts while the afferent arteriole remains unchanged, the glomerular filtration rate __________. does not change decreases increases cannot be determined

increases If the efferent arteriole constricts while the afferent arteriole remains unchanged, the glomerular filtration rate increases. This will increase glomerular hydrostatic pressure.

In response to warmth, the scrotum __________. increases its surface area pulls the testes closer to the body becomes heavily wrinkled decreases blood flow to the testes

increases its surface area Viable sperm cannot be produced in abundance at core body temperature (37°C). The superficial location of the scrotum, which holds the testes, provides a temperature about 3°C lower than core temperature, allowing viable sperm to be produced.The scrotum is affected by temperature changes. When it is cold, the scrotum reduces its surface area by becoming heavily wrinkled; it also shortens to pull the testes closer to the body wall. In warm temperatures, the scrotum becomes flaccid and loose to increase its surface area, and hangs lower to keep the testes away from the warmer body trunk.

Chyme entering the large intestine normally consists of __________. excess nutrients that the body does not need large proteins, nucleic acids, and other molecules not generally utilized as an energy source indigestible fiber, enteric bacteria, and water all fat-soluble molecules that are unable to cross the small intestine epithelium

indigestible fiber, enteric bacteria, and water Correct The small intestine absorbs virtually all nutrients from chyme before it reaches the large intestine, leaving only indigestible wastes, water, and millions of bacteria.

Which of the following does NOT serve as a source of acids in the body? aerobic breakdown of glucose CO2 in the blood fat metabolism ingesting of bicarbonate

ingesting of bicarbonate

Tears and mucus membranes would be a part of which defense system?

innate external defenses Correct Yes, innate external defenses (surface barriers) are the first line of defense and include tears, mucus membranes, and the skin.

Phagocytotic cells such as macrophages identify a variety of enemies by recognizing markers unique to pathogens. They would be classified as which type of defense system?

innate internal defenses Correct Yes, the macrophage recognizes many molecules found on pathogens that are not present on normal body cells.

Which lung volume tends to be the largest in healthy male and female adults? -tidal volume -inspiratory reserve volume -expiratory reserve volume -residual volume

inspiratory reserve volume Correct In most healthy adults, inspiratory reserve volume is greater than tidal volume, expiratory reserve volume, and residual volume.

What does a high concentration of NaCl in the renal tubule at the juxtaglomerular apparatus (JGA) most likely indicate? excessive NaCl reabsorption due to high GFR excessive NaCl reabsorption due to low GFR insufficient NaCl reabsorption due to low GFR insufficient NaCl reabsorption due to high GFR

insufficient NaCl reabsorption due to high GFR Correct After glomerular filtration, NaCl is actively reabsorbed at many locations along the renal tubule. If the filtrate is moving through the tubule quickly, less reabsorption is possible, so more NaCl gets left behind. This means that at the JGA, the NaCl concentration within the filtrate will be high.

Which of the following is the standard substance used to measure the GFR? protein glucose inulin drug metabolites

insulin Inulin is the standard substance used to measure the GFR because it is freely filtrated and not reabsorbed or secreted by the kidney. Creatinine is often used to give a "quick and dirty" estimate of GFR because it does not need to be intravenously infused into the patient as does inulin.

What protein can be released by infected cells to help protect cells that have not yet been infected?

interferon Correct Interferon is a small protein that can stimulate nearby healthy cells to defend themselves against viral infection.

A cell preparing to undergo meiosis duplicates its chromosomes during -meiosis II -prophase I -interphase -metaphase I -anaphase I

interphase Correct Chromosomes are duplicated during interphase.

__________ cells, located between seminiferous tubules, produce testosterone. Follicle Myometrial Thalamic Interstitial Sustentacular

interstitial right answer feedback:Interstitial, or Leydig cells, produce testosterone.

In males, which of the following includes the correct atrget and result of LH stimulation -sustentocytes: increased testosterone -sustentocytes: increased spermatogenesis -interstitial cells: increased androgen-binding protein (ABP) secretions -interstitial cells: increased androgen secretions

interstitial cells: increased androgen secretions Correct In males, LH targets interstitial cells. As a result of LH stimulation of interstitial cells, the levels of androgens, such as testosterone, increase.

Edema is the accumulation of fluid in the __________. plasma organelles interstitial space intracellular space

interstitial space

During which phase in the control of the digestive system would bicarbonate and bile be stimulated? intestinal phase gastric phase cephalic phase

intestinal phase Correct Yes, food in the intestines initiates a reflex that stimulates secretions of bicarbonate, digestive enzymes, and bile.

The largest percentage of body water is located in what compartment? intracellular fluid extracellular fluid interstitial fluid blood plasma

intracellular fluid

Which pressure is the result of the natural tendency of the lungs to decrease their size (because of elasticity) and the opposing tendency of the thoracic wall to pull outward and enlarge the lungs? -intrapulmonary pressure -atmospheric pressure -intrapleural pressure

intrapleural pressure Correct Yes, the lungs tend to decrease their size while the chest wall tends to pull the thorax outward. This makes the intrapleural pressure more negative than the other two pressures (described as subatmospheric), thus keeping the lungs inflated.

In pneumothorax, the lung collapses because ______. -intrapleural pressure is lower than trans-pulmonary pressure -intrapleural pressure is equal to intrapulmonary pressure -intrapulmonary pressure is lower than trans -pulmonary pressure intrapulmonary pressure is higher than atmospheric pressure

intrapleural pressure is equal to intrapulmonary pressure Correct Intrapleural pressure (Ppul) is the gas pressure within the pleural cavity, while intrapulmonary pressure (Pip) is the gas pressure within the alveoli. Normally Ppul is less than Pip to maintain lung expansion. If Ppul exceeds Pip, then the lungs collapse.

When given to a patient, which of the following substances would increase his or her urinary output? -intravenous saline -antidiuretic hormone -aldosterone -albumin

intravenous saline

Which of the following is produced in the stomach and contributes directly to the absorption of vitamin B12? Vasoactive intestinal peptide (VIP) hydrochloric acid intrinsic factor pepsinogen

intrinsic factor Correct Without the production of intrinsic factor by gastric parietal cells, the body would not be able to absorb vitamin B12 in the small intestine.

Pica, the desire to consume substances that are not normally considered food, such as chalk or clay, may be triggered by a deficiency of __________. calcium sodium water iron

iron

Hemoglobin -uses ATP to move oxygen from blood to body cells. has five subunits. -is the site of cellular respiration. -is found in blood plasma. -is a protein that can bind four molecules of oxygen.

is a protein that can bind four molecules of oxygen

What is the period during the cardiac cycle when the valves leading to and from the ventricles are completely closed and blood volume in the ventricles remains constant as the walls contract?

isovolumetric contraction phase Correct The prefix -iso means "equal," so during any isovolumetric stage the volume does not change.

Which of the following is NOT a physical characteristic of freshly voided urine in a healthy person? A given volume of urine has a greater specific gravity than the same volume of distilled water. It is slightly aromatic. It is clear and pale to deep yellow. It is slightly basic in pH.

it is slightly basic in pH

Which of the following statements about the urinary system is INCORRECT? It metabolizes vitamin D to its active form. It produces epinephrine. It produces renin, which helps regulate blood pressure. It produces erythropoietin, which stimulates red blood cell formation.

it produces epinephrine

The only organ(s) of the body that can remove excess nonvolatile fixed acids is/are the __________. sweat glands spleen kidney lungs liver

kidney right answer feedback:The only organ of the body that can remove excess nonvolatile fixed acids is the kidney. Fixed acids are also called metabolic acids.

Most water is excreted via the ______________. -kidneys -intestines -skin -lungs

kidneys

Which organs are most important for the long-term regulation of acid-base homeostasis? -kidneys -lungs -gastrointestinal tract organs -cardiovascular organs

kidneys Correct The kidneys regulate pH over long-term time periods.

Antidiuretic hormone (ADH) acts on the _________ to __________ water excretion. -kidneys; decrease -intestines; decrease -kidneys; increase -intestines; increase

kidneys; decrease

The major calyces are the _______. -pyramid-shaped structures in the renal medulla -large branches of the renal pelvis -functional units of the kidneys -expanded ends of nephrons -capsules surrounding each kidney

large branches of the renal pelvis

Which of the following helps to protect against tracheal obstruction. -carina of the trachea -trachealis muscle -larynx -tracheal cartilage

larynx Correct The larynx is superior to the trachea in the respiratory tract. The laryngeal opening (glottis) is covered by the epiglottis during swallowing, normally preventing ingested materials from passing into the trachea.

Failure in a particular structure of the heart tends to cause a backup of blood in the lungs, known as pulmonary congestive heart failure. Failure of which structure of the heart would lead to such a backup? -pulmonary semilunar valve -right atrium -left ventricle -right ventricle

left ventricle Correct Failure in the left ventricle can cause increased blood hydrostatic pressure in the lungs, causing fluid buildup in the alveoli.

When the concentration of ADH increases, __________. less urine is produced less water is reabsorbed by the nephron and collecting duct the specific gravity of the urine decreases more salt is secreted by the nephron blood volume decreases

less urine is produced

Angiotensinogen is constantly produced by the __________. -adrenal cortex -liver -kidney -atrial cells of the heart

liver The liver continuously secretes angiotensinogen.

If the transpulmonary pressure equals zero, what will happen to the lung? -lung volume will stay the same -lungs will inflate -lungs will collapse

lungs will collapse Correct Yes, the transpulmonary pressure creates the suction that keeps the lungs inflated. When room air enters the pleural space, transpulmonary pressure is zero and the lungs deflate - this is known as a pneumothorax.

Which of the following are MISMATCHED? luteal phase; characterized by decreased progesterone levels ovulation; release of a secondary oocyte ovarian cycle; monthly series of events associated with maturation of an egg follicular phase; development of a secondary oocyte within a dominant follicle right answer feedback:

luteal phase; characterized by decreased progesterone levels The ovarian cycle includes the monthly events that are associated with the maturation of an egg. The ovarian cycle includes the follicular phase, ovulation, and the luteal phase. During the follicular phase, a secondary oocyte develops within a dominant follicle. During ovulation, the secondary oocyte is released from the ovary. After ovulation, the ruptured follicle in the ovary collapses and the remaining granulosa cells are modified into an endocrine structure called the corpus luteum. The corpus luteum secretes progesterone and some estrogen.

Which hormone is most responsible for triggering ovulation? -progesterone -estrogen -follicle-stimulating hormone (FSH) -luteinizing hormone (LH)

luteinizing hormone (LH) Correct Estrogen levels increase during the follicular phase, which triggers a surge of LH. This surge of LH triggers ovulation.

The most important renal mechanism for regulating acid-base balance of the blood involves __________. maintaining phosphate balance maintaining CO2 balance maintaining HCO3 - balance maintaining water balance

maintaining HCO3 - balance Correct Excretion of bicarbonate is a long-term mechanism that the kidneys contribute to acid-base regulation.

The renal hilum lies on the __________ surface of the kidney. medial lateral superior inferior

medial The renal hilum lies on the medial surface of the kidney.

Which of these meiosis stages is functionally the most similar to mitosis? -meiosis I -meiosis II -meiosis I with II -neither meiosis I nor II

meiosis II Correct Meiosis II is fundamentally identical to mitosis. Prophase II, anaphase II, metaphase II, and telophase II have the same events as their corresponding stages in mitosis.

_______ is the stage of life in females when reproductive hormones decline and menstrual cycles become erratic.

menopause

During what phase of the female's uterine cycle is the uterine lining shed? -secretory of postovulatory, phase -preovulatory phase -menstrual phase -proliferative phase

menstrual phase Correct: The functional layer of the endometrium is shed during days 1 through 5 of the uterine cycle, a process called menses or menstruation.

A patient is admitted to the hospital with the following plasma values: pH = 7.2, pCO2 = 25 mmHg, and HCO3 −− = 18 mEq/L. What is the acid base imbalance? -respiratory acidosis with metabolic compensation -metabolic acidosis with no compensation -metabolic alkalosis with respiratory compensation -metabolic acidosis with respiratory compensation

metabolic acidosis with respiratory compensation Correct Yes, the low bicarbonate indicates this is a metabolic acidosis. Since CO2 is below the normal range, this indicates there is a respiratory compensation.

Starvation would cause which of the following acid-base conditions? Also, determine what type of compensation (metabolic or respiratory) there would be. -metabolic alkalosis with respiratory compensation -respiratory alkalosis with metabolic compensation -metabolic acidosis with respiratory compensation -respiratory acidosis with metabolic compensation Submit

metabolic acidosis with respiratory compensation Correct Yes, this is known as a ketosis (due to the breakdown of fat for metabolism), and the respiratory system would increase respiration to compensate for the excess hydrogen ions.

Diarrhea can lead to which acid/base disturbance? Assuming compensation, would it be a metabolic or respiratory compensation? -metabolic acidosis with respiratory compensation -respiratory alkalosis with metabolic compensation -metabolic alkalosis with respiratory compensation -metabolic alkalosis with respiratory compensation

metabolic acidosis with respiratory compensation Correct Yes, with the loss of bicarbonate ions, this is a metabolic acidosis. If compensated, it would be a respiratory compensation.

A patient is admitted to the hospital with the following plasma values: pH = 7.5, pCO2 = 45 mmHg, and HCO3 −− = 30 mEq/L. What is the acid-base imbalance? -respiratory alkalosis with metabolic compensation -respiratory acidosis with metabolic compensation -metabolic alkalosis with no compensation -metabolic alkalosis with respiratory compensation

metabolic alkalosis with no compensation Correct Yes, since HCO3¯ is 30 (above the normal range of 22 to 26 mEq/L), this is definitely a metabolic alkalosis. There is no respiratory compensation since CO2 is within the normal range.

The area of the sperm cell that contains many mitochondria is the __________. tail acrosome midpiece head

midpiece The head of the sperm contains the cell's nucleus. Atop the nucleus is a package of enzymes called the acrosome. The enzymes in the acrosome allow the sperm cell to penetrate the egg. The sperm midpiece contains many mitochondria spiraled tightly around the contractile filament of the tail. The main role of the mitochondria is to provide energy for motility.

A friend has acid indigestion after eating a big meal. Which of the following would act as a buffer and help ease his pain? -milk of magnesia, with a pH of 10 -black coffee, with a pH of 5 -cola, with a pH of 3 -pure water, with a pH of 7

milk of magnesia, with a pH of 10 Correct Excellent! Excess acid can be buffered by the addition of an alkaline or base.

Prolactin causes __________. myometrial contractions and let-down reflex milk production by the breast tissue uterine contractions only increased hCG excretion during the first month of pregnancy

milk production by the breast tissue right answer feedback:Prolactin stimulates milk production.

The citric acid cycle occurs in the __________ of cells and is an __________ process. -cytosol; aerobic -cytosol; anaerobic -mitochondria; aerobic -mitochondria; anaerobic

mitochondria; aerobic Correct The citric acid cycle occurs in the mitochondria of cells. While the citric acid cycle does not directly use oxygen, it is part of the aerobic pathways for metabolism.

In what organelle would you find acetyl CoA formation, the citric acid cycle, and the electron transport chain? mitochondrion nucleus Golgi apparatus lysosome chloroplast

mitochondrion Correct All of the steps of cellular respiration except glycolysis take place in the mitochondrion.

Which of the following are mechanisms of intrinsic control of glomerular filtration (renal autoregulation)? -myogenic mechanism and tubuloglomerular feedback -sympathetic nervous system control and the renin-angiotensin mechanism -tubuloglomerular feedback and the renin-angiotensin mechanism -myogenic mechanism and sympathetic nervous system control

myogenic mechanism and tubuloglomerular feedback Correct Both of these mechanisms occur strictly within kidney (i.e., intrinsic controls). The myogenic mechanism is mediated by smooth muscle within the afferent arteriole. In contrast, tubuloglomerular feedback is mediated by macula densa cells of the juxtaglomerular apparatus (JGA).

The muscular layer of the uterine wall is called the __________. epimetrium myometrium endometrium perimetrium

myometrium right answer feedback:The muscular layer of the uterine wall is the myometrium (myo- means "muscle").

All of the following structures are derived from mesoderm EXCEPT __________. the liver nails cardiac muscle smooth muscle bone

nails right answer feedback:Mesoderm specializes into numerous structures located throughout the body. Nails, hair and the epidermis, however, are derived from ectoderm.

An important factor directly affecting the glomerular filtration rate is __________. negative pressure blood osmotic pressure capsular osmotic pressure capsular hydrostatic pressure net filtration pressure

net filtration pressure

If the osmotic pressure in the glomerular capillaries increased from 28 mm Hg to 35 mm Hg, would net filtration increase or decrease? -net filtration would increase -net filtration would not be altered =net filtration would decrease

net filtration would decrease Correct Yes, because osmotic pressure opposes filtration, increasing osmotic pressure would decrease net filtration.

Which net pressure draws fluid into the capillary?

net osmotic pressure Correct Yes, the proteins exert colloid osmotic pressure, which draws fluid into the capillary.

The first major event of organogenesis is __________. mesenchyme production amnion formation sporulation neurulation

neurulation Neurulation is the first event of organogenesis. Sporulation does not take place in humans. Neurulation is the first event of organogenesis. Amnion formation is not the first event in organogenesis. Neurulation is the first event of organogenesis. The formation of mesenchyme is not the first event in organogenesis. Neurulation is the first event of organogenesis.

What is the limiting factor for the reabsorption of most actively transported solutes in the proximal tubule? -number of sodium-potassium ATPase pumps in the basolateral membrane -number of transport carriers in the luminal membrane -number of transport carriers in the basolateral membrane

number of transport carriers in the luminal membrane Correct Yes, the number of carriers is the rate-limiting factor. For example, in Diabetes Mellitus, plasma glucose levels are very high, and the sodium-glucose transporter cannot transport (reabsorb) all the glucose passing through the proximal tubule. Glucose is therefore found in the urine.

Sperm are known to bear __________ that respond to chemical stimuli that help them locate the oocyte. tails hydrolytic enzymes olfactory receptors acrosomes

olfactory receptors Sperm are known to bear olfactory receptors that respond to chemical stimuli that help them locate the oocyte. The sperm has a tail that allows it to be motile, but it does not play a role in helping it locate the oocyte. The hydrolytic enzymes help the sperm penetrate the layers around the oocyte during fertilization, but they do not play a role in locating the oocyte. Acrosomes contain hydrolytic enzymes that help the sperm penetrate the membrane around the secondary oocyte, but they do not play a role in locating the oocyte.

Phagocytes are unable to adhere to bacteria that have external capsules concealing their membrane carbohydrates. Our immune system gets around this problem by coating such pathogens with __________.

opsonins Correct Opsonins are complement proteins or antibodies; both provide "handles" to which phagocyte receptors can bind. Any pathogen can be coated with opsonins, a process called opsonization ("to make tasty"), which greatly accelerates phagocytosis of that pathogen.

What pressure is responsible for reabsorption and for pulling fluids into the venous end of capillaries?

osmotic pressure in capillary (OPc) Correct Osmotic pressure is based on the concentration of dissolved materials that attract water across a membrane. The osmotic pressure in a capillary (OPc) is the force drawing fluid into the blood at the venous end.

The organ that makes estrogen and progesterone is the __________. ovary pineal gland vestibular gland vagina hypothalamus

ovary right answer feedback:The ovaries, which make estrogen and progesterone, are the primary sex organs of the female.

The corpus luteum is formed at the site of _______.

ovulation

The surge in LH that occurs during the middle of the ovarian cycle triggers __________. FSH release activation of primordial follicles menstruation ovulation uterine-lining secretion

ovulation right answer feedback:The surge in LH that occurs during the middle of the ovarian cycle triggers ovulation, which occurs, on average, on day 14 of the monthly cycle.

When estrogen reaches a certain level in the body, it triggers a positive feedback loop that triggers an LH surge. This triggers ovulation. -stratum functionalis is shed -glandular secretions commence -menstruation begins -ovulation occurs

ovulation occurs Correct When estrogen reaches a certain level in the body, it triggers a positive feedback loop that triggers an LH surge. This triggers ovulation.

Which step in the metabolism of nutrients provides a cell with the greatest gain in ATP? -glycolysis -gluconeogenesis -oxidative phosphorylation -Krebs cycle

oxidative phosphorylation Correct Oxidative phosphorylation is carried out by the electron transport chain at the end of cellular respiration. This process uses the high-energy coenzymes produced in glycolysis and the Krebs cycle to synthesize ATP.

Which of the following is NOT correctly matched? -glycolysis: anaerobic respiration -oxidative phosphorylation: net loss of ATP -chemiosmosis: hydrogen ion gradients power ATP synthase -citric acid cycle: hydrogen atoms from carbon compounds are transferred to energy carriers

oxidative phosphorylation: net loss of ATP Correct Oxidative phosphorylation, the last step in carbohydrate metabolism, generates more ATP than any other step in the process (resulting in a large net gain of ATP).

Which maternally derived hormone induces and controls labor via a positive feedback mechanism? oxytocin progesterone estrogen hCG ADH right answer feedback:

oxytocin right answer feedback:Oxytocin induces and controls labor via a positive feedback mechanism. Oxytocin is produced in the mother's hypothalamus and is secreted by the posterior pituitary.

Which of the following enzymes is important for breaking down protein? pancreatic lipase pancreatic amylase pepsin trypsin

pancreatic lipase

This group of enzymes digests the majority of ingested fat. lingual lipases pepsin and stomach acid bile salts pancreatic lipases

pancreatic lipases Correct Pancreatic lipases break the bonds between fatty acid chains and glycerol after the fats have been emulsified

If a male undergoes a vasectomy, which of the following can no longer take place? -passage of sperm cells from the rete testes to the epididymis duct -passage of sperm cells from the seminiferous tubules to the rete testes -passage of sperm cells from the epididymis duct to the epididymis tail -passage of sperm cells from the epididymis to the urethra

passage of sperm cells from the epididymis to the urethra Correct Since the ductus (vas) deferens is severed during a vasectomy, sperm cells can no longer pass from the epididymis to the penis.

The antivenom used to treat a venomous snake bite is an antibody produced in an animal such as a horse. Suppose these antibodies are injected into a patient who has been bitten by a venomous snake. How would you classify the resulting humoral immunity?

passive immunity, artificially acquired Correct The immunity to venom (usually short lived--the protection ends once the antibodies are naturally degraded by the body) is passive because the patient did not produce the antibodies, and it is artificially acquired since it was injected during a medical procedure.

Health workers working with diphtheria commonly receive a serum with antibodies against the pathogen. What type of immunity would this be?

passive, artificial immunity Correct Passive immunities are conferred when antibodies are given to an individual. Artificially acquired immunity occurs when humans intervene to confer immunity.

What are the two major avenues for identifying carriers of a recessive disorder?

pedigrees and blood tests

The renal __________ is continuous with the ureter. -pelvis -medulla -cortex -glomerulus

pelvis

Which male structure is homologous to the female's clitoris? -pubis -scrotum -prostate gland -epididymis -penis

penis right answer feedback:Like the clitoris, the penis has erectile tissues.

The __________ cushions the kidney and helps attach it to the posterior body wall. fibrous capsule perirenal fat capsule renal pelvis ureter

perirenal fat capsule The perirenal fat capsule cushions the kidney and helps attach it to the posterior body wall.

Which of the following is an organ shared by the respiratory system and the digestive system? -pharynx -trachea -larynx -esophagus

pharynx Correct The funnel-shaped pharynx connects the nasal cavity and mouth superiorly to the larynx and esophagus inferiorly. Commonly called the throat, the pharynx is the site where our respiratory pathway, from nose to larynx, crosses the digestive pathway, from mouth to esophagus.

During which of these stages are the aortic and pulmonary valves open?

phase 2b Correct During phase 2b, the pressure in the ventricles exceeds that of the aorta and pulmonary trunk, so their valves open and allow blood to be ejected.

The expression of genes is called the __________. genome pedigree phenotype genotype karyotype

phenotype The actual genetic makeup is called the genotype, but the expression of the genotype is the phenotype.

The ______________ is an individual's outward appearance, while the ______________ is an individual's genetic makeup

phenotype; genotype

Inspiratory neurons send information to the diaphragm via what nerve? phrenic nerve intercostal nerves glossopharyngeal nerve vegus nerve

phrenic nerve Correct Yes, the phrenic nerve innervates the diaphragm. Stimulation causes the diaphragm to contract (increasing volume and decreasing pressure), thus causing inspiration.

What cells make antibodies?

plasma B cells Correct Yes, the plasma B cells make antibodies in response to specific antigens.

Characteristics that have a continuous phenotypic trait are due to _______ inheritance.

polygene

One of the changes that occurs in the pacemaker potential (unstable resting membrane potential) in the SA node (an autorhythmic cell) is a decreased efflux of what ion?

potassium Correct Yes, if there is a decreased efflux of potassium while there is a normal influx of sodium, the inside of the cell would become less negative. Thus, threshold would be reached. The ability of these autorhythmic cells to spontaneously depolarize is what results in the pacemaker potential.

What is the most abundant cation in intracellular fluid? magnesium sodium bicarbonate calcium potassium

potassium right answer feedback:Potassium is the most abundant cation in the intracellular fluid and is required for essential metabolic activities, influencing the resting membrane potential and normal neuromuscular functioning.

As your skeletal muscles contract during physical activity, more blood is returned to the heart.. Which variable would be affected and what would be the outcome of this action

preload would be increased, which would result in a larger cardiac output Correct More blood returning to the heart would increase the volume of blood in the ventricles at the end of their filling phase (called end diastolic volume, or EDV). A larger EDV results in greater stretching of the myocardium, or a greater preload. Stretching (lengthening) the contractile cells brings them closer to their optimal length, allowing them to produce more force when stimulated to contract. The stronger contraction results in a larger stroke volume, and therefore a larger cardiac output.

Which portion of the penis is removed during circumcision? -prepuce (foreskin) -bulb of the penis -glans penis -crus of the penis

prepuce (foreskin) Correct The prepuce, or foreskin, forms a covering of the glans penis and is removed during circumcision. This procedure may confer some health benefits, particularly in cases of chronic infection, and is also a common religious practice.

An individual diagnosed with celiac disease would be prescribed a gluten-free diet in order to __________. prevent inflammation caused by malabsorption of gluten protein reduce absorption of high-calorie dietary carbohydrates increase vitamin absorption reduce absorption of dietary fats

prevent inflammation caused by malabsorption of gluten protein Correct In a person with celiac disease, gluten breakdown products interact with the immune system in the digestive tract, leading to damage of the intestinal lining.

Cardiac tamponade results in ineffective pumping of blood by the heart. This is because the excessive amount of fluid in the pericardial cavity will ______. -prevent the visceral layer of the serous pericardium from properly surrounding the heart -prevent the heart from filling properly with blood -prevent proper oxygenation of the blood -interfere with the ability of this fluid to lubricate the serous membranes

prevent the heart from filling properly with blood Correct The heart is wrapped by a double-walled sac called the pericardium. The pericardial cavity lies between the parietal and visceral layers of the serous pericardium. It is filled with a very small amount of serous fluid. If additional fluid fills the pericardial space, it adds pressure on the outside of the heart, preventing it from filling normally.

During early gastrulation, what structure(s) appear(s) on the dorsal aspect of the embryonic disc and establish(e)s the longitudinal axis of the embryo? coelom notochord primitive streak somites

primitive streak Gastrulation begins when a groove with raised edges called the primitive streak appears on the dorsal aspect of the embryonic disc and establishes the longitudinal axis of the embryo. Somites are derived from mesoderm and specialize into sclerotome, myotome, and dermatome. The notochord is a rod that provides axial support of the embryo and is derived from mesoderm. The coelom is ventral body cavity.

All of the following would occur if there were an increase in plasma osmolality EXCEPT __________. concentrated urine production of large amounts of urine release of ADH thirst

production of large amounts of urine right answer feedback:If the plasma osmolality were to increase, the body would conserve water and scant amounts of concentrated urine would be formed.

Which of the following would increase sodium excretion? -glucocorticoids -aldosterone -progesterone -estrogens

progesterone

In which uterine phase does the functional layer of the endometrium start to rebuild? preovulatory ovulation menstrual secretory proliferative right answer feedback:

proliferative right answer feedback:The functional layer of the endometrium starts to rebuild in the proliferative phase, which is initiated by rising estrogen levels.

Hypercalcemia could cause ________.

prolonged T wave. Correct The T wave on an ECG tracing represents ventricular repolarization. Repolarization requires the net efflux of K+ ions. Recall that changes in normal concentrations of ions (like Ca2+) in the plasma can affect the ability of other ions to move in and out of the cell.

Which of the following is NOT an effect of estrogen? promotes the development of secondary sex characteristics in females helps to sustain the density of the skeleton promotes oogenesis promotes diuresis (water loss)

promotes diuresis (water loss) Estrogen promotes oogenesis, stimulates the growth of the breasts, increases deposits of subcutaneous fat, leads to widening and lightening of the pelvis, promotes the development of secondary sex characteristics in females, sustains the density of the skeleton, and lowers cholesterol. Estrogen does not promote diuresis (water loss), nor does it cause lactation.Progesterone promotes the secretory phase of the uterine cycle, promotes diuresis, and stimulates the production of viscous cervical mucus. During pregnancy, progesterone quiets the myometrium and acts with estrogen to cause mammary glands to achieve their mature milk-producing state.

Genetic variation of individual chromosomes occurs during __________. -metaphase I -anaphase I -prophase I -prophase II

prophase I Correct During prophase I, genetic recombination can occur as arms of homologous chromosomes become spliced with one another.

Chromosomes can exchange genetic information during a process called "crossing over." This occurs when homologous chromosomes are lined up in pairs. When does this happen? -metaphase II -prophase I -anaphase I anaphase II

prophase I Correct During prophase I, homologous chromosomes align with each other and undergo crossing over.

During meiosis, segments of nonsister chromatids can trade places. This recombination of maternal and paternal genetic material is a key feature of meiosis. During what phase of meiosis does recombination occur? -prophase I -Metaphase I -anaphase I -telophase I -meiosis II

prophase I Correct Segments of nonsister chromatids trade places during prophase I, resulting in recombination.

At what point during meiosis do homologous chromosomes pair up? -metaphase I -anaphase I -prophase II -prophase I

prophase I Correct The key event in meiosis, the pairing of homologous chromosomes, happens in prophase I. This sets the stage for halving the amount of DNA during the first cellular division.

The correct order of events during meiosis is -metaphase I, prophase I, telophase I, anaphase I, cytokinesis, meiosis II. -prophase I, anaphase I, metaphase I, telophase I, meiosis II, cytokinesis. -prophase I, anaphase I, telophase I, metaphase I, meiosis II. -prophase I, metaphase I, anaphase I, telophase I, cytokinesis, meiosis II. -metaphase I, prophase I, anaphase I, telophase I, cytokinesis, meiosis II.

prophase I, metaphase I, anaphase I, telophase I, cytokinesis, meiosis II. Correct Meiosis starts with prophase I and continues with metaphase I, anaphase I, telophase I, and cytokinesis. This is followed by meiosis II.

Which of the following is NOT correctly matched with its description? -propulsion: physical breakdown of ingested food in the GI tract -ingestion: taking food into the digestive tract -absorption: transport of chemically digested nutrients into the blood or lymph -mechanical breakdown: churning movements in the GI tract

propulsion: physical breakdown of ingested food in the GI tract Correct Propulsion is the movement of ingested food through the GI tract.

Which of the following are NOT correctly matched? nuclease: DNA and RNA digestion amylase: carbohydrate digestion lipase: fat digestion protease: lipid digestion

protease: lipid digestion

Which substance would NOT normally be expected in urine? potassium protein uric acid sodium water

protein

Which substance would be found in higher concentration if the membrane were damaged? -creatinine -glucose -protein -chloride

protein Correct Yes, large proteins are not normally filtered by a healthy glomerular membrane.

The most important buffer system in the intracellular fluid compartment (ICF) is the __________ buffer system. bicarbonate sulfate phosphate chloride protein

protein right answer feedback:Most of the buffering power of body fluids resides in cells, and most of this reflects the buffering activity of intracellular proteins.

Which of the following digestive processes could be affected by a patient's taking a large amount of antacids? carbohydrate digestion protein digestion lipid digestion nucleic acid digestion

protein digestion Correct Protein digestion is partially accomplished by pepsin, an enzyme that works best in a pH range of 1.5 to 2.5.

Enzymatic breakdown of which of the following compounds doesn't begin until it reaches the stomach? lipids carbohydrates proteins

proteins

The colloid osmotic pressure in the capillary is caused by __________.

proteins in blood Correct Yes, the non-diffusible proteins in the plasma exert the colloid osmotic pressure, which pulls fluid into the capillary.

Where in the nephron does most solute reabsorption occur? -glomerulus -proximal convoluted tubule -collecting duct -distal convoluted tubule

proximal convoluted tubule Correct The proximal convoluted tubule (PCT) is where most reabsorption takes place.

In females, _______ is the stage of life when reproductive hormones first begin surging and reproduction is first possible.

puberty

Which receptors inhibit inspiration during hyperinflation of the lungs? -hypothalamic receptors irritant receptors pulmonary stretch receptors -peripheral chemoreceptors

pulmonary stretch receptors Correct Yes, inspiration stimulates the pulmonary stretch receptors (PSRs), which send input to the respiratory centers, inhibiting further inspiration.

Which type of chemical induces fever?

pyrogens Correct Pyrogens target neurons in the hypothalamus to increase the set point for thermoregulation, raising body temperature above normal (37°C).

What is the most significant direct effect of aldosterone release? -activation of angiotensinogen into angiotensin I -rapid drop in blood pressure -stimulate renin production via juxtaglomerular cells of the kidneys -reabsorption of sodium in kidney tubules

reabsorption of sodium in kidney tubules Correct Aldosterone causes the reabsorption of sodium, and because water follows salt, water is also reabsorbed, increasing blood pressure and volume.

The presence of an incompetent tricuspid valve would have the direct effect of causing ______. -reduced efficiency in the delivery of blood to the lungs -reduced efficiency in the delivery of blood to the myocardium -reduced efficiency in the delivery of blood from the lungs to the heart -reduced efficiency in the delivery of blood from the head to the heart

reduced efficiency in the delivery of blood to the lungs Correct The tricuspid valve separates the right atrium and the right ventricle. It must remain tightly closed during ventricular contraction so blood can be pumped out of the ventricle and into the pulmonary arteries.

ECF osmolarity is primarily dependent on ___________. -water content of the body only -blood pressure -relative quantities of sodium and water -sodium content of the body only

relative quantities of sodium and water Correct ECF osmolality is primarily a measure of the concentration of sodium per kilogram of water. This concentration depends on both water and sodium levels, and a disruption of either affects osmolality.

Although effective in treating erectile dysfunction, Viagra has the side effect of reducing systemic blood pressure by causing ______. -a reduction in cardiac output -a reduction in parasympathetic stimulation of penile arterioles -relaxation of muscle tissue in arteries -decreased blood flow to the ventricular walls

relaxation of muscle tissue in arteries Correct Smooth muscle relaxation would lead to vasodilatation of arteries. Remember that blood pressure is directly affected by blood vessel diameter.

Which of the following is the primary function of the juxtaglomerular complex? concentrating urine releases chemical signals that regulate the rate of filtrate formation a system that protects the nephron from some chemicals found in blood reabsorption of Na+ and other ions

releases chemical signals that regulate the rate of filtrate formation Correct The macula densa cells and granular cells of the juxtaglomerular complex (JGC) release, respectively, vasoactive chemicals and renin. These chemical messengers lead to changes in vasomotor activity and Na+ reabsorption that affect the rate of filtrate formation and systemic blood pressure.

Under normal resting conditions, the __________ arteries deliver one-fourth of the total cardiac output (about 1200 ml) to the kidneys each minute. renal cortical radiate segmental interlobar

renal

All of the following are layers of the filtration membrane in the glomerular membrane EXCEPT the __________. fenestrated endothelium renal capsule visceral layer basement membrane

renal capsule

The glomerular (Bowman's) capsule and glomerulus make up the __________. nephron loop of Henle papilla renal pelvis renal corpuscle

renal corpuscle

The __________ is the darker, reddish-brown area of the kidney that exhibits cone-shaped tissue masses called renal pyramids. renal column renal cortex renal pelvis renal medulla

renal medulla

Which of the following would take the longest time to regulate blood pressure?

renal regulation Correct Renal mechanisms mediate the long-term control of blood pressure by counteracting fluctuations in blood pressure by altering blood volume. For example, if blood pressure is too low, more water will be reabsorbed to the blood from the kidney, thus increasing blood volume and blood pressure

Granular cells of the juxtaglomerular apparatus (JGA) regulate GFR indirectly through which mechanism? myogenic mechanism tubuloglomerular feedback mechanism sympathetic nervous system controls renin-angiotensin mechanism

renin-angiotensin mechanism Correct When systemic blood pressure decreases, granular cells release renin which ultimately causes the formation of angiotensin II. Angiotensin II causes widespread vasoconstriction of systemic arterioles and the increase of blood volume due to aldosterone release.

Which process results in increased blood pressure in response to hormone release? adrenergic response myogenic mechanism renin-angiotensin-aldosterone mechanism tubuloglomerular response countercurrent mechanism

renin-angiotensin-aldosterone mechanism

This lung CANNOT be directly measured using a spirometer -tidal volume -residual volume -inspiratory reserve volume -expiratory reserve volume

residual volume Residual volume refers to the amount of air left in the lungs after a tidal expiration

Someone who is suffocating would develop _________________. -respiratory alkalosis respiratory acidosis -metabolic acidosis -metabolic alkalosis

respiratory acidosis

A patient is admitted to the hospital with the following plasma values: pH = 7.2, pCO2 = 55 mmHg, and HCO3 −− = 30 mEq/L. What is the acid base imbalance? -metabolic alkalosis with respiratory compensation -metabolic acidosis with respiratory compensation -respiratory alkalosis with metabolic compensation -respiratory acidosis with metabolic compensation

respiratory acidosis with metabolic compensation Correct Yes, this is a respiratory acidosis with metabolic compensation. Since pCO2 is above the normal range, it is the cause of the acidosis. HCO3 −− is increased to compensate, which is a metabolic compensation.

Emphysema can lead to which acid/base disturbance? What would be the compensation? -respiratory alkalosis; kidneys will retain more H+ and excrete HCO3 -respiratory acidosis; hyperventilation -respiratory acidosis; kidneys will retain more HCO3 -metabolic acidosis; hypoventilation by the lungs Submit

respiratory acidosis; kidneys will retain more HCO3 Correct Yes, a patient with emphysema would retain CO2, leading to a respiratory acidosis. The kidneys would compensate by retaining HCO3 −− and excreting H+.

Hyperventilation can lead to __________. -metabolic alkalosis -respiratory acidosis -respiratory alkalosis -metabolic acidosis

respiratory alkalosis

A homeostatic control mechanism controls respiration. What acts as the effector(s) in this system? -peripheral chemoreceptors respiratory muscles central chemoreceptors medulla oblongata

respiratory muscles Correct Yes, the respiratory muscles change the volume of the thoracic cavity (and thus the pressure), resulting in inspiration and expiration.

Tubular secretion is important for all EXCEPT which of the following? ridding the body of excess glucose controlling blood pH disposing of substances, such as certain drugs eliminating undesirable substances or end products that have been reabsorbed by passive processes

ridding the body of excess glucose

Which of the following arterial blood levels is the most powerful respiratory stimulant? -rising CO2 levels -arterial pH -low CO2 level -low O2 level

rising CO2 level Correct Of all the chemicals influencing respiration, CO2 is the most potent and the most closely controlled. An elevation of only 5 mmHg in arterial PCO2 doubles alveolar ventilation, even when arterial O2 levels and pH haven't changed. When PO2 and pH are below normal, the response to elevated PCO2 is even greater. On the other hand, arterial PO2 must drop substantially, to at least 60 mmHg, before O2 levels become a major stimulus for increased ventilation.

Electrolyte balance in the body usually refers to the balance of __________. salts proteins carbohydrates lipids

salts

The testes are housed in the _______, which _______.

scrotum; lowers sperm development temperature

Which cell type is ovulated from the ovary? ovum oogonium primary oocyte secondary oocyte tertiary oocyte

secondary oocyte right answer feedback:Secondary oocytes are ovulated from the ovary. Females do not ovulate true egg cells. The secondary oocyte does not complete meiosis II and become a true ovum until a sperm penetrates the cell.

Which of the following GI hormones promotes a pancreatic juice rich in bicarbonate ions? motilin gastrin secretin GIP (gastric inhibitory peptide)

secretin Correct Yes, secretin (from S cells in the duodenum) causes both the liver and pancreas to secrete bicarbonate into the small intestine.

An increase in HCl (hydrochloric acid) arriving in the duodenum would stimulate which hormone that would help to counteract the effects of HCl? CCK (cholecystokinin) gastrin GIP (gastric inhibitory peptide) secretin

secretin Correct Yes, secretin stimulates the duct cells in the pancreas and liver to secrete a bicarbonate-rich solution that will bind hydrogen ions and increase the pH.

What types of antigen do mature T cells normally not recognize?

self-antigens Correct Due to negative selection during maturation, T cells do not recognize self-antigens.

Which of the following structures produces the greatest amount of the secretions found in semen? -prostate -seminal glands -bulbo-urethral glands -epididymis

seminal glands Correct The secretions of the seminal glands account for some 70% of semen volume and function to increase sperm motility and fertilizing ability.

The _______ is (are) responsible for producing seminal fluid.

seminal vesicles and prostate gland

Which of the following shows the correct pathway for sperm cells as they leave the testes? -seminiferous tubule, rete testis, epididymis, ductus deferens -ductus deferens, epididymis, rete testis, seminiferous tubule, -ductus deferens, epididymis, seminiferous tubule, rete testis -rete testis, seminiferous tubule, epididymis, ductus deferens

seminiferous tubule, rete testis, epididymis, ductus deferens Correct This is the correct pathway. Sperm are stored in the tail of the epididymis until they are ejaculated. If they are not ejaculated, they will eventually break down and be phagocytized.

Sperm is produced in the __________. -ejaculatory duct -epididymis -ductus deferens -seminiferous tubules

seminiferous tubules Correct Sperm are made in the seminiferous tubules. Rather, fluid flow in the seminiferous tubules carries sperm as they are made toward an exit point from the testis.

Sperm are produced in the __________. seminiferous tubules prostate gland seminal gland spermatic cord interstitial cells epididymis

seminiferous tubules right answer feedback:Seminiferous tubules are coiled tubes within the testes that function in sperm production.

In order, list the structures sperm will pass through from the testes to the external urethral orifice. -epididymis, seminiferous tubules, ampulla, vas deferens, prostatic urethra, ejaculatory duct, spongy urethra -epididymis, ampulla, seminiferous tubules, vas deferens, spongy urethra, prostatic urethra, ejaculatory duct -seminiferous tubules, epididymis, vas deferens, ampulla, ejaculatory duct, prostatic urethra, spongy urethra ejaculatory duct, epididymis, -seminiferous tubules, vas deferens, spongy urethra, prostatic urethra, ampulla

seminiferous tubules, epididymis, vas deferens, ampulla, ejaculatory duct, prostatic urethra, spongy urethra right answer feedback:Sperm are produced in the seminiferous tubule and exit via the urethra.

What stimulates increased respiration at the beginning of exercise? -decreased plasma oxygen levels -increased hydrogen ion levels -sensory input from receptors in joints, neural input from the motor cortex, and other factors -increased plasma carbon dioxide levels

sensory input from receptors in joints, neural input from the motor cortex, and other factors Correct Yes, at the beginning of exercise, blood gases have not changed; thus, other factors such as anticipation of exercise contribute to the increase in respiration.

The process of implantation generally begins __________ and is usually completed by __________. the day before ovulation; the third day after ovulation the day after ovulation; the third day after ovulation six to seven days after ovulation; the twelfth day after ovulation two to three days after ovulation; the sixth or seventh day after ovulation

six to seven days after ovulation; the twelfth day after ovulation Six to seven days after ovulation, given a properly prepared endometrium, implantation begins. Implantation, when successful, takes about five days; it is usually completed by the twelfth day after ovulation.

Normal arterial blood pH is __________. slightly alkaline slightly acidic strongly alkaline neutral highly acidic

slightly alkaline right answer feedback:Normal arterial blood pH is 7.4, which is slightly alkaline.

Which of the following behaviors would most likely result in an increased alveolar ventilation rate as compared to that of normal breathing? -breathing rapidly into a paper bag -breathing slowly into a paper bag -rapid shallow breathing -slow deep breathing

slow deep breathing Correct Slow breathing provides adequate time for gases to pass into the alveoli, while breathing deeply increases the number of alveoli being utilized. The combination of these factors increases effective ventilation, or alveolar ventilation rate.

In what portion of the gastrointestinal tract does most salt and water absorption occur? small intestine mouth colon stomach

small intestine

Absorption of nutrients primarily occurs in the __________. small intestine stomach large intestine mouth

small intestine Correct Most nutrients are absorbed in the small intestine before the chime reaches the ileum.

Which of the following mechanisms is NOT used to propel lymph through lymphatic vessels?

small, heart-like pumps . Correct There is no direct pumping action in the lymphatic system. Movement of fluid depends on factors such as gravity and body movement.

The most prevalent electrolyte in the extracellular fluid is __________. potassium magnesium sodium calcium chloride phosphate

sodium

Adlosterone causes the reabsorption of _________ in the kidney tubule. -chloride -potassium -sodium -water

sodium Correct The secretion of aldosterone stimulates the synthesis and retention of more sodium channels and sodium-potassium pumps in the kidney tubule, therefore enhancing sodium reabsorption.

What component in body fluids adds to the fluids' electrical conductivity? -lipids -glucose -urea -sodium

sodium Correct Electrolytes such as sodium are so named because they dissociate into ions in water and can create electrical currents in the body fluids.

Which of the electrolyte solutes is most responsible for osmotic pressure in body fluids and cells? -sodium -phosphate -potassium -calcium

sodium Correct Sodium is the most abundant cation in ECF, and the amount of sodium present significantly affects ECF volume and water distribution throughout the body.

The active transport of which ion out of proximal convoluted tubule cells causes the reabsorption of both water and solutes? -potassium -sodium -chloride

sodium Correct Yes, the active transport of sodium out of the cell, across the basolateral membrane into the interstitium, provides the driving force for reabsorption of both water and solutes.

Unlike females, males produce gametes throughout life due to the presence of _______ in their seminiferous tubules.

spermatogonia

Conversion of haploid spermatids to functional sperm is specifically called __________. spermatogenesis meiosis spermiogenesis mitosis

spermiogenesis During spermiogenesis, spermatids are converted to functional sperm: The spermatids lose excess cytoplasm, elongate, and form a tail. The resulting sperm (spermatozoa) have a head, midpiece, and tail.

Which of the following structures makes up most of the male urethral length? spongy urethra membranous urethra prostatic urethra ejaculatory duct

spongy urethra right answer feedback:The spongy urethra makes up 75% of the male urethra.

The __________ is shed during menstruation. stratum basalis stratum functionalis perimetrium myometrium

stratum functionalis The uterine wall has three layers: the perimetrium, myometrium, and endometrium. The endometrium has two layers: the stratum functionalis and the stratum basalis. The stratum functionalis (or functional layer of the endometrium) is shed during menstruation in response to declining levels of progesterone. During the proliferative phase of the uterine cycle, the stratum basalis will regenerate the stratum functionalis.

Which layer of the uterus is the site for implantation of a fertilized egg? -stratum functionalis of the endometrium -myometrium -stratum basalis of the endometrium -perimetrium

stratum functionalis of the endometrium Correct Implantation occurs at the stratum functionalis, the innermost layer of the uterus.

Consider the following characteristics of the cells found in muscle tissue. Which feature is shared by both cardiac muscle and skeletal muscle? -striations -triads -intercalated discs -branched cells

striations Correct Since both cardiac and skeletal muscles possess arrangements of motor proteins in regularly arrayed sarcomeres, they both display the banding known as striations.

What is the testicular target for follicle-stimulating hormone (FSH) -sustentocytes (Sertoli cells) -spermatozoa -interstitial endocrine cells (Leydig cells) -spermatogonia

sustentocytes (Sertoli cells) Correct Sustentocytes in the walls of the seminiferous tubules respond to FSH by releasing androgen-binding protein, which maintains high testosterone levels near developing sperm cells.

Norepinephrine is the neurotransmitter released by which fibers? sympathetic postganglionic fibers interneurons of the enteric nervous system both sympathetic and parasympathetic preganglionic fibers parasympathetic postganglionic fibers

sympathetic postganglionic fibers Correct Yes, norepinephrine is the neurotransmitter of the sympathetic postganglionic fibers. An increase in the sympathetic system would decrease digestion.

Which of the following is NOT a function of the large intestine? absorption of water synthesis of intrinsic factor absorption of electrolytes synthesis of vitamins

synthesis of intrinsic factor Correct Intrinsic factor is produced in the stomach; it is required for absorption of vitamin B12 in the small intestine.

The primary sex organ(s) of the male is/are the __________. prostate epididymis testes penis seminal vesicle

testes right answer feedback:The testes are the primary sex organs of males; they are also called gonads.

Which of the following is considered a primary sex organ in males? scrotum penis testes prostate gland

testes wrong answer feedback:The primary sex organs, or gonads, are the organs responsible for gamete production. In males, the testes are the gamete-producing organs; in females, the ovaries accomplish this important function. The remaining structures of the reproductive system are called accessory reproductive organs.

During anaphase I, what structures separate and move to opposite poles of the cell? nucleoli tetrads nonhomologous chromosomes sister chromatids right answer feedback:

tetrads right answer feedback:The tetrads, composed of replicated homologous chromosomes, will separate and migrate to the poles of the cell during anaphase I.

The most important factor that influences K+ secretion is _________. -the concentration of K+ in the ECF -the concentration of Na+ in blood plasma -water balance in the body -the level of aldosterone in the plasma

the concentration of K+ in the ECF

During inhalation, -the diaphragm and rib muscles contract -the diaphragm relaxes -the volume of the thoracic cavity decreases -oxygen molecules move into the lungs, and carbon dioxide molecules move out of the lungs -air moves up the trachea

the diaphragm and rib muscles contract Correct The contraction of these muscles causes air to enter the lungs.

In order to cause cardiac muscle contraction, the contractile cells must also depolarize. What causes the depolarization of the contractile cells?

the flow of positive ions from adjacent cells Correct Yes, the flow of positive ions from the autorhythmic cells (or adjacent cells) brings the membrane to threshold initiating depolarization of the contractile cell.

Infants are more likely to experience problems regulating acid-base balance because of several factors, including __________. lower amount of fluid intake and output in infants high residual volume of infant lungs the inefficiency of infant kidneys decreased surface area relative to body volume low rate of insensible water loss from skin

the inefficiency of infant kidneys right answer feedback:The kidneys are immature at birth; they are approximately half as proficient at concentrating urine as mature kidneys. Infant kidneys are also inefficient in ridding the body of acids.

If the compliance of the thoracic wall is decreased, ______. -the intrapleural pressure would not decrease normally during inhalation -the intrapulmonary pressure would remain lower than the atmospheric pressure -the airway resistance would be decreased -None of the listed response is correct

the intrapleural pressure would not decrease normally during inhalation Correct As the size of the thoracic cavity increases, so does its volume. This causes intrapleural pressure to go below atmospheric pressure so that air (gases) can move into the lungs during inspiration. If the thoracic cavity cannot change its size (volume), then intrapleural pressure will not decrease and normal air movement will not occur.

Which of the following is the countercurrent multiplier in the kidney? -the loop of Henle of a juxtamedullary nephron -the vasa recta -the glomerular (Bowman's) capsule around the glomerulus -the proximal convoluted tubule

the loop of Henle of a juxtamedullary nephron

Internal and external respiration depends on several factors. Which of the following is NOT an important factor in gas exchange? -the molecular weight of the gas -available surface area -partial pressure of the gases -rate of blood flow through the tissue

the molecular weight of the gas Correct Yes, molecular weight is not an important factor in gas exchange.

The reason glucose is detected in the urine of individuals with uncontrolled diabetes mellitus is that __________. glucose is secreted in the collecting ducts of diabetics glucose cannot be reabsorbed by the kidney glucose is too large to be filtered by the nephron the transport maximum for glucose reabsorption has been exceeded

the transport maximum for glucose reabsorption has been exceeded

Cancer treatments such as chemotherapy target rapidly dividing, cancerous cells, but can also destroy noncancerous cells with a rapid rate of mitosis. Which small intestinal features are LEAST likely to be damaged by chemotherapy? enterocytes microvilli the muscularis layer of the intestine mucosal stem cells

the muscularis layer of the intestine Correct The muscularis is a smooth muscle bilayer. Smooth muscle has a relatively slower rate of mitosis.

Hypersecretion of aldosterone results in hypokalemia, which causes hyperpolarization of neurons; this in turn results in ______. -increased speed of sodium-potassium pump activity in order to compensate for the reduced concentration of potassium ions -decreased plasma membrane permeability to potassium ions -the need for a stronger than normal stimulus in order to trigger an action potential -a craving for more salt in the diet Submit

the need for a stronger than normal stimulus in order to trigger an action potential Correct To trigger an action potential cells must reach threshold (critical electrical value required to open voltage-gated ion channels). If the membrane potential is hyperpolarized and falls below normal resting membrane potential, more cations must enter the cytoplasm for the cell to reach threshold.

Which of the following is NOT a chemical barrier that helps prevent infections?

the pH of the blood Correct Although the blood is usually sterile, the slightly alkaline pH does not prohibit pathogen growth.

Which of the following structures help(s) to keep the testes at a cool temperature? the interstitial endocrine cells the rete testis the testicular arteries the pampiniform plexus

the pampiniform plexus right answer feedback:The pampiniform plexus absorbs heat from testicular arteries, cooling the arterial blood before it enters the testes.

Which of the following has the LEAST influence over the pH of blood plasma? -the phosphate buffer system -respiratory rate -the protein buffer system -the bicarbonate buffer system

the phosphate buffer system

Micturation is ____________. -the release of urine from the bladder via the urethra -a form of glomerular filtration -a mechanism for the concentration urine -a method of tubular reabsorption -the production of urine

the release of urine from the bladder via the urethra CORRECT:Also called urination or voiding, micturition entails emptying the urinary bladder to eliminate metabolic nitrogenous wastes from the body.

What structure in the intrinsic cardiac conduction system determines heart rate? -the cardiac nerve -the sinoatrial (SA) node -the vagus nerve -the atrioventricular (AV) node

the sinoatrial (SA) node Correct The SA node is composed of pacemaker cells that initiate and set the initial pace, or the sinus rhythm, of the heartbeat.

Which of the following structures passes through the inguinal canal? the prostatic urethra the ejaculatory duct the epididymis the spermatic cord

the spermatic cord right answer feedback:The spermatic cord passes through the inguinal canal.

Which of the following is NOT a characteristic of the stomach? -the mucosa is organized into large, longitudinal folds called rugae -the stomach produces a double layered coat of alkaline mucus -the stomach releases enzymes that digest carbohydrates -the muscularis externa has three layers of smooth muscle

the stomach releases enzymes that digest carbohydrates Correct The chief cells of the stomach secrete enzymes that chemically digest lipids and proteins, but not carbohydrates.

Which of the following best describes glomerular filtration rate (GFR)? -the volume of urine leaving the kidneys per minute the volume of filtrate created at ---the glomerulus per liter of blood flowing through the glomerular capillaries -the volume of filtrate created by the kidneys per minute -the volume of blood flowing through the glomerular capillaries per minute

the volume of filtrate created by the kidneys per minute Correct Fluid and small solutes that leave the glomerulus are collectively termed filtrate. Glomerular filtration is driven by glomerular hydrostatic pressure (HPg) and produces ~125 ml of filtrate per minute.

What would be the effect on urine output if sodium channels in the tubule cells were inhibited?

the volume would increase

Which of the following puts infants at increased risk for dehydration? -their low surface area to volume ratio -their inefficient kidneys -their high residual lung volume -their low metabolic rate

their inefficient kidneys

Which statement about vasectomies is FALSE? They involve a minor surgical procedure. The part of the ductus deferens that lies in the scrotum is cut. They are a highly effective method of birth control. They are approximately 10% reversible.

they are approximately 10% reversible right answer feedback:Approximately 50% of vasectomies are reversible.

T cells achieve self-tolerance in the __________.

thymus Correct T cells are "educated" (go through a rigorous selection process) as they mature in the thymus; their goal is to become immunocompetent and self-tolerant. A self-tolerant T lymphocyte will not react to self-antigens so that it does not attack the body's own cells.

Which lymphoid organ(s) serve(s) as the site where T lymphocytes become immunocompetent T cells?

thymus gland Correct The thymus gland is the site where T lymphocyte precursors mature to become immunocompetent T lymphocytes.

Which of the following is not a hormone involved in water or electrolyte balance? thyroxine aldosterone antidiuretic hormone atrial natriuretic peptide

thyroxine Correct Thyroxine is a thyroid hormone that regulates metabolic activities and is not involved in water and electrolyte balance.

Which volumes are combined to provide the inspiratory capacity? -expiratory reserve volume (ERV) and residual volume (RV) -tidal volume (TV), inspiratory reserve volume (IRV), and expiratory reserve volume (ERV) -tidal volume (TV), inspiratory reserve volume (IRV), expiratory reserve volume (ERV), and residual volume (RV) -tidal volume (TV) and inspiratory reserve volume (IRV)

tidal volume (TV) and inspiratory reserve volume (IRV) Correct The inspiratory capacity, which is the total amount of air that can be taken into the lungs after a normal relaxed exhalation, is equal to the tidal volume (TV) plus inspiratory reserve volume (IRV).

What is the role of acetyl CoA in the process of cellular respiration? -to carry electrons to the electron transport chain -to enter the process of glycolysis -to enter the citric acid cycle -to synthesize ATP

to enter the citric acid cycle Correct The citric acid cycle begins with the binding of acetyl CoA to oxaloacetic acid to produce citric acid.

Which of the following is NOT a function of the lymphatic system?

to participate in gas exchange at capillaries Correct While the lymphatic system does have a function at capillaries, it does not have to do with gas exchange.

What is the function of the blood testis barrier? -to prevent activation of the immune system of the male against the developing sperm -to provide only select nutrients to the developing sperm -to ensure that semen contains only sperm and seminal fluid -to filter out male sex hormones Submit

to prevent activation of the immune system of the male against the developing sperm Correct: A male's immune system would identify sperm as foreign bodies. The blood testis barrier formed by sustentocytes in seminiferous tubules prevents that interaction.

What is the function of meiosis? to produce four cells, each with identical chromosomes to produce cells with double the chromosome number of the original cell to produce cells with half the chromosome number of the original spermatogonium or oogonium to produce genetically identical cells through the two nuclear divisions

to produce cells with half the chromosome number of the original spermatogonium or oogonium Correct Sperm and egg cells have half the chromosome number of the spermatogonium or oogonium. This is necessary so that they can combine to form a 2n zygote.

The reason that the testes are suspended in the scrotum is to __________. to provide a cooler temperature protect sperm from the immune system to reduce the chance of infection from the enteric bacteria place sperm storage sites nearer to the penis create extra space for the sex organs

to provide a cooler temperature right answer feedback:Spermatogenesis requires a cooler temperature than the core body temperature.

What is the primary function of the overall reactions of cellular respiration? -to regenerate ATP -to produce proteins -to oxidize glucose -to metabolize nutrients

to regenerate ATP Correct Cellular respiration takes energy from our food nutrients during glycolysis and the citric acid cycle and transfers that energy into a usable form of energy in ATP.

The only 100% effective method of birth control is __________. total abstinence Depo-Provera vasectomy MAPs right answer feedback:

total abstinence Total abstinence is the only 100% effective method of birth control.

Most solutes that are reabsorbed in the proximal convoluted tubule use which of the following pathways? -transcellular -paracellular

transcellular Yes, most substances are reabsorbed first through the luminal and then through the basolateral membranes of the proximal convoluted tubule.

Macula densa cells of the juxtaglomerular apparatus (JGA) regulate GFR through which intrinsic mechanism? renin-angiotensin mechanism myogenic mechanism tubuloglomerular feedback sympathetic nervous system control

tubuloglomerular feedback Correct The JGA is a region of the nephron where the afferent arteriole and its associated tubule are closely apposed. This anatomical arrangement allows macula densa cells to adjust GFR according to the NaCl concentration in filtrate. This is called tubuloglomerular feedback because it allows the contents of the tubules (tubulo-) to affect the glomerular filtration rate.

Using the same graph as in Part A, what is the average number of oxygens bound to hemoglobin at a saturation of 50%? -two -three -four -one

two Correct Binding of oxygen to an individual hemoglobin protein occurs at one oxygen per hemoglobin representing 25% saturation; two oxygens per hemoglobin, 50%; three oxygens per hemoglobin, 75%; and four oxygens per hemoglobin, 100% saturation. If an average of two oxygens are bound in all hemoglobins collectively, there is 50% saturation, shown at a blood PO2PO2 of 25 mm of Hg.

For successful fertilization to occur, coitus must occur NO more than __________ and NO later than __________. 24 hours before ovulation; two days after two days before ovulation; 24 hours after three days before ovulation; two days after two days before ovulation; two days after

two days before ovulation; 24 hours after The oocyte is viable for 12 to 24 hours after it is cast out of the ovary. The chance of pregnancy drops to almost zero the next day. Most sperm retain their fertilizing power for 24 to 48 hours after ejaculation. Consequently, for successful fertilization to occur, coitus must occur no more than two days before ovulation and no later than 24 hours after.

Which of the following substances is the largest component of urine by weight after water? urea creatinine inulin uric acid

urea

Which structure is the muscular tube that delivers urine to the bladder? prostate ureter urethra renal pelvis papillary duct

ureter

The major source of water loss from the body is __________. sweat urine insensible skin loss insensible respiratory loss feces

urine Urine accounts for roughly 60% of the body's water loss.

What part of the female system is the usual site of fertilization of the ovulated oocyte? -cervical canal -uterine (fallopian tube) -vagina -uterus

uterine (fallopian) tube Correct Eggs that successfully implant in the uterus are first fertilized in the uterine tube. This does not imply a "tubal pregnancy," where the zygote implants in the uterine tube.

The usual site of fertilization is the __________. ovary uterus vulva uterine tube vagina

uterine tube right answer feedback:The usual site of fertilization is the uterine, or fallopian, tube.

Which of the following is an incorrect matching of female reproductive structures with their functions? -vagina; birth control -ovarian follicle: ovum (egg) production -uterus: fertilization of ovum by sperm -uterine tube; transport ovum to uterus

uterus: fertilization of ovum by sperm Correct The uterine tube is the location where fertilization generally occurs. The uterus (womb) is the organ that receives, retains, and nourishes the fertilized ovum during pregnancy.

Which nephron capillary bed specializes in forming concentrated urine? glomerulus efferent arteriole vasa recta peritubular capillaries

vasa recta The vasa recta serve juxtamedullary nephrons, extending deep into the medulla, and play an important role in forming concentrated urine.

Which of the following form successively larger vessels that carry blood toward the heart?

veins Correct Veins carry blood toward the heart and so are said to "join," "merge," and "converge" into the successively larger vessels approaching the heart.

Which of the following is mismatched?

veins: resistance vessels Correct With their large lumens and thin walls, veins are low resistance vessels that can accommodate a fairly large blood volume. Veins are called capacitance vessels and blood reservoirs because they can hold up to 65% of the body's blood supply at any time. With their small diameters, it is the arterioles that are known as resistance vessels.

Reabsorption of fluid into the capillary takes place at the arterial end or venous end of the capillary

venous Correct Yes, because the hydrostatic pressure of blood (which favors filtration out of the capillary) is lowest in the venous end of the capillary.

Once collected, lymph is ultimately transported into __________.

venous circulation Correct From the terminal lymphatic ducts, lymph rejoins venous circulation via the subclavian veins.

What area in the brain sets the respiratory rhythm? -pontine respiratory group (PRG) -dorsal respiratory group (DRG) -ventral respiratory group (VRG) -hypothalamus

ventral respiratory group (VRG) Correct Yes, the VRG is the rhythm-generating center in the medulla.

Repolarization of an autorhythmic cell is due to the opening of which channels?

voltage-gated potassium channels Correct Yes, opening of voltage-gated potassium channels causes positive potassium ions to move out of the cell. This efflux of potassium causes the cell to become more negative inside thus, repolarizing the cell.

ADH causes the reabsorption of ____________ in the kidney tubule. -potassium -sodium -water -chloride

water CORRECT ADH (antidiuretic hormone) causes aquaporins to be inserted into the cell membranes of the collecting duct cells, facilitating the reabsorption of water from the filtrate back into the blood

Focus your attention on parts (3) and (4) of Focus Figure 28.1. Which statement best describes the result of binding and fusion of the sperm's membrane to the oocyte plasma membrane? =They cause an increase in Ca2+levels within the sperm. -They block multiple sperm from entering the oocyte. =They make the sperm's membrane more fragile. -They allow the contents of the sperm to enter the oocyte cytoplasm.

-They allow the contents of the sperm to enter the oocyte cytoplasm. Correct The sperm's membrane binds to the oocyte's sperm-binding receptors. This binding causes the sperm and oocyte plasma membranes to fuse together, allowing the contents of the sperm to enter the oocyte cytoplasm.

The small blue circles in the figure just inside the oocyte membrane represent vesicles containing calcium ions that are spilled into the extracellular space to prevent polyspermy. When does this take place? -just after the sperm head penetrates the corona radiata -just after the sperm head enters the intermembrane space -just after the sperm head penetrates the zona pellucida -just after the sperm's head enters the oocyte membrane

-just after the sperm's head enters the oocyte membrane Correct The release of calcium into the extracellular space occurs just after the sperm head enters the oocyte. The calcium effectively blocks sperm receptors in the zona pellucida.

Which of the following disorders is NOT inherited as a simple recessive trait? cystic fibrosis schizophrenia albinism Down syndrome Tay-Sachs disease

Down syndrome Down syndrome is a chromosome abnormality due to nondisjunction.

Which statement about maternal hormones during pregnancy is accurate? Estrogen levels increase throughout gestation. hCG levels remain stable throughout gestation. hCG levels increase throughout gestation. Progesterone levels decline throughout gestation. Estrogen levels decline throughout gestation.

Estrogen levels increase throughout gestation. right answer feedback:Estrogen and progesterone levels increase throughout pregnancy.

The X and Y sex chromosomes are the best examples of homologous chromosomes. True False

False The X and Y sex chromosomes are not homologous in the true sense. The Y, which contains the gene that determines maleness, is much smaller than the X chromosome. The X bears over 1400 genes, and a disproportionately large number of them provide code for proteins important to brain function. Because the Y carries only 200 genes, it lacks many of the genes present on the X.

The larger the number of offspring, the greater the likelihood that the observed ratios will conform to the values predicted by the Punnett square. True False

True A simple diagram, called the Punnett square, is used to figure out, for a single trait, the possible gene combinations that would result from the mating of parents with known genotypes. The Punnett square predicts only the probability of a particular genotype (and phenotype). The larger the number of offspring, the greater the likelihood that the observed ratios will conform to the predicted values. Question

Which form of inheritance most often results in a recessive trait being expressed more often in males than in females? dominant-recessive inheritance extranuclear inheritance X-linked inheritance polygene inheritance

X-linked inheritance When a gene is carried on the X chromosome, the male will express that trait, even if it is recessive. This is because males have only one X chromosome. Females have two X chromosomes and can usually mask a recessive trait carried on only one of them.

What is the name for the period of time that extends from the last menstrual period until birth, which is approximately 280 days? a gestation period an embryo pregnancy the conceptus

a gestation period Correct The gestation period is the 40 week period of time that includes oocyte maturation, ovulation, fertilization, and embryonic and fetal development.

Genes that are located on the same chromosome are said to be __________. dominant linked syncopated tied crossed

linked Genes that are located on the same chromosome are said to be linked. They are inherited together unless they are broken up during the crossing-over process.

Extrachromosomal inheritance involves genes passed on by the mother's __________. mitochondria ribosomes smooth ER chromosomes Golgi bodies

mitochondria Extrachromosomal inheritance involves genes passed on by the mother's mitochondria. Mutant forms of mitochondrial genes cause some rare genetic diseases.

The appearance of freckles is considered the __________. karyotype genotype genome phenotype

phenotype The appearance of freckles is considered the phenotype.

In a monohybrid cross, what is the predicted ratio of offspring with the dominant phenotype compared to those with the recessive phenotype? -1:2:1 -3:1 -all will have the dominant phenotype -1:1

-3:1 Excellent! The prediction would be 1 homozygous dominant and 2 heterozygous, all of which would have the dominant phenotype, and the remaining 1 would be homozygous recessive and have the recessive phenotype.

Which of the following is NOT a change in female anatomy or physiology during pregnancy? -A tightening of pelvic ligaments to help bear the extra weight of the growing fetus -An increase in blood volume by as much as 40% -Nasal stuffiness and difficulty breathing -Increased urine output because of increased metabolic wastes generated by the mother and fetus

-A tightening of pelvic ligaments to help bear the extra weight of the growing fetus Correct The hormone relaxin relaxes pelvic ligaments and the pubic symphysis, allowing the pelvis to widen in preparation for birth.

Which statement is most correct? -If you have two copies of a dominant gene, you are considered heterozygous. -Alleles are variations of the same gene. -If you only have one copy of a gene, you are homozygous. -If you have two copies of a gene for one trait, you are heterozygous.

-Alleles are variations of the same gene. Alleles for a gene represent all of the variations of a trait for any given gene.

Nicole has the following genotype: Gene 1: Aa Gene 2: BB Gene 3: cc Based on this, which of the following statements is correct? -Nicole is heterozygous only for gene 1. -Nicole is heterozygous for genes 1 and 3. -Nicole is homozygous only for gene 3. -Nicole is homozygous for genes 1 and 2.

-Nicole is heterozygous only for gene 1. Excellent! Heterozygous means she has two different alleles for the same gene.

An individual has two of the same alleles for a gene for eye color. One of her eyes is deep blue, and the other is a lighter blue. Which of the following is true? -She is homozygous for that gene. -She is homozygous for the phenotype. -She is homozygous for that gene, but heterozygous for the phenotype. -She is heterozygous for that gene.

-She is homozygous for that gene. Excellent! She has two copies of the same allele for this gene, which makes her homozygous for that gene. There are a variety of reasons why she may have two different eye colors, but they are not related to zygosity.

Focus your attention on part (2) of Focus Figure 28.1. Which of the following statements about the acrosomal reaction is correct? -The acrosomal reactions of many sperm expose the sperm membrane. -The acrosomal enzymes released from many sperm digest holes through the zona pellucida. -Binding of sperm to the sperm-binding receptors in the corona radiata causes the Ca2+ levels within the sperm to increase. -The decrease of Ca2+Ca2+ levels

-The acrosomal enzymes released from many sperm digest holes through the zona pellucida. Correct After penetrating the corona radiata, sperm bind to ZP3 (zona protein 3) glycoprotein receptors found within the zona pellucida to trigger the acrosomal reaction. The acrosomal reaction involves release of acrosomal enzymes from the sperm to digest holes in the zona pellucida. Hundreds of sperm must release acrosomal enzymes that digest the zona pellucida and clear a path that enables the fertilizing sperm to reach the oocyte plasma membrane receptors.

Which of the following does not match the term to its correct definition/description? -recessive: gene expressed when present -genotype: genetic makeup of an individual -phenotype: physical appearance of an individual -heterozygous: two different alleles are present Submit

-recessive: gene expressed when present Recessive traits are only expressed if a person has two recessive alleles.

A couple's first child has cystic fibrosis. The gene for cystic fibrosis is autosomal recessive, and the mode of inheritance is simple dominant/recessive inheritance. Neither parent has the disease. What are the odds that the couple's next child will have the disease? All their children will have the disease because both parents carry the gene for cystic fibrosis. Zero, because they have already had an affected child, so their next three children will not have the disease. 1 in 4 Zero, but their remaining children will all be carriers.

1 in 4 The scenario indicates that both parents are asymptomatic carriers. If you construct a Punnett diagram, you will find that the odds of any offspring of this coupling has a 1 in 4 chance of inheriting the two recessive genes, leading to the disease.

Focus your attention on part (5) of Focus Figure 28.1. Drag and drop the terms to the appropriate blanks in the sentences. Terms may be used once, more than once, or not at all.

1. Entry of the sperm's contents causes Ca2+Ca2+ levels Entry of the sperm's contents causes C a 2 + levels in the oocyte's cytoplasm to rise, triggering the cortical reaction. in the oocyte's cytoplasm to rise, triggering the cortical reaction. 2. The cortical reaction The cortical reaction involves the exocytosis of cortical granules near the oocyte plasma membrane. involves the exocytosis of cortical granules near the oocyte plasma membrane. 3. Polyspermy occurs when multiple Polyspermy occurs when multiple sperm enter the oocyte. Monospermy occurs when a single sperm enters the oocyte. sperm enter the oocyte. Monospermy occurs when a single sperm enters the oocyte. 4. In humans, after a single sperm enters the cytoplasm In humans, after a single sperm enters the cytoplasm, further sperm entry is prevented. , further sperm entry is prevented. 5. Zonal inhibiting proteins (ZIPs), enzymes from the cortical granules, cause the zona pellucida Zonal inhibiting proteins (ZIPs), enzymes from the cortical granules, cause the zona pellucida to harden and the zona pellucida to be clipped off. to harden and the sperm receptors Zonal inhibiting proteins (ZIPs), enzymes from the cortical granules, cause the zona pellucida to harden and the zona pellucida to be clipped off. to be clipped off. 6. The block of polyspermy The block of polyspermy occurs as a result of the cortical reaction, by detaching any sperm bound to the receptors and preventing additional sperm from entering. occurs as a result of the cortical reaction, by detaching any sperm bound to the receptors and preventing additional sperm from entering.

Focus your attention on part (1) of Focus Figure 28.1. Drag and drop the terms to the appropriate blanks in the sentences. Terms may be used once, more than once, or not at all.

1. Sperm freshly deposited in the vagina are incapable of penetrating an oocyte. 2. As sperm travel through the female reproductive tract, secretions of the cervix, uterus and uterine tubes capacitate the sperm by making their membranes more fragile and capable of releasing enzymes. 3. When capacitated sperm approach a secondary oocyte, they are able to weave past granulosa cells of the corona radiata with the aid of hyaluronidase enzymes present on their surface.

With the exception of gametes, human cells have __________ pairs of chromosomes. 12 23 46 55 60

23 Most human cells are diploid, containing 23 pairs of chromosomes, for a total of 46 chromosomes per cell. Only the haploid egg and sperm have 23 total chromosomes.

How many chromosomes are in a normal haploid human cell? 10 20 23 46

23 There are 46 chromosomes in a normal human diploid cell. There are 23 chromosomes in a haploid human cell. Gametes (egg and sperm cells) are haploid cells.

Heterozygous parents who have had one child with a recessive disease have a __________% chance of having a second child born with the same recessive disease. 12.5 25 50 75

25 Heterozygous parents who have had one child with a recessive disease have a 25% chance of having a second child born with the same recessive disease.

What is the probability that a child will be born with a recessive trait if both parents are heterozygous for the trait? 10% 25% 50% 75%

25% Each parent has a 50% chance of donating the recessive allele to the baby. 50% × 50% = 25%

If two parents are both heterozygous for an autosomal dominant trait, what is the probability that they will have a child with the recessive form of that trait? 0% 50% 25% 75%

25% Since both parents are heterozygous, they each have a 50%% chance of contributing the recessive allele. A Punnett square demonstrates that the likelihood of a child receiving the recessive allele from both parents is one in four, or 25%%.

The number of different gametes that can be produced in a male, based on independent assortment alone, equals __________. 2 2^3 2^6 2^23

2^23 The number of possible different gametes resulting from independent assortment during meiosis I is calculated by 2n, where n is the number of homologous pairs. There are 23 homologous pairs of chromosomes in humans; therefore, this is the correct answer.

Consider a dihybrid cross (AaBb x AaBb). How many offspring would be predicted to be heterozygous for both traits? 9/16 1/2 3/4 1/4

3/4

An organism with five pairs of homologous chromosomes could produce __________ different gametes on the basis of independent assortment alone. 6 8 32 45

32 The number of different gamete types resulting from independent assortment of homologous chromosomes during meiosis I can be calculated for any genome from the formula 2n, where n is the number of homologous pairs. For five pairs of homologous chromosomes, it would be 2 × 2 × 2 × 2 × 2 = 32.

Use the following information to solve the problem using a Punnett square: Assume that the dominant B allele encodes brown eyes and the recessive b allele encodes blue eyes. An individual with the genotype bb has children with an individual of the genotype Bb. What is the percent chance that they would have children with blue eyes? 0% 25% 50% 100%

50% According to the assumptions in the problem, in order for a child to have blue eyes he or she must have a bb genotype. A Punnett square analysis would predict that 50% of the offspring from this couple would have the bb genotype and thereby express the recessive blue eye trait.

Implantation of the blastocyst begins __________ after ovulation. 1 day 3 days 6 to 7 days 3 weeks 1 month

6 to 7 days right answer feedback:It takes about 3 to 4 days after fertilization for the egg to travel to the uterus, where it floats for 2 to 3 days before it begins implantation.

The Apgar score range that indicates a healthy baby is __________. 0 to 2 2 to 4 4 to 6 6 to 8 8 to 10

8 to 10 right answer feedback:An Apgar score of 8 to 10 indicates a healthy baby. Lower scores indicate one or more physiological problems.

A person who inherits the A and the O blood type alleles will possess which blood type? A B AB O

A The A blood type allele is dominant and the O blood type allele is recessive. In the heterozygous state, the O blood type allele is not phenotypically expressed.

Drag and drop the labels into the appropriate blanks in the sentences. Each term will be used once.

After birth, the ductus arteriosus becomes the ligamentum arteriosum. In the newborn, the foramen ovale transitions to a wall-like depression called the fossa ovalis. After birth, the umbilical vein will become the ligamentum teres. The fetal ductus venosus will form the ligamentum venosum in the newborn. The umbilical arteries in the fetus become the median umbilical ligaments in the newborn. Correct The various adaptations in the fetus transition to specific structures in the newborn; several of these adaptations are ligamentous structures, including the ligamentum teres, median umbilical ligaments, ligamentum arteriosus, and the ligamentum venosus. The fetal foramen ovale will become the fossa ovalis in the newborn

Sam has type A blood. The blood type of his partner is not known. Which of the following blood types are not at all possible among their offspring? type O type B type A All of the listed blood types are possible.

All of the listed blood types are possible. Depending on whether Sam is heterozygous with a recessive allele to donate or if he donates the A allele, all of the choices listed are possible for Sam's offspring.

If two parents are heterozygous (Tt) for a trait, which of the following is NOT a possibility for their offspring? Half of their offspring would be heterozygous. All of their offspring would be dominant for the trait. Half of the potential offspring would be genotypically different than the parents. Most of the offspring would have the dominant phenotype.

All of their offspring would be dominant for the trait. One quarter of their potential offspring would be recessive for the trait.

Drag and drop the labels onto the figure to identify the structures in the fetus.

Correct The umbilical vessels transport blood between the fetus and placenta, whereas the ductus arteriosus and ductus venosus help blood bypass the fetal lungs and liver, respectively. The foramen ovale is located in the heart and serves to shunt blood from the right atrium to the left atrium, bypassing the lungs.

An allele that is able to mask the expression of its partner allele is said to be __________. homozygous dominant heterozygous recessive

DOMINANT Sometimes, one allele masks or suppresses the expression of its partner, Such am allele is said to be dominant, whereas the allele that is masked is said to be recessive

There are three alleles that determine blood type. IA and IB are codominant, and i is recessive to both of them. Serina has Type B blood. Which of the following might be Serina's genotype? IBIB or ii IBi or IAIB IBIB or IBi IBIB only

IBIB or IBi Excellent! The B allele is dominant to i, so Serina would be Type B if she has at least one copy of the B allele. However, the A allele is codominant, so if she had even one A allele, she would also show that phenotype. For example, IAIB would produce Type AB blood.

What is the function of the hormone hCG (human chorionic gonadotropin)? It inhibits the production of estrogen and progesterone by the ovary. It stimulates the production of FSH by the pituitary. It increases to high levels near birth, helping initiate the birth process. It causes the uterine lining to slough off or shed to maintain pregnancy. It signals the corpus luteum to continue producing estrogen and progesterone to maintain the pregnancy.

It signals the corpus luteum to continue producing estrogen and progesterone to maintain the pregnancy. right answer feedback:hCG from the fetal trophoblast cells signals the corpus luteum of the ovary to continue producing estrogen and progesterone, which maintains the endometrium and prevents menstruation.

Many genetic disorders, such as albinism, cystic fibrosis, and Tay-Sachs disease, are inherited as simple recessive traits.

True Some examples of recessive inheritance are the more desirable genetic condition; for example, normal vision is due to recessive alleles, but astigmatism is a dominant trait. However, most genetic disorders are inherited as simple recessive traits. These include conditions as different as albinism, a lack of skin pigmentation; cystic fibrosis, a condition of excessive mucus production that impairs lung and pancreatic functioning; and Tay-Sachs disease, a disorder of brain lipid metabolism caused by an enzyme deficit that shows itself a few months after birth.

A person without a Y chromosome will __________. be sterile but show male characteristics always show male characteristics always show both male and female characteristics show both male and female characteristics

always show both male and female characteristics The Y chromosome has the SRY gene, which is necessary for the development of male characteristics.

The most common form of fetal testing is __________. amniocentesis gene therapy chorionic villi sampling imprinting

amniocentesis The most common form of fetal testing is amniocentesis, in which fetal cells extracted from the amniotic fluid are tested for abnormalities.

Which of these is the innermost (closest) structure that completely surrounds the embryo? -placenta -amnion -yolk sac -chorion

amnion Correct The amniotic fluid, found in the amnion and intimate with the embryo, protects the embryo from physical trauma and maintains a constant homeostatic temperature.

Implantation is completed after the __________. yolk sac forms nervous system forms amnion has formed blastocyst is entirely surrounded by endometrium corpus luteum deteriorates

blastocyst is entirely surrounded by endometrium right answer feedback:Implantation, which takes about a week, is completed after the blastocyst is entirely surrounded by endometrium (typically 12 days after ovulation).

Suppose that a child received one of the purple chromosomes (top) from their father and one of the green ones (bottom) from their mother. Which of the following would be the correct phenotype for the child? -brown hair and brown eyes. -blonde hair and blue eyes. -blonde hair and brown eyes. -brown hair and blue eyes.

brown hair and brown eyes. The child would be heterozygous for each character, and since brown hair and eyes are both dominant alleles, he or she would have brown hair and brown eyes.

If the allele for brown hair is represented by B, it means that __________. brown hair is a recessive trait brown hair is a dominant trait the gene for brown hair is carried on the paternally derived chromosome the gene for brown hair is carried on the maternally derived chromosome

brown hair is a dominant trait By convention, a dominant allele is represented by a capital letter.

The function of the ductus arteriosus is to __________. deliver oxygen-rich blood to the placenta bypass the coronary circuit deliver oxygen-poor blood to the placenta deliver oxygen-rich blood to the liver bypass the pulmonary circuit

bypass the pulmonary circuit right answer feedback:The function of the ductus arteriosus is to bypass the pulmonary circuit in order to help prevent congestion in the developing lungs.

Heterozygous individuals that can pass on recessive, abnormal conditions even if they do NOT express the disease are referred to as __________. phenotypically challenged carriers zygotic deleterious donators recessively compromised right answer feedback:

carriers While they don't have the disease, they can pass, or "carry," it to their offspring.

Which of the following processes may separate linked genes during meiosis? differentiation independent assortment chiasma, or crossover epigenetics allele segregation

chiasma, or crossover Chiasma, or crossover, separates linked genes during meiosis and increases genetic variation in the offspring.

The outermost extraembryonic membrane is the __________. chorion placenta allantois amnion yolk sac

chorion right answer feedback:The outermost extraembryonic membrane is the chorion, which is part of the placenta.

After fertilization, the zygote goes through a rapid period of cell divisions called __________. blastulation implantation organogenesis cleavage teratogenesis

cleavage right answer feedback:After fertilization, the zygote goes through a rapid period of cell divisions called cleavage, resulting in the blastocyst, which has many smaller cells.

In the human blood type AB, both alleles are expressed and are therefore __________. codominant sex-linked incompletely dominant dominant polygenic right answer feedback:

codominant In the human blood type AB, the alleles are codominant.

Which of these genetic conditions is displayed by this figure if it represents the inheritance of albinism from two heterozygous parents, and only the homozygous recessive offspring expresses the trait? sex-linked inheritance polygenic inheritance complete dominance incomplete dominance

complete dominance Because inheriting a single allele for normal pigmentation causes one to exhibit the ability, the gene is exhibiting complete dominance.

Which of the following is NOT a common metabolic/physiological change that occurs in pregnant women? increased urine production glucose sparing hypermetabolism increased respiratory rate decreased blood pressure

decreased blood pressure right answer feedback:Decreased blood pressure is atypical in pregnant women. Both heart rate and blood pressure increase during pregnancy.

A karyotype is a complete __________. haploid complement display of chromosomes display of autosomes display of sex hormones diploid complement display of homologous chromosome pairs

diploid complement display of homologous chromosome pairs A karyotype is a complete diploid complement display of homologous chromosome pairs.

In dominant-recessive inheritance, if an individual is heterozygous, which phenotype will be expressed? segregated dominant autosomal recessive

dominant Dominant alleles will be expressed in the heterozygous state.

An implantation that takes place in a site other than the uterus is called __________. amniotic ectopic metopic hyadatic phytid right answer feedback:

ectopic right answer feedback:Most ectopic pregnancies occur in the uterine tubes.

A phenocopy occurs when __________. two individuals have exactly the same genetic composition an individual carries two identical alleles environmental influences mimic the effect of a gene individual has an exact copy of another's genotype

environmental influences mimic the effect of a gene A phenocopy occurs when an individual develops a phenotype that is similar to a condition caused by mutant genes but is actually caused by environmental factors and does not have a genetic basis.

A Punnett square CANNOT be used to determine the possible genetic outcomes for which type of inheritance? dominant-recessive inheritance incomplete dominance inheritance multiple-allele inheritance extranuclear inheritance

extranuclear inheritance Extranuclear inheritance refers to the mitochondrial genes passed on almost exclusively from the mother. A Punnett square is used to show the possible alleles both parents can contribute.

Amniocentesis and chorionic villus sampling are both examples of a category of genetic screening called __________. human gene therapy genomic imprinting fetal testing carrier recognition

fetal testing Fetal testing is used when there is a known risk of a genetic disorder. The most common type of fetal testing is amniocentesis, in which a wide-bore needle is inserted into the amniotic sac through the mother's abdominal wall, and about 10 mL of fluid is withdrawn. Chorionic villus sampling (CVS) suctions off bits of the chorionic villi from the placenta for examination. A small tube is inserted through the vagina and cervical canal and guided by ultrasound to an area where a piece of placental tissue can be removed. Karyotyping can be done almost immediately on the rapidly dividing chorionic cells, much faster than with amniocentesis.

Which structure represents the remnants of the atrial fetal shunt? ligamentum teres ductus arteriosus fossa ovalis ligamentum venosum foramen ovale

fossa ovalis right answer feedback:The fossa ovalis represents the remnants of the atrial fetal shunt.

The development of the primary germ layers is called __________. gastrulation organogenesis blastulation placentation cleavage

gastrulation right answer feedback:The development of the primary germ layers is called gastrulation. The three layers that form are the ectoderm, endoderm, and mesoderm.

A type of treatment useful in correcting single-gene disorders is __________. amniocentesis chorionic villi sampling gene therapy carrier recognition

gene therapy The most common approach involves transferring a correct gene via a virus to the affected cells to restore normal function. This is called gene therapy.

Which of the following human conditions best exemplifies polygenic inheritance? height sickle-cell trait red-green color blindness hemophilia

height Height is determined by a number of genes, as well as by nutrition and other factors, so it exemplifies polygenic inheritance.

When the two alleles controlling a trait are different, the individual is __________ for the trait. heterozygous homozygous recessive dominant

heterozygous When the two alleles controlling a trait are different, the individual is heterozygous for the trait

An individual who expresses a recessive phenotype must have what genotype? heterozygous with incomplete dominance heterozygous homozygous dominant homozygous recessive

homozygous recessive Excellent! Genetically, this is the only way to have a recessive phenotype.

The form of inheritance in which the heterozygous state is expressed as an intermediate is __________. dominant-recessive inheritance polygenic inheritance multiple-allele inheritance codominance incomplete dominance sex-linked inheritance

incomplete dominance In incomplete dominance, the heterozygous state is expressed as an intermediate between both homozygous states.

Which of the following factors is NOT considered to be a teratogen? alcohol iron nicotine sedatives anticoagulants

iron right answer feedback:Iron is not a teratogen. Teratogens are substances that can cause severe congenital abnormalities.

If a male inherits a sex-linked gene for color blindness, __________. it will never be expressed it will only be expressed 25% of the time it will be expressed only if two copies are present it will always be expressed

it will always be expressed Color blindness is carried on the X chromosome, and males always express X-linked genes.

Most human traits are determined by __________. nutrition during infancy multiple genes a single gene a single allele

multiple genes Most human traits are determined by multiple genes.

Codominant alleles are present in __________. polygene inheritance multiple-allele inheritance dominant-recessive inheritance sex-linked inheritance wrong answer feedback:

multiple-allele inheritance Although we inherit only two alleles for each gene, some genes exhibit more than two allele forms, leading to a phenomenon called multiple-allele inheritance. For example, three alleles determine the ABO blood types in humans: IA, IB, and i. Each of us receives two of these. The IA and IB alleles are codominant, and both are expressed when present, resulting in the AB blood type. The i allele is recessive to the other two alleles.

A permanent structural change of a gene is called __________. nondisjunction linkage mutation trisomy

mutation A permanent structural change of a gene is called mutation. This change might affect the functioning of a gene.

Which of the following is NOT one of the three basic levels of gene control? mutations epigenetic marks small RNAs protein-coding genes

mutations Our genome is a biochemical system of awesome complexity, with three levels of controls. Protein-coding genes make up only the first level and account for less than 2% of the DNA of a human cell. The second layer appears to be the product of the abundant "RNA-only genes," formerly believed to be "junk," that are found in the non-protein-coding DNA. Small RNAs control timing of programmed cell death during development and can also prevent translation of another gene. Epigenetic marks form the third layer of gene controls. Epigenetic marks, or the lack of them, may predispose a cell for transformation from normal to cancerous, and even slight deviations in the epigenetic marks on specific chromosomes can result in devastating human illness.

Linked genes are __________. on different chromosomes on the same chromosome usually recessive usually dominant

on the same chromosome Genes on the same chromosome are said to be linked because they are transmitted as a unit to daughter cells during mitosis. The linkage of genes does not determine the dominant-recessive pattern of their inheritance.

The umbilical vein carries blood that is rich in __________ from the placenta to the fetus. -metabolic wastes -carbon dioxide -oxygen -nitrogen

oxygen Correct The umbilical cord is comprised of the umbilical vein and two umbilical arteries. Lending to knowledge from the cardiovascular system and a comparison of veins and arteries: Arteries take blood away from the heart and veins return blood back to the heart. Here, the umbilical vessels are in relationship to the fetus; therefore, the umbilical vein transports oxygenated and nutrient-rich blood to the fetus, and the umbilical arteries carry deoxygenated, nutrient-poor blood from the fetus to the placenta.

Which type of inheritance is most likely responsible for a wide, continuous range of phenotypes for a certain trait (such as height) among human adults? dominant-recessive inheritance multiple-allele inheritance sex-linked inheritance polygene inheritance

polygene inheritance Polygene inheritance refers to the inheritance of traits that are controlled by many pairs of alleles. As a result, the many combinations of the different pairs of alleles involved create a wide range of phenotypes.

Traits that display continuous phenotypic variation are usually determined by which form of inheritance? multiple-allele inheritance incomplete dominance sex-linked inheritance polygene inheritance dominant-recessive inheritance

polygene inheritance Traits that display continuous phenotypic variation are usually determined by polygene inheritance.

Which hormone produced by the placenta causes the mother's pubic symphysis to loosen and widen? estrogen progesterone hCG FSH relaxin

relaxin right answer feedback:Relaxin relaxes pelvic ligaments and joints. This increased flexibility eases birth passage.

Red-green color blindness exhibits __________ inheritance. polygene sex-linked multiple-allele dominant-recessive

sex-linked Traits that are associated with the X or Y chromosomes (sex chromosomes) exhibit sex-linked inheritance. Red-green colorblindness is inherited as a sex-linked trait.Blood type is determined by several different alleles, making this a trait that exhibits multiple-allele inheritance. The alleles for blood type exhibit what is called codominance. This means that each inherited allele will equally contribute to the phenotype of the organism.Polygene inheritance depends upon multiple gene pairs at different locations acting in tandem that have a "dose" or additive effect on the phenotype.Dominant-recessive inheritance reflects the interaction of dominant and recessive alleles. Traits that are sex-linked or autosomal can exhibit a dominant-recessive relationship.

Which of the following is an example of a disorder that exhibits incomplete dominance? Tay-Sachs disease syndactyly cystic fibrosis sickle-cell anemia

sickle-cell anemia People with one sickle cell allele usually present with the normal phenotype, but stress can force the expression of the normally recessive trait.

The 46 chromosomes of a zygote come from __________. the egg the sperm the mother and the father

the mother and the father

X-linked recessive traits are expressed more frequently in males because __________. they have two X chromosomes there is no corresponding allele on their Y chromosomes they have more chromosomes than females they have fewer chromosomes than females they have two Y chromosomes right answer feedback:

there is no corresponding allele on their Y chromosomes Males tend to express X-linked conditions more often than females because any defect in the X chromosome (from the mother) has no counterpart on the Y chromosome.

Which of the following is NOT a function of the placenta? respiratory gas transfer nutrient transfer urine formation waste elimination hormone production

urine formation right answer feedback:Urine formation is not a function of the placenta.

With respect to their role in fetal circulation, the ductus venosus, foramen ovale, and ductus arteriosus are examples of a(n) _________________. -vascular shunt -hilum -anastomosis -vascular plexus

vascular shunt Correct The concept of a vascular shunt is one that routes materials from one point to another, bypassing a particular area. In this case, the ductus venosus, foramen ovale, and ductus arteriosus function to bypass blood from the developing fetal liver and lungs.


Ensembles d'études connexes

AP Psych: Chapter 6 - Perception- Selective Attention/Perceptual Illusions/Perceptual Organization

View Set

Substance Abuse intervention and treatment, Final 3

View Set

Rome and the Republic Final Exam

View Set

NCLEX Gastrointestinal review (Saunders questions)

View Set

ch 16 lower leg/ankle/foot conditions

View Set

Nursing Fundamental Exam 2- Evolve questions

View Set

Chapter 15: Nonadaptive Evolution and Speciation

View Set

VAN ED TX- REAL ESTATE FINANCE Unit 2

View Set